Download as pdf or txt
Download as pdf or txt
You are on page 1of 227

INCOME FROM SALARIES

CHARGING SECTION: - Section 15 of Income Tax Act


- Employer-Employee relationship between payer & payee.
- Salary is chargeable to tax on “due” basis or “receipt” basis,
whichever is earlier.
- Advance Salary & Advance against salary: -
Advance salary is taxable when it is received by the employee
irrespective of the fact whether it is due or not. However,
Advance against salary is in nature of loan which is adjusted with
salary over a specified time period. It cannot be taxed as salary.

ALLOWANCES:
A. Duty Based allowances: - Exempt upto amount spent in
performance of the duties of an office. In short, they are taxable
to the extent amount saved.
Examples: Travelling Allowance, Conveyance Allowance, Helper
Allowance, Research Allowance, Uniform Allowance.

B. Partially exempted personal allowances: - (Along with their


exempted amounts)
1. Special Compensatory Allowance: ₹800 or; ₹300 per month
depending on specified locations. ₹7,000 per month in Siachen
area of Jammu and Kashmir.
2. Border Area Allowance/Remote locality Allowance/Difficult
Area Allowance/Disturbed Area Allowance: ₹1,300 or ₹1,100 or
₹1,050 or ₹750 or ₹300 or ₹200 per month depending on
specified locations.
3. Special Compensatory (Tribal Areas/Schedule Areas/ Agency
Areas) Allowance (Specified States): ₹200 per month.

1
4. Any Allowance granted to an employee working in any transport
system to meet his personal expenditure during his duty
performed: 70% of such allowance upto maximum of ₹10,000
per month.
5. Children Education Allowance: ₹100 per month per child upto
maximum of 2 children.
6. Hostel Expenditure Allowance: ₹300 per month per child upto
maximum of 2 children.
7. Compensatory Field Area Allowance (Specified Areas in
specified states): ₹2,600 per month.
8. Compensatory Modified Field Area Allowance (Specified Areas
in specified states) ₹1,000 per month.
9. Counter insurgency allowance: ₹3,900 per month.
10. Transport Allowance granted to employee who is handicapped:
₹3,200 per month.
11. Underground Allowance: ₹800 per month.
12. High Altitude Allowance granted to member of armed forces
operating in high altitude areas:
For altitude of 9,000 to 15,000 feet: ₹1,060 per month
For above 15,000 feet: ₹1,600 per month.
13. Special Compensatory highly active field area allowance granted
to member of armed forces: ₹4,200 per month.
14. Island Duty Allowance granted to members of Armed Forces in
Andaman & Nicobar and Lakshadweep group of islands: ₹3,250
per month.

C. Allowances which are fully exempt

1. Allowances to Supreme Court/High Court Judges.


2. Allowances received from United Nations Organization
(UNO).
3. Compensatory Allowance received by Judge under article
222(2) of the constitution.
4. Sumptuary allowances given to High Court Judges and
Supreme Court Judges.

2
5. Allowances or perquisites payable outside India by
Government to a citizen of India for services rendered
outside India. {Section 10(7)}

D. ANY OTHER PERSONAL ALLOWANCES – Fully Taxable.

Example: Pet Allowance, Tiffin Allowance, Medical Allowance,


etc.

Illustration 1 (Leave Travel Concession):

Rajesh went to Srinagar on a holiday on 15.11.2022 with his wife


and three children (one son is 6 years and twin daughters 3 years).
They went by air economic class and the total cost of tickets paid
by the employer was ₹58,000 (₹43,000 for adults and ₹15,000 for
the minor children). Compute the amount of Leave Travel
Concession exempt.

Will the answer be any different if among his 3 children the twins
are 6 years old and son is 3 years old?

Solution:

Case 1: One son is 6 years and twin daughters 3 years old:

Entire amount of ₹58,000 shall be exempt as for employee and


his spouse, the travel is by economy class. Further, there is case
of multiple births (twins) after the first child and therefore, for all
3 children the exemption of leave encashment can be claimed.

Case 2: Twins are 6 years old and son is 3 years old:

Exemption shall be for employee and his spouse and 2 children.


Exemption cannot be given for all 3 children as first there was
birth of twins and then another child i.e. son.
Hence, Exemption:
3
a. For employee and his spouse: ₹43,000
b. For 2 children (₹15,000 * 2/3): ₹10,000
Total exemption: ₹53,000

Notes:

1. The exemption will be available only for two surviving


children for an individual born after 01.10.1998. There is no
restriction for children born before 01.10.1998 and also in
case of multiple births (twins/triplets) after one child.

2. It is assumed that the leave encashment claimed is within


limit of 2 travels in block of 4 calendar years.

PERQUISITES:

TAX FREE PERQUISITES:

1. Telephone facility provided to employee at his residence


2. Transport facility provided by an employer engaged in
business of carrying of passengers or goods to his employees
either free of charge or at concessional rate.
3. Privilege passes and privilege ticket orders granted by Indian
railways to its employees
4. Perquisites allowed outside India by the Government
5. Employer’s Contribution to Staff Group Insurance Scheme
6. Annual Premium by employer on personal accident policy
effected by him on the life of employee
7. Refreshment provided to employees during working hours in
office premises
8. Subsidized meal provided to an employee during working
hours at office of business premises provided the value of such
meal is upto ₹50

4
9. Recreational facilities including club facilities provided to
employees in general
10. Amount spent on training of employees
11. Leave Travel Concession {Subject to conditions and limits}
12. Medical facilities subject to certain prescribed limits.
13. Rent Free official residence provided to judge of HC or SC
14. Conveyance facilities provided to judges of HC & SC.
15. Value of free or concessional education facilities provided to
children of employee, if the value of such education is upto
₹1,000 per month. {When it exceeds ₹1,000 p.m.per child,
then amount in excess shall be taxable. Alternate view: Entire
amount is taxable.}
16. Interest free or concessional loan if such loans are made
available for medical treatment in respect of prescribed
diseases or; where the amount of loans are not exceeding in
aggregate ₹20,000.
17. Gift given by employer to employee, where the value of such
gift is upto ₹5,000 p.a.in aggregate during the previous year.
{When value exceeds ₹5,000 p.a., then amount in excess of
₹5,000 p.a. is taxable. Alternate View: Entire amount is
taxable.}
18. Use of laptops and computer of employer by employee, even
if it is used for personal use only by the employee.

VALUATION OF TAXABLE PERQUISITES:


1. ACCOMODATION:
PARTICULARS Amount (₹)
a. Valuation of Rent-Free Unfurnished
Accommodation (Note 1) Xxxxx
b. Add: Value of Furniture (Note 2) Xxxxx
c. Therefore, Value of Rent-Free Furnished
Accommodation (a + b) Xxxxxx
d. Less: Rent recovered from employee, if Xxxxxx
any
e. Therefore, Value of Concessional
Furnished accommodation (c-d) Xxxxxxx
5
Valuation of Concessional Furnished Accommodation shall
be:

Value of Rent-Free Unfurnished Accommodation


(-) Rent recovered from employee.

NOTE 1: Valuation of Rent-Free Unfurnished


Accommodation:
Case 1: Accommodation is allotted to Government employee:

Valuation is as per LICENSE FEES

Case 2: Accommodation is allotted to any other employee:

A. Accommodation is owned by the employer:


Population of city as per 2001 census Valuation as %
where the accommodation is allotted of SALARY*
Exceeding 25 lakhs 15
Between 10 – 25 lakhs 10
Below 10 lakhs 7.5

B. Accommodation is taken on rent by the employer:

Actual rent paid by the employer


Or;
15% of salary*.
(Whichever is lower)

Here, Salary* =
Basic salary + Dearness Allowance (Terms of
Employment/Retirement benefit) + Commission (Any) +
Bonus + Any other monetary benefits + Taxable element
of allowances.

6
Note 2: Valuation of Furniture:

Case 1: If furniture is OWNED by the employer:

10% p.a. of the Original Cost of the asset.

Case 2: If furniture is taken on rent by the employer:

Actual Rent paid by the employer.

IF ACCOMMODATION IS PROVIDED BY ANY EMPLOYER IN


A HOTEL:

24% of the Salary paid


Or;
Actual charges payable to such hotel
(Whichever is lower)
Less: Amount recovered from the employee, if any.

Note: It would not be a perquisite if the employee is


provided such accommodation for period not exceeding
aggregate 15 days on his transfer from one place to
another.

2. INTEREST FREE/ CONCESSIONAL LOAN:

It is taxable only if the aggregate of loan is exceeding ₹20,000.


Taxable value is computed as follows:

- Consider the rate of Interest charged by State Bank of India (SBI)


on such loans as on 1st day of the Previous Year.
- Apply it on “maximum outstanding monthly balance” as on last
day of each month.
- Take sum of such perquisite amount for each month.
- Reduce the Interest amount, recovered, if any from the
employee. This will give Value of Concessional Loan.
7
3. USE OF MOVABLE ASSETS:
Motor Car:

Sr. Car Expenses Wholly Partly personal use


No owned/Hired met by Official
by use
1. Employer Employer Not a Upto 1.6 litres: ₹1,800 p.m.
perquisite Above 1.6 litres: ₹2,400 p.m.
For Chauffer, Add ₹900 p.m.
2. Employee Employer Not a Actual amount of expenditure
perquisite incurred by the employer as
reduced by the perquisite value
arrived at in (1) above.
3. Employer Employee - Upto 1.6 litres: ₹600 p.m.
Above 1.6 litres: ₹900 p.m.
For Chauffer, Add ₹900 p.m.

Note:

Car is owned by the employer and expenses are also met by the
employer &; such car is used wholly for personal purposes of the
employee:
Actual expenditure incurred by employer on running and
maintenance expenses
Add: Normal wear and tear (computed at 10% p.a. of actual cost of
motor car)
Less: Amount charged from the employee for such use.

Other than Laptop & Motor Cars –


Owned by the employer: 10% p.a. of the actual cost of such asset
Taken on Rent by employer: Actual rent payable by the employer
Note: If any amount is recovered from the employee towards use
of such asset, then it should be reduced to determine the value of
this perquisite.

8
4. TRANSFER OF MOVABLE ASSETS:
PARTICULARS Amount
(₹)
a. Original cost of the asset to the employer xxxxx
b. Less: Normal wear and tear at specified
rate** for each competed year from date of
purchase of asset by employer till its date of
transfer to employee (Note 1) (xxxxx)
c. Therefore, Value of the asset as on date of
transfer to the employee xxxxxx
d. Less: Amount recovered from employee (xxxxxx)
e. Therefore, Value of Perquisite Xxxxxx

Rates of Normal Wear and Tear:

a. Computers and electronic items: 50% on WDV basis


b. Motor Cars: 20% on WDV basis
c. Any other asset: 10% on SLM basis.

5. Medical Benefits/ facilities:

- It is taxable as profits in lieu of salary


- Following medical facilities are exempt from tax:
a. Value of medical treatment provided to an employee or any
member of his family* in any hospital maintained by employer.
b. Reimbursement of expenditure actually incurred on medical
treatment on employee or any of his family* member
- in any hospital maintained by Government/ local
authority/any other hospital approved by the Government
- in respect of prescribed diseases
c. Premium paid to effect an insurance on the health of
employee.
d. Reimbursement of premium paid to effect an insurance on
health of employee or for the family of an employee
e. Amount paid towards expenditure incurred outside India on
medical treatment:
9
i. For medical treatment & stay (stay also includes stay of
attendant): Exempt to the extent amount permitted by RBI.
ii. For Travelling of patient and attendant: Exempt if the Gross
Total Income of the employee before including said
expenditure does not exceed ₹2 lakhs.

Family* means SPOUSE & CHILDREN (dependent or


independent, married or unmarried). It also includes PARENTS,
BROTHERS & SISTERS of the individual if they are wholly and
mainly dependent on him.

Illustration 2 (Valuation of Perquisite of Concessional Loan):


Housing Loan @6% per annum is given to Mr. Bala. Amount outstanding
on 01.04.2022 is ₹6,00,000. Mr. Bala pays ₹12,000 per month towards
principal on 5th of each month. Compute taxable value of perquisite
given that lending rate of State Bank of India (SBI) as on 01.04.2022 is
10% p.a.
Solution:
The perquisite shall be computed at concessional rate of interest that
is 10% p.a. (-) 6% p.a. = 4% p.a.
Computation of Taxable Value of perquisite for the Assessment Year
2023-24 (Relevant to Previous Year 2022-23):
Month Amount Computation Interest @
outstanding at end 4% p.a. i.e.
of the year Value of
(₹) Perquisite
(₹)
April, 5,88,000 5,88,000 * 4% p.a. * 1,960
2022 (6,00,000 (-) 1/12
12,000)
May, 5,76,000 5,76,000 * 4% p.a. * 1,920
2022 (5,88,000 (-) 1/12
12,000)

10
June, 5,64,000 5,64,000 * 4% p.a. * 1,880
2022 (5,76,000 (-) 1/12
12,000)
July, 5,52,000 5,52,000 * 4% p.a. * 1,840
2022 (5,64,000 (-) 1/12
12,000)
August, 5,40,000 5,40,000 * 4% p.a. * 1,800
2022 (5,52,000 (-) 1/12
12,000)
Sept, 5,28,000 5,28,000 * 4% p.a. * 1,760
2022 (5,40,000 (-) 1/12
12,000)
Oct, 5,16,000 5,16,000 * 4% p.a. * 1,720
2022 (5,28,000 (-) 1/12
12,000)
Nov, 5,04,000 5,04,000 * 4% p.a. * 1,680
2022 (5,16,000 (-) 1/12
12,000)
Dec, 4,92,000 4,92,000 * 4% p.a. * 1,640
2022 (5,04,000 (-) 1/12
12,000)
Jan, 4,80,000 4,80,000 * 4% p.a. * 1,600
2023 (4,92,000 (-) 1/12
12,000)
Feb, 4,68,000 4,68,000 * 4% p.a. * 1,560
2023 (4,80,000 (-) 1/12
12,000)
Mar, 4,56,000 4,56,000 * 4% p.a. * 1,520
2023 (4,68,000 (-) 1/12
12,000)
Total Value of 20,880
Perquisite

11
Illustration 3 (Medical Benefits):
Ms. Rakhi is an employee in a private company. She receives the
following medical benefits from the company during the previous year
2022-23:
1. Reimbursement of following medical expenses incurred by Ms. Rakhi
a. On treatment of her self-employed daughter in a Private Clinic:
₹4,000
b. On treatment of herself by Family Doctor: ₹8,000
c. On treatment of her mother-in-law dependent on her in a nursing
home: ₹5,000
2. Payment of premium on Mediclaim policy taken on her health
₹7,500.
3. Medical Allowance: ₹2,000 per month.
4. Medical Expenses reimbursed on her son’s treatment in a
Government Hospital: ₹5,000
5. Expenses incurred by company on the treatment of her minor son
abroad: ₹1,05,000
6. Expenses in relation to stay of Rakhi and her minor son abroad for
medical treatment: ₹1,20,000
(Limit prescribed by RBI for this is ₹2,00,000)
Discuss about the taxability of above benefits and allowance in hands
of Rakhi.
Solution:
Taxability of medical benefits and allowances in the hands of
Ms.Rakhi for the Assessment Year 2023-24:
1. Reimbursement of Medical expenses (Fully Taxable): ₹17,000
(₹4,000 + ₹8,000 + ₹5,000)
2. Payment of premium on Mediclaim policy taken on health of Rakhi: -
(Not Taxable)
12
3. Medical Allowance (Fully Taxable, being an allowance personal in
nature, ₹2,000 * 12): ₹24,000
4. Medical expenses reimbursed on her son’s treatment in a
Government Hospital (Not Taxable)
5. Expenses incurred by company on treatment of her minor son abroad
(Not Taxable)
6. Expenses in relation to stay of Rakhi and her minor son abroad for
medical treatment (Not Taxable)

TAX TREATMENT OF PROVIDENT FUNDS:


PARTICULARS STATUTORY RECOGNISED UNRECOGNISED
Provident Fund Provident Fund Provident Fund
(SPF) (RPF) (UPF)
Employer’s Fully Exempt Exempt upto Fully Exempt
Contribution 12% of salary
(Note 1)
Employee’s Eligible for Eligible for Not eligible for
Contribution deduction u/s 80C deduction u/s deduction
80C
Interest Credited to PF Fully Exempt Exempt upto Fully Exempt
on employer’s 9.5%
contribution
Interest Credited to PF Exempt upto Exempt upto Fully Exempt
on employee’s certain limit 9.5%
contribution
Amount received on Fully Exempt Exempt on Fully Taxable
maturity satisfaction of (Note 2)
conditions

Notes:
1. Salary for the purpose of RPF Exemption = Basic + Dearness Allowance
(Terms of employment/ Retirement benefits) + Commission (% of
sales)

13
2. Amount on maturity received out of Employer’s own contribution and
interest on it is taxable under the head salaries. Interest on Employee’s
contribution is taxable under the head Income from Other Sources.
Employee’s contribution received back is not taxable as it is not an
income.

Illustration 4 (Perquisite Valuation – Accommodation, gift, motor Car,


Medical Insurance + RPF):

Ms. Jaya is the marketing manager in XYZ Ltd. She gives you the
following particulars:

a. Basic Salary: ₹65,000 per month


b. Dearness Allowance: ₹22,000 per month (30% is for retirement
benefits)
c. Bonus: ₹17,000 per month
d. Her employer has provided her with an accommodation on 1st
November, 2022 at a concessional rent. The house was taken on
lease by XYZ Ltd. for ₹12,000 per month. Company recovered
₹4,800 per month from salary of Ms. Jaya.
e. The employer gave her a gift voucher of ₹8,000 on her birthday.
f. She contributes 18% of her salary (Basic + DA) towards Recognised
Provident Fund and the company contributes the same amount.
g. The company pays medical insurance premium to effect insurance
on the health of Ms. Jaya: ₹18,000
h. Motor Car owned by the employer (Cubic capacity exceeds 1.6 litre)
provided to Ms. Jaya from 1st November, 2022, which is used for
both official and personal purposes. The motor car repair and
running expenses of ₹50,000 were fully met by the company. The
motor car was self-driven by the employee.
Compute the Income chargeable to tax under the head salaries in
the hands of Ms. Jaya for the Assessment Year 2023-24.

14
Solution:

Computation of Income chargeable to tax under the head salaries


in the hands of Ms. Jaya for the Assessment Year 2023-24
(Relevant to Previous Year 2022-23):
PARTICULARS Amount (₹) Amount (₹)
a. Basic Salary (₹65,000 * 12) 7,80,000
b. Dearness Allowance (₹22,000 * 2,64,000
12)
c. Bonus (₹17,000 * 12) 2,04,000
d. Valuation of Concessional
Unfurnished Accommodation
(Note 1) 36,000
e. Gift Voucher (₹8,000 - ₹5,000) 3,000
f. Medical Insurance Premium of
employee paid by employer
(Tax Free) -
g. Value of Motor Car (₹2,400 * 12) 12,000
h. Employer’s Contribution to RPF
18% {7,80,000 +
(2,64,000*30%)} 1,54,656
Less: Exempt upto 12% of salary
12% {7,80,000 +
(2,64,000*30%)} (1,03,104) 51,552
i. Therefore, Gross Taxable Salary 13,50,552
j. Less: Deductions under section
16
(ia) Standard Deduction 50,000 (50,000)
i. Therefore, Net Taxable Salary 13,00,552

Note:
Valuation of Concessional Unfurnished Accommodation:
The accommodation is taken on rent by the employer. Hence, Valuation
shall be lower of the following:
Rent paid by the employer (₹12,000 * 5) ₹60,000
Or;
15
15% of Salary (15% of ₹4,43,000) ₹66,450
Therefore, Value of Rent-Free Unfurnished Accommodation:
₹60,000
Less: Rent recovered from employee (₹4,800 * 5) -₹24,000
Therefore, Value of Perquisite ₹36,000

Note:
Here, Salary = Basic + DA (TOE) + Bonus
= (₹65,000 * 5) + (₹22,000 * 30% * 5) + (₹17,000 * 5)
= ₹4,43,000
RETIREMENT BENEFITS:
1. PENSION:
A. UNCOMMUTED PENSION:
It is pension received periodically (monthly basis). It is FULLY
TAXABLE.
B. COMMUTED PENSION:
It means lumpsum amount taken by inter-changing the whole or
part of uncommuted pension. It is converting the future right to
receive pension into a lumpsum amount receivable immediately.
Exemption:
i. For employees of Central Govt./ Local Authorities/
Statutory Corporation/ Members of Civil services/
Defense services/Judges of SC & HC:
FULLY EXEMPT.
ii. Other Employees:
- If employee is in receipt of Gratuity also: 1/3rd of such
pension had he commuted 100%.
- If employee is not receiving gratuity: 1/2 of such pension,
had he commuted 100%.
16
GRATUITY:

A. Received while still in employment: FULLY TAXABLE.

B. Received at time of retirement:


o By Central Government employees/Members of Civil
Service/Local Authority employees: Fully Exempt
o Other Employees:
a. Covered by Payment of Gratuity Act:
Exemption shall be least of following:
1. 15/26 * last drawn salary (Note 1) * No. of years of
service (Note 2)
2. Actual Gratuity received
3. ₹20,00,000

Note 1:
Salary = Basic + DA (Any)

Note 2:
6 months or part thereof is taken as year. E.g.: - 26 years &
7 months = 27 years

b. Not covered by Payment of Gratuity Act:


Exemption shall be least of the following:
1. 1/2 * Average Salary of last 10 months (Note 3) * No.
of completed years of service (Fraction of year to be
neglected.)
2. Actual Gratuity received
3. ₹20,00,000
Note 3:
Salary = Basic + DA (Terms of Employment/Retirement
Benefits) + Commission (% of sales)

17
Illustration 5 (Gratuity – Covered by Payment of Gratuity Act, Pension,
House Rent Allowance, RPF):
Mr. Anand is an employee of XYZ Company Ltd. at Mumbai and covered
by Payment of Gratuity Act. He retires at age of 64 years on 31.12.2022
after completing 33 years and 7 months of service. At the time of
retirement his employer pays ₹20,51,640 as Gratuity and ₹6,00,000 as
accumulated balance of Recognised Provident Fund. He is also entitled
for monthly pension of ₹8,000. He gets 75% of pension commuted for
₹4,50,000 on 01.02.2023.
Determine the salary chargeable to tax for Mr. Anand for the
Assessment Year 2023-24 with help of the following information:
1. Basic Salary (₹80,000 * 9): ₹7,20,000
2. Bonus: ₹36,000
3. House Rent Allowance (₹15,000 * 9): ₹1,35,000
4. Rent paid by Mr. Anand (₹10,000 * 9): 90,000
5. Employer’s Contribution towards RPF: ₹1,10,000
6. Professional Tax paid by Mr. Anand: ₹2,000
Note: Salary and Pension falls due on last day of each month.
Solution:
Computation of Taxable Salary of Mr. Anand for the Assessment Year
2023-24 (Relevant to Previous Year 2022-23):
PARTICULARS Amount (₹) Amount (₹)
a. Basic Salary (₹80,000 * 9) 7,20,000
b. Bonus 36,000
c. House Rent Allowance 1,35,000
Less: Exempted (Note 1) (18,000) 1,17,000
d. Employer’s Contribution
towards RPF 1,10,000
Less: Exempt (Note 2) (86,400) 23,600
e. Gratuity 20,51,640
Less: Exempted (Note 3) (15,69,231) 4,82,409
f. Commuted Pension 4,50,000
Less: Exempted (Note 4) (2,00,000) 2,50,000
18
g. Uncommuted Pension (Note 5) 12,000
h. Accumulated balance of
Recognised Provident Fund
(Fully Exempt as received after 5
years of service) -
i. Therefore, Gross Taxable Salary 16,41,009
j. Less: Deductions under section
16
(ia) Standard Deduction 50,000
(iii) Professional Tax 2,000 (52,000)
k. Therefore, Net Taxable Salary 15,89,009

Notes:
1. House Rent Allowance exemption:

Least of following:

a. Actual House Rent Allowance received: ₹1,35,000


b. Rent Paid (-) 10% of Salary
(10,000 * 9) – (10% of ₹7,20,000 ) ₹18,000
c. 50% of Salary ₹3,60,000
(50% of ₹7,20,000)

Therefore, Exemption shall be ₹18,000

Here,
Salary = Basic + DA (Terms of Employment) + Commission (% of
sales)
i.e. ₹7,20,000 + Nil + Nil = ₹7,20,000

2. Exemption of Employer’s Contribution to Recognised Provident


Fund:

12% of salary
i.e. 12% of ₹7,20,000 = ₹86,400

19
Here,
Salary = Basic + DA (Terms of Employment) + Commission (% of
sales)
i.e. ₹7,20,000 + Nil + Nil = ₹7,20,000

3. Exemption of Gratuity – Covered by Payment of Gratuity Act

Least of following:

a. 15/26 * Last Drawn Salary * No. of years of service


15/26 * 80,000 * 34 = ₹15,69,231

b. Actual Gratuity received = ₹20,51,640

c. ₹20,00,000

Therefore, Exemption shall be ₹15,69,231 being the least of


following.

4. Exemption of Commuted Pension:

He is also receiving gratuity; therefore, Exemption shall be 1/3rd


of such pension had he commuted 100%.

Amount received if 100% was commuted:

For 75% = ₹4,50,000


For 100% = x

Therefore, x = ₹4,50,000 * 100/75 = ₹6,00,000


Exemption = 1/3rd of ₹6,00,000 = ₹2,00,000

5. Taxable Uncommuted Pension:

a. Received in January 2023: ₹8,000


b. Received in February & March, 2023 ₹4,000
20
(₹8,000 * 25% being uncommuted element * 2)
c. Therefore, Total Taxable Uncommuted Pension ₹12,000

Illustration 6 (Gratuity – Not covered by Payment of Gratuity Act):


Mr. Shah, an Accounts manager has retired from JK Ltd. on 15.01.2023
after rendering services for 30 years 7 months. His salary is ₹25,000 per
month upto 30.09.2022 and ₹27,000 thereafter. He also gets ₹2,000 per
month as Dearness Allowance (55% of it is a part of salary for computing
retirement benefits.) He is not covered by Payment of Gratuity Act,
1972. He has received ₹8,00,000 as Gratuity from his employer
company. Compute the exemption under section 10(10) and Income
under the head salary.
Solution:
Computation of Net Taxable Salaries for the Assessment Year 2023-24
(Relevant to Previous Year 2022-23):
PARTICULARS Amount (₹) Amount (₹)
a. Basic Salary
(₹25,000 * 6) + (₹27,000 * 3.5) 2,44,500
b. Dearness Allowance
(₹2,000 * 9.5) 19,000
c. Gratuity
Amount received 8,00,000
Less: Exempt (Working Note) (4,00,500) 3,99,500
d. Therefore, Gross Taxable Salary 6,63,000
e. Less: Deductions under section
16
(ia) Standard Deduction 50,000 (50,000)
f. Therefore, Net Taxable Salary 6,13,000

Working Note:
Gratuity Exempted – Not Covered by Payment of Gratuity Act, 1972
Least of following 3:

21
1. 1/2 * Average Salary for 10 months preceding the month of
retirement * No. of completed years of service
1/2 * ₹26,700 * 30 years = ₹4,00,500
2. Actual Gratuity Amount received = ₹8,00,000
3. ₹20,00,000
Therefore, Exemption shall be ₹4,00,500
Note:
Salary = Basic + Dearness Allowance (Terms of Employment)
PARTICULARS Amount (₹)
a. Basic (₹25,000 * 7) + (₹27,000 * 3) 2,56,000
b. Dearness Allowance (₹2,000 * 10 * 55%) 11,000
c. Therefore, Total Salary for 10 months 2,67,000
d. Average Salary (2,67,000/10) 26,700

3. LEAVE ENCASHMENT:
A. Received during period of service: FULLY TAXABLE
B. Received on Retirement:
- By Government Employees: FULLY EXEMPT
- By other than Government Employees:
Exemption shall be least of following:
1. Leaves standing to the credit of the employee at the time of
retirement (restricted to maximum of 30 days for each year of
service rendered)/30 * Average salary
2. Actual Leave Encashment received
3. ₹3,00,000 (-) leave encashment earlier claimed as exemption
4. 10 * Average Salary
Note:
Salary = Basic + DA (Terms of Employment) + Commission (% of sales)
Average of last 10 months is to be considered.

22
Illustration 7 (Leave Encashment):
Mr.S retired from a company on 31st December, 2022 after serving
for 32 years 9 months. He received ₹2,50,000 towards encashment
of earned leave lying unavailed by him. Other particulars are as
follows:
a. Basic Salary from 1.6.2021 to 31.5.2022 @ ₹18,000 per month
b. Basic Salary from 1.6.2022 to 31.12.2022 @ ₹21,000 per month
c. Dearness Pay (treated as salary) @ 10% of basic salary.
d. Earned Leave due @ 35 days per completed year of service.
e. Earned Leave actually availed during service: 540 days.

Compute exempted and taxable amount as per section 10(10AA) of


Income Tax Act, 1961.

Solution:
Computation of Taxable Leave Encashment for the Assessment Year
2023-24 (Relevant to Previous Year 2022-23):
PARTICULARS Amount (₹) Amount (₹)
a. Leave Encashment received 2,50,000
b. Less: Exemption
Least of following:
1. Leaves Availed standing to the
credit of the employee at the
time of retirement * Average
Salary (Note) 3,04,920
2. 10 * Average Salary
(10 * 21,780) 2,17,800
3. Actual Leave Encashment
received 2,50,000
4. Amount specified 3,00,000
c. Therefore, Exemption (2,17,800)
d. Therefore, Taxable Leave
Encashment 32,200

23
Note:
AVERAGE SALARY
Here, Salary = Basic + DA (TOE) +
Commission (% of sales)
a. Basic Salary from 1.2.2022 – 31.5.2022
(₹18,000 * 4) 72,000
b. Basic Salary from 1.6.2022 – 30.11.2022
(₹21,000 * 6) 1,26,000
Total Basic 1,98,000
c. Dearness Allowance (10% of Basic) 19,800
d. Total Salary for 10 months 2,17,800
e. Therefore, Average Salary (2,17,800/10) 21,780

LEAVES STANDING TO CREDIT OF


EMPLOYEE
a. No. of completed years of service 32
b. Leaves Entitlement each year 35 days
c. Restricted to maximum of 30 days in a year
(Therefore, 35 or 30, whichever is lower) 30 days
d. Total no. of leaves entitlement (32 * 30) 960 days
e. Less: Leaves Availed 540 days
f. Therefore, Leaves standing to the credit of
employee at time of retirement 420 days
g. Converted in months (420/30) 14 months

Therefore, 14 months *21,780 ₹3,04,920

RETRENCHMENT COMPENSATION:
It is actually profits in lieu of salary.
Exemption shall be lower of following:
a. 15 days average pay * completed years of service and part thereof
in excess of 6 months
b. ₹5,00,000
Notes:

24
1. The above limits will not be applicable to cases where the
compensation is paid under any scheme approved by the Central
Government for giving special protection to workmen under
certain circumstances.
2. Salary includes all elements including value of perquisites but
excludes bonus, contribution to retirement benefit scheme and
any gratuity payable on termination of his service.

Illustration 8 (Retrenchment Compensation):


Mr. Govind received retrenchment compensation of ₹10,00,000
after 30 years 4 months of service. At the time of retrenchment, he
was receiving basic salary of ₹20,000 per month & dearness
allowance ₹5,000 per month. Compute his taxable retrenchment
compensation.
Solution:
Computation of Taxable amount of Retrenchment Compensation
of Mr. Govind for the Assessment Year 2023-24:
Retrenchment Compensation received: ₹10,00,000
Less: exempted (Note) (₹3,75,000)
Taxable Retrenchment Compensation ₹6,75,000
Note:
Exemption of Retrenchment Compensation:
Lower of the following:
a. 15/30 * 25,000 * 30 ₹3,75,000
b. Amount specified ₹5,00,000
Therefore, Exemption shall be ₹3,75,000
Salary = Basic + Dearness Allowance = ₹20,000 + ₹5,000 = ₹25,000

25
VOLUNTARY RETIREMENT RECEIPTS (VRS):
- Taxable as profits in lieu of salary.
- Exempted to the extent of ₹5,00,000*
- Where exemption for voluntary retirement compensation has
been allowed in any assessment year, then no exemption
thereunder shall be allowed to him in any other assessment year.

PAY SCALE:
Illustration 9 {for computing Basic Salary in case of “PAY
SCALE”}:

Mr. Anand is a regular employee of Malpani Ltd. in Mumbai. He


was appointed on 01.03.2022 in pay scale of ₹25,000 - ₹2,500 -
₹35,000. Compute his basic salary for Assessment Year 2023-24.

Solution:

a. 01.04.2022 – 28.02.2023 (₹25,000 * 11): ₹2,75,000


b. 01.03.2023 – 31.03.2023 (₹27,500 * 1): ₹27,500
Total Basic Salary ₹3,02,500

Deductions from Salary:

1. Standard Deduction {Section 16(ia)}


Allowed to the extent of ₹50,000 or Gross Taxable Salary, whichever is
lower.

2. Entertainment Allowance {Section 16(ii)}:

It is first to be included in the salary and thereafter the following


deduction is to be made from Gross Salary. It is allowed only to
GOVERNMENT EMPLOYEE.

26
Deduction is lower of following:

a. Actual Entertainment Allowance


b. ₹5,000 p.a.
c. 20% of basic salary

3. Profession Tax on employment:

In case, employer has paid profession tax on behalf of employee,


first include it in the Gross Taxable Salary and then deduction is
allowed.

ILLUSTRATION 10 (Deductions u/s 16):

Mr. Goyal received the following emoluments during the Previous


Year ending 31.03.2023.

Basic Pay: ₹40,000


Dearness Allowance: ₹15,000
Commission: ₹10,000
Entertainment Allowance: ₹4,000
Medical Expenses reimbursed: ₹25,000
Profession Tax paid: ₹2,000 (₹1,000 was
paid by his employer)
Mr. Goyal contributes ₹5,000 towards Recognised Provident Fund. He
has no other income. Determine the income from salary for Assessment
Year 2023-24, if Mr. Goyal is a State Government Employee.

Solution:

PARTICULARS Amount (₹) Amount (₹)


a. Basic Salary 40,000
b. Dearness Allowance 15,000
c. Commission 10,000
d. Entertainment Allowance 4,000
e. Medical expenses reimbursed 25,000
27
f. Profession Tax paid by the 1,000
employer
GROSS SALARY 95,000
Less: Deductions under section 16
(ia) Standard Deduction 50,000
(ii) Entertainment Allowance 4,000
(iii) Profession Tax 2,000 (56,000)
Therefore, Income from Salary 39,000

Notes:

1. Deduction of Entertainment Allowance is allowed as employee is


a government employee.
It is allowed to the extent of least of following:
a. Entertainment Allowance received: ₹4,000
b. ₹5,000
c. 20% of basic salary (20% of 40,000): ₹8,000
Hence, it shall be ₹4,000.

2. Employee’s Contribution to RPF is not taxable under the head


salary. It is not an income. It is allowed as deduction under section
80C.
Illustration 11 (Covering Perquisite of Transfer of Assets of employer
to employee):
Compute value of perquisites on transfer of Assets of employer to
employee in following 3 independent cases:
a. Company purchased a Motor Car on 15th February, 2018 for
₹10,00,000. This was transferred to an employee on 3rd May,
2022 for ₹3,00,000.
b. Company purchased computer for ₹75,000 on 1st June, 2020. It
was transferred to employee on 31st May, 2022 for ₹10,000.
c. A camera purchased by company on 8th July, 2019 is transferred
to employee on 8th December, 2022. It’s purchase price was
₹1,20,000 while it is sold to employee for ₹30,000. (Hint: ₹54,000)
28
Solution:
Computation of value of perquisites on transfer of assets by
employer to employee:
a. Motor Car
PARTICULARS Amount (₹)
a. Original cost of the asset to the employer 10,00,000
b. Less: Normal Wear and Tear for Year 1 @ 20% (2,00,000)
c. WDV 8,00,000
d. Less: Normal Wear and Tear for year 2 (1,60,000)
e. WDV 6,40,000
f. Less: Normal Wear and Tear for year 3 (1,28,000)
g. WDV 5,12,000
h. Less: Normal wear and tear for year 4 (1,02,400)
i. Value at the time of transfer 4,09,600
j. Less: Amount recovered from employee (3,00,000)
k. Value of Perquisite 1,09,600
Note: No. of COMPLETED YEARS is considered.

b. Computer:
PARTICULARS Amount (₹)
a. Original cost of the asset to the employer 75,000
b. Less: Normal wear and tear @ 50% (37,500)
c. WDV 37,500
d. Less: Normal wear and tear @ 50% (18,750)
e. Value at time of transfer 18,750
f. Less: Amount recovered from employee (10,000)
g. Therefore, Value of Perquisite 8,750

C. Camera:
PARTICULARS Amount (₹)
a. Original cost of the asset to the employer 1,20,000
b. Less: Normal wear and tear @ 10% SLM (36,000)
c. Value at time of transfer 84,000
d. Less: Amount recovered from employee (30,000)
e. Value of Perquisite 54,000

29
INCOME FROM HOUSE PROPERTY

Charging section: Section 22 of Income Tax Act


“ANNUAL VALUE” of “PROPERTY” “OWNED” by the assessee is
chargeable to tax under this head.
NOTES:
1. Ownership also includes deemed ownership as per section 27.
2. The annual value of property used by the owner for the business
or profession carried on by him shall not be taxable under this
head.
3. Annual Value of portions of property occupied by assessee for
purpose of any business or profession carried on by him will be
taxable under the head PGBP.
4. Also, properties of assessee engaged in business of letting out of
properties is taxable under the head PGBP.
5. Annual Value of property held as stock in trade would be treated
NIL for a period of 2 years from the end of financial year in which
certificate of completion of construction of property is obtained,
if such property is not let out during such period.
FORMAT for Computation of Income from House Property:
PARTICULARS Amount (₹) Amount (₹)
a. Gross Annual Value (GAV) – Note 1 xxxxx
b. Less: Municipal Taxes paid by the
owner of the property (xxxx)
c. Therefore, Net Annual Value Xxxxx
(NAV)
d. Less: Deductions under section 24
(a) Standard Deduction @ 30% of xxxx
NAV
(b) Interest on Borrowed Capital – xxxx (xxxx)
Note 2
e. Therefore, Income from House
Property xxxxx
30
Note 1: GROSS ANNUAL VALUE
Computation of Gross Annual Value:
1. Reasonable Expected Rent
a. Municipal Value
b. Fair Rent
c. Higher of a & b
d. Standard Rent
e. Lower of c & d

2. Annual Rent (Actual Rent * 12)


Less: Unrealized Rent (See Rule 4 – Note 6), if any
xxxxxx

3. Gross Annual Value (Higher of 1 and 2) – if there is no vacancy.


Notes:
1. Where let out property is vacant for part of the year and owing to
vacancy, the actual rent is lower than the expected rent, then the
actual rent received or receivable will be GAV of the property.

2. For any 2 Self-Occupied Properties of the assessee, at his option,


the Net Annual Value is always NIL.

3. Where the property is Let Out for the part of the year and self-
occupied for part of the year, then Reasonable Expected Rent
(RER) for the whole year shall be considered for determining GAV.
(Hence, GAV shall be higher of such RER and Actual Rent). In such
cases, Municipal Taxes for the whole year will be allowed as
deduction.

4. For Deemed To Be Let Out Properties, RER = GAV.

5. In case of a House Property where a portion is let out and portion


is self-occupied, then separate computations for these both
portions considering the areas of each portion.
31
6. Conditions of Rule 4 to be satisfied in order to deduct the
Unrealised Rent from Annual Rent:

a. The tenancy is bonafide.


b. The defaulting tenant has vacated, or steps have been taken
to compel him to vacate the property.
c. The defaulting tenant is not in occupation of any other
property of the assessee.
d. The assessee has taken all reasonable steps to institute legal
proceedings for recovery of the unpaid rent or satisfies the
Assessing Officer that legal proceedings would be useless.

NOTE 2: INTEREST ON BORROWED CAPITAL


Sr. CONDITIONS Amount of deduction
No.
1. Loan Borrowed before 01.04.1999 Actual interest
payable in aggregate
for 1 or 2 self-
occupied properties,
subject to maximum
of ₹30,000.
2. Loan borrowed on or after
01.04.1999 Actual interest for 1
i. Loan for acquisition or construction or 2 Self- Occupied
provided that the construction is properties, subject to
completed within 5 years from end of maximum of
F.Y. in which capital was borrowed. ₹2,00,000.

Actual interest for 1


ii. Loan for Repairs, renewal. or 2 Self- Occupied
properties, subject to
maximum of ₹30,000.
Notes:
1. The total interest deduction under 1 & 2 cannot exceed
₹2,00,000.

32
2. All the above limits are for current year interest + pre-
construction period interest.
PRE-CONSTRUCTION PERIOD INTEREST:
- It is the period prior to the previous year in which the property
is acquired or construction is completed.
- It can be claimed as deduction over a period of 5 years in equal
annual installments commencing from the year of acquisition or
completion of construction.
Illustration 1:
Ganesh has 3 houses, all of which are self-occupied. The particulars of
the houses for the Previous Year 2022-23 are as under:
Figures in ₹
PARTICULARS House 1 House 2 House 3
Municipal Valuation p.a. 3,00,000 3,60,000 3,30,000
Fair Rent p.a. 3,75,000 2,75,000 3,80,000
Standard Rent p.a. 3,50,000 3,70,000 3,75,000
Date of completion/purchase 31.03.1999 31.03.2002 01.04.2015
Municipal Taxes paid during
the year 12% 8% 6%
Interest on money borrowed
for repair of property during
current year - 55,000 -
Interest for current year on
money borrowed in April, 2015
for purchase of property - - 1,75,000
Compute Ganesh’s Income from House Property for the Assessment
Year 2023-24 and suggest which houses should be opted by Ganesh to
be assessed as self-occupied so that his tax liability is minimum.
SOLUTION:
Let us first calculate income from each house property assuming that
they are deemed to be let out.
Computation of Income from House Property of Ganesh for the
Assessment Year 2023-24:
33
Figures in ₹
PARTICULARS House 1 House 2 House 3
Gross Annual Value
1. Reasonable Expected
Rent = GAV
a. Municipal Value 3,00,000 3,60,000 3,30,000
b. Fair Rent 3,75,000 2,75,000 3,80,000
c. Higher of a and b 3,75,000 3,60,000 3,80,000
d. Standard Rent 3,50,000 3,70,000 3,75,000
e. Therefore, 3,50,000 3,60,000 3,75,000
RER=GAV
Less: Municipal Taxes paid (36,000) (28,800) (19,800)
(3,00,000 (3,60,000 * (3,30,000 *
* 12%) 8%) 6%)
Therefore, Net Annual 3,14,000 3,31,200 3,55,200
Value
Less: Deductions under
section 24
(a) Standard Deduction @
30% of NAV (94,200) (99,360) (1,06,560)
(b) Interest on Borrowed
Capital - (55,000) (1,75,000)
Therefore, Income from
House Property 2,19,800 1,76,840 73,640
Ganesh can opt to treat any 2 of the above properties as Self-
Occupied.
Option 1: House 1 & House 2 are SELF OCCUPIED and House 3 is
Deemed to be Let Out
PARTICULARS Amount (₹)
House 1 (Self Occupied) Nil
House 2 (Self Occupied) – Interest on borrowed
capital restricted to ₹30,000 as the Loan is for repaid
of House Property (30,000)
House 3 (Deemed to be let out) 73,640
Therefore, Income from House Property 43,640

34
OPTION 2: House 1 and House 3 are self-occupied and House 2 is
Deemed to be Let Out:
PARTICULARS Amount (₹)
House 1 (Self Occupied) Nil
House 2 (DLO) 1,76,840
House 3 (Self Occupied) (1,75,000)
Therefore, Income from House Property 1,840

OPTION 3: House 2 and House 3 are self-occupied and House 1 is


Deemed to be Let Out:
PARTICULARS Amount (₹)
House 1 (DLO) 2,19,800
House 2 (Self Occupied) (30,000)
House 3 (Self Occupied) (1,75,000)
Therefore, Income from House Property 19,800

Since, Option 2 is most beneficial, Ganesh should opt to treat House 1


and House 3 as Self-Occupied and House 2 as Deemed to be Let Out.
His Income from House Property would be ₹1,840 accordingly for the
Assessment Year 2023-24.

PROVISIONS FOR ARREARS OR RENT AND UNREALISED RENT


RECEIVED SUBSEQUENTLY: (Section 25A)
- Unrealised Rent received subsequently or Arrears of rent shall
be taxable under the head Income from House Property in that
previous year in which such rent is actually received.
- It is taxable irrespective of the fact that the assessee is no longer
owner of the property in that previous year in which such
amount is actually received.
- A deduction of 30% is allowed from the amount received as
unrealised rent or arrears of rent by the assessee.

35
ILLUSTRATION 2:
Mr. Anand sold his residential house property in March, 2022.
In June 2022, he recovered rent of ₹10,000 from Mr. Gaurav, to whom
he had let out his house for 2 years from April, 2016 to March, 2018. He
could not realise two months rent of ₹20,000 from him and to that
extent his actual rent was reduced while computing income from house
property for A.Y. 2018-19.
Further, he had let out his property from April, 2018 to February, 2018
to Mr.Satish. In April, 2020, he had increased the rent from ₹12,000 to
₹15,000 per month and the same was subject matter of dispute. In
September, 2022, the matter was finally settled and Mr.Anand received
₹69,000 as arrears of rent for the period April, 2020 to February, 2022.
Would the recovery of unrealised rent and arrears of rent be taxable in
the hands of Mr. Anand, and if so in which year?
SOLUTION:
- Since, the unrealised rent and arrears of rent was
recovered/received in PY 2022-23 the same would be taxable in
AY 2023-24 under section 25A.
- It would be taxable irrespective of the fact that the assessee was
no longer owner of the property in that PY in which such
amount was received.
- A deduction of 30% would be allowed from these amounts
while computing income from house property.
Computation of Income from House Property of Mr. Anand for the AY
2023-24 (Relevant to PY 2022-23):
PARTICULARS Amount (₹)
a. Unrealised Rent recovered 10,000
b. Arrears of rent received 69,000
79,000
c. Less: Deduction @ 30% of above (23,700)
d. Therefore, Income from House Property 55,300

36
Treatment of Income from Co-owned Property:
- Self- Occupied Property:

Annual Value of Property of each co-owner will be NIL and each


co-owner shall be entitled for deduction of ₹30,000/₹2,00,000,
as the case may be, on account of interest on borrowed capital.

- Let out property:

The income from house property shall be computed as if the


property is owned by one owner and thereafter the income so
computed shall be apportioned amongst each co-owner as per
their specific share.

Deemed Ownership (Section 27)

The following persons, though not legal owners of a property are


deemed to be owners:
a. Transferor of the property, where the property is transferred to
the spouse or to a minor child except minor married daughter,
without adequate consideration.
b. Holder of impartible estate.
c. Member of a co-operative society.
d. Person in possession of a property.
e. Person having right in the property for a period not less than 12
years.

COMPOSITE RENT:
The owner of the property may sometimes receive rent in respect of
building as well as other assets like furniture/plant & machinery and
also for different services like lifts, security, power backup. The amount
so received is known as composite rent.

37
Tax Treatment of composite rent:
- Sum attributable to use of property – Taxable under the head
Income from House Property.
- Sum attributable to use of services – Taxable under the head
PGBP/ IOS.
Manner of splitting up:
- If let out building and other assets are inseparable:
Then taxable under the head PGBP/IOS
- If let out building and other assets are separable:
Then, income from letting out of building is taxable under the
head income from house property while income from letting
out of other assets is taxable under the head PGBP/IOS.

ILLUSTRATION 3:
Sanjay commenced construction of a residential house intended
exclusively for his residence on 1.12.2021. He raised a loan of
₹8,00,000 at 15% p.a. interest for the purpose of construction on
01.11.2021. Finding that there was an over run in the cost of
construction, he raised a further loan of ₹9,00,000 at 14% p.a. on
1.9.2022.
What is the interest allowable under section 24 in the Assessment
Year 2023-24, assuming that the construction was completed on
31.03.2023?
SOLUTION:
Computation of Interest on Borrowed Capital allowable under
section 24(b) of the Income Tax Act for the Assessment Year 2023-
24
(Relevant to Previous Year 2022-23):

38
PARTICULARS Amount (₹) Amount (₹)
a. Interest for the Current
Previous Year i.e. P.Y. 2022-23
₹8,00,000 * 15% * 12/12 1,20,000
₹9,00,000 * 14% * 7/12 73,500 1,93,500
b. Pre-Construction Period
Interest 10,000
th
1/5 of {₹8,00,000 * 15% *
5/12}
c. TOTAL INTEREST 2,03,500
d. Allowable Deduction {Actual or
₹2,00,000, whichever is lower
as the property is SELF 2,00,000
OCCUPIED}

Note:
- Construction is completed on: 31.03.2023
- Pre-Construction period till: 31.03.2022
- Pre- Construction Period 5 months
(01.11.2021 to 31.03.2022)

Income from House Property situated outside India:


- In case of R-OR: Income from House Property situated outside
India will always be taxable.
- In case of R-NOR/ NR: Income from House Property situated
outside India will be taxable only if it is received in India.

39
PROFITS AND GAINS FROM BUSINESS OR PROFESSION (PGBP)

Charging Section: Section 28 of Income Tax Act


Items of income chargeable to tax as income under the head PGBP are
as under:
a. Income from business or profession
b. Any compensation or other payment due to or received
c. Income from specific services performed for its members by a
trade, profession or business.
d. Incentives received or receivable by assessee carrying on export
business:
- Profit on sale of import entitlement.
- Cash assistance against exports under any scheme of
Government of India
- Customs duty or excise re-paid or repayable as drawback
- Profit on transfer of Duty Entitlement Pass Book Scheme or
Duty-Free Replenishment Certificate.
e. Value of any benefit or perquisite.
f. Sum due to, or received by, a partner of a firm.
g. Any sum received or receivable, in cash or kind, under an
agreement
h. Any sum received under a keyman insurance policy.
i. Fair Market Value of inventory on its conversion/ treatment as
capital asset.
j. Sum received on account of capital asset referred under section
35AD.

40
FORMAT FOR COMPUTATION OF PGBP:
PARTICULARS Amount Amount
(₹) (₹)
a. Net Profit as per Profit and Loss Account Xxxxx
b. Add: Expenses debited but not allowed as
deductions/ Specific disallowances /
Deemed Income i.e. income chargeable
under this head
- Depreciation debited (Allowed Xxxx
separately)
- Income Tax {Disallowed u/s 40(a)(ii)} Xxxx
- Amounts towards expenses payable to
residents on which tax was deductible
at source but not deducted during the
P.Y. or deducted but not deposited upto
due date of filing Income Tax Return u/s
139(1):
30% of the such expense is disallowed
(The same is allowed in the Previous xxxx
Year in which the tax is deducted and
remitted) {Section 40(a)(ia)}
- Any expenditure incurred, in respect of
which payment is made for goods,
services or facilities to a related person,
to the extent same is excessive or
unreasonable {Section 40A(2)} xxxx
- Any expenditure incurred in respect of
which payment or aggregate of
payments to a person exceeding
₹10,000 in a single day is made
otherwise than by an Account Payee
Cheque/ Bank Draft/ECS xxxx
{Section 40(A)(3)}
- Certain sums payable by the assessee
which have not been paid on or before
due date of filing of ITR {Section 43B} xxxx
- Personal Expenses

41
{Not allowed as per Section 37} xxxx
- Capital expenditure
{Not allowed as per Section 37} xxxx
- Repairs of capital nature
{Not allowable as per section 30 & 31} xxxx
- Amortization of Preliminary
expenditure u/s 35D/ Expenditure
incurred under Voluntary Retirement
scheme u/s 35DDA {4/5th of such
expenditure to be added back} xxxx
- Fine or Penalty for infringement or
breach of law {However, penalty in
nature of damages for delay in
completion of a contract, being
compensatory in nature, is allowable.} xxxx
- All expenses related to income which is
not taxable under this head
{e.g. Municipal taxes in respect of
residential house property} xxxx
- Any sum paid by the assessee as an
employer by way of contribution to
pension scheme u/s 80CCD exceeding
10% of salary xxxx
- Salary, remuneration, interest
received by a partner from the firm to
the extent the same is deductible in the
hands of firm as per section 40(b) xxxx
- Bad Debts allowed as deduction u/s
36(1) in an earlier P.Y., now recovered.
{Deemed Income as per section 41(4)} xxxx
- Remission or Cessation of a trading
liability {Deemed as income u/s 41(1)} xxxx Xxxxx
c. Sub Total {a + b} Xxxxx
d. Less: Expenditure allowable as deduction
but not debited to statement of Profit and
Loss i.e. Specific deductions/ Items
credited but not taxable under this head:

42
- Depreciation as per Income Tax Rules
including Additional depreciation xxxx
- Balance additional depreciation @ 10%
of actual cost of Plant and Machinery
acquired and installed during preceding
P.Y i.e. P.Y 2021-22 in this case and put
to use for less than 180 days in that year xxxx
- Dividend Income {Taxable u/h IFOS} xxxx
- Agricultural Income xxxx
- Interest on Securities/ Savings Bank
Account/ FD {Taxable under the head xxxx
IFOS}
- Profits on sale of Capital assets taxable
u/h Capital Gains xxxx
- Rental Income from House Property
{Taxable u/s Income from HP} xxxx
- Winnings from Lotteries, Horse Races,
games, etc {Taxable u/s IFOS} xxxx
- Gifts {As they are either not taxable or;
Taxable u/s IFOS} xxxx
- Income Tax Refund {Not Taxable} xxx
- Interest on Income Tax Refund
{Taxable u/h IFOS} xxxx (xxxx)
e. Therefore, PGBP. Xxxx

Illustration 1:
Compute Profits and Gains from Business or Profession of Mr.A for the
Assessment Year 2023-24 from the Profit and Loss Account and
additional information given for the Financial Year 2022-23:
Dr. Cr.

PARTICULARS Amount PARTICULARS Amount


(₹) (₹)
To Salaries 18,00,000 By Gross Profit 80,00,000
To Electricity 1,20,000 By Rent received for
Expenses Residential House 4,20,000
43
To Municipal Taxes for By Interest on Fixed
residential House 20,000 Deposit 12,000
To Travelling Expenses 4,00,000 By Gift received
from Father-in-Law 68,000
To Income Tax 2,25,000 By Dividend from
Indian Companies 15,000
To Bad Debts 75,000 By Gains on sale of
Shares 1,25,000
To Provision for Bad By Income Tax
Debts 65,000 Refund (includes
interest of ₹2,000) 12,000
To Penalty – GST 88,000 By Winnings from
Lotteries 2,50,000
To Bonus to staff 1,50,000
To Printing and
Stationary 1,22,000
To Donation to
National Defence 60,000
Fund
To Office Expenses 42,000
To Depreciation 1,62,000
To Professional Fees 6,00,000
To Purchase of Land 3,50,000
To Net Profit 46,23,000
TOTAL 89,02,000 TOTAL 89,02,000

Additional Information:
1. Depreciation as per provisions of Income Tax Rules is ₹2,00,000.
2. Out of Salaries of ₹18,00,000, ₹6,00,000 are paid to Mr.A.
3. Out of total travelling expenses, ₹1,00,000 relates to vacation
taken by Mr. A along with his family at Singapore.
4. Out of Bonus of ₹1,50,000 to staff, ₹20,000 has been paid on 1st
November, 2023.
5. Out of the Printing and Stationary Expenses, ₹40,000 are paid in
cash to Millennium Stationers on 14th January, 2023.

44
6. Out of Professional Fees of ₹6,00,000, ₹2,00,000 has been paid to
Mrs. A. The Fair Value of Services provided by her is ₹1,20,000.
Further, on other fees of ₹1,00,000 paid to Mr.X, tax has not been
deducted at source.
SOLUTION:
Computation of Profits and Gains from Business or Profession of
Mr. A for the Assessment Year 2023-24
(Relevant to Previous Year 2022-23):
PARTICULARS Amount Amount (₹)
(₹)
a. Net Profit as per Profit and
Loss Account 46,23,000
b. Add: Expenses debited but
not allowed as deductions/
Specific disallowances
- Salaries debited for Sole
Proprietor 6,00,000
- Municipal Taxes for
residential House 20,000
- Traveling Expenses
(Personal Element) 1,00,000
- Income Tax 2,25,000
- Provision for Bad Debts 65,000
- GST Penalty 88,000
- Bonus to Staff {Section 20,000
43B}
- Printing and Stationery
paid in cash {Section 40,000
40(A)(3)}
- Donation {Not related to 60,000
business}
- Depreciation debited
{Allowed separately} 1,62,000

45
- Excessive amount of
Professional fees paid to
relative {Section 40(A)(2)}
₹2,00,000 (-) ₹1,80,000 20,000
- Amount paid towards 30,000
expenses to a resident
without TDS {Section
40(a)(ia)} 30% of ₹1,00,000
- Purchase of Land {Capital
Expenses} 3,50,000 17,80,000
c. Sub Total 64,03,000
d. Less: Items Credited but not
taxable under this head/
specific allowances
- By Rent received from
House Property
{Considered u/h Income 4,20,000
from House Property}
- Interest on Fixed deposit
{Taxable u/h IFOS} 12,000
- Gift received from Father-
in-law {Not Taxable, being
from relative} 68,000
- Dividend from Indian
Companies {Taxable u/h 15,000
IFOS}
- Gains on Sale of Shares
{Taxable u/h Capital Gains} 1,25,000
- Income Tax Refund
(Not Taxable) 10,000
- Interest on Income Tax
Refund (Taxable u/h IFOS) 2,000
- Winnings from lotteries
(Taxable u/h IFOS) 2,50,000
- Depreciation as per Income
Tax Rules 2,00,000 (11,02,000)
e. Therefore, Profits and Gains
from Business or Profession 53,01,000
46
IMPORTANT DEDUCTIONS/ ALLOWANCES:
1. Section 32: Depreciation
- Conditions to be satisfied:
a. The assessee must own the assets, wholly or partly.
b. The assets should be actually used by the assessee for the purpose
of his business during the Previous Year.
Note:
- If assets are not used exclusively for the business or
profession of the assessee but for other purposes as well, the
depreciation allowable would be a proportionate part of
depreciation allowance to which the assessee would be
otherwise entitled.
- Use includes passive use in certain circumstances. Courts have
held that in certain circumstances, an asset can said to be in
use even when it is “kept ready for use”. E.g.: Depreciation can
also be claimed on stand by equipment and fire extinguishers
if they are ready for use. Therefore, use includes passive use
as well.

- Computation of depreciation:
a. Depreciation is claimed on the BLOCK OF ASSETS.
b. It is claimed on Written Down Value (WDV) basis.
Written Down Value of block of asset is computed as follows:
PARTICULARS Amount (₹)
a. Opening Written Down Value of the Block
of Asset as on 01.04 of the P.Y. XXXXX
(01.04.2022)
b. Add: Additions at Actual Cost during the XXXXX
P.Y.
c. Sub Total XXXXX
d. Less: Sale Proceeds of Assets from the
block sold during the P.Y. (XXXX)
e. Therefore, WDV for claiming Depreciation XXXX
f. Depreciation for the P.Y. at the rate
prescribed in Rule 5 (XXXX)

47
g. Therefore, WDV at the end of the P.Y. i.e.
opening WDV for next P.Y.
{as on 01.04.2023} XXXXX
c. In case of purchase of assets during the Previous Year, if it is put to
use for less than 180 days, then depreciation is allowed at half of
normal.
d. Meaning of ACTUAL COST of asset:
- It means actual cost of the asset to the assessee as reduced by
that portion of cost thereof, if any, as has been met directly or
indirectly by any other person or authority.
- However, if any payment or aggregate of payment is made to
a supplier of a asset in a day amount exceeding ₹10,000 in
mode other than Account Payee Cheque, Bank Draft or ECS
(Now it also includes RTGS, IMP, Net Banking, Debit Card,
credit Card, UPI, NEFT, BHIM, Adhaar Pay) then such amount
shall not form part of Actual Cost.
- Also refer Explanation to section 43(1) of Income Tax Act.
e. Rates of Depreciation for important block of assets:
a. Buildings – mainly used for residential purposes except hotels:
5%
b. Building not used for residential purposes: 10%
c. Purely Temporary erections such as wooden structures: 40%
d. Furniture and fittings including electrical fittings: 10%
e. Motor Car (other than those used in business of running them
on hire): 15%
f. Motor Buses, Motor Lorries, Motor Taxis used in business of
running them on hire: 30%
g. Aeroplanes, Aeroengines: 40%
h. Pollution Control Equipments: 40%
i. Live Saving Medical Equipment: 40%
j. Computers: 40%
k. Books (Annual publication or other than annual publications)
owned by assessee carrying on profession: 40%
l. Books owned by assessee carrying on business in running
lending libraries: 40%
m. Plant and Machine (General Rate): 15%
48
n. Know How, Patents, Copy rights, trademarks, licenses,
franchises or any other business or commercial rights of
similar nature, NOT BEING GOODWILL: 25%
- Additional Depreciation:
a. It is allowed on NEW PLANT AND MACHINERY @ 20%
b. of ACTUAL COST
c. to any assessee who is engaged in business of manufacture
or production of any article or thing.
d. In case, the plant and machinery is put to use for less than
180 days in the P.Y. in which it is purchased, then it is
allowed @ 10% in that P.Y. and balance 10% in subsequent
P.Y.
e. Such additional depreciation is NOT AVAILABLE in respect
of:
1. Second Hand Plant and Machinery
2. Any machine or plant installed in office premises,
residential accommodation or in any guest house.
3. Office appliances or road transport vehicles.
4. Any machine or plant, the whole or part of the actual
cost of which is allowed as deduction (whether by way
of depreciation or otherwise) in computing PGBP of any
one P.Y.
Illustration 2:
Mr. X, a Proprietor engaged in manufacture business furnishes the
following particulars:
PARTICULARS Amount (₹)
a. Opening balance of Plant and Machinery as on
01.04.2022 i.e. WDV as on 31.03.2022 after
reducing depreciation for P.Y. 2021-22. 30,00,000
b. New Plant and Machine purchased and put to
use on 08.06.2022 20,00,000
c. New Plant and Machine acquired and put to use
on 15.12.2022 8,00,000
d. Computer acquired and installed in office
premises on 02.01.2023 3,00,000
49
Compute the amount of depreciation and additional depreciation as
per the Income Tax Act, 1961 for the Assessment Year 2023-24. Assume
that all the assets were purchased by way of Account Payee Cheque.
Solution:
Computation of Depreciation and Additional Depreciation for the
Assessment Year 2023-24 (Relevant to Previous Year 2022-23):
Figures in ₹
PARTICULARS Plant and Computer
Machinery (40%)
(15%)
a. Opening WDV as on 01.04.2022 30,00,000 -
b. Add: Additions at Actual Cost
during the Previous Year 28,00,000 3,00,000
c. Therefore, Sub Total 58,00,000 3,00,000
d. Less: Depreciation for the (12,90,000) (60,000)
Previous Year (₹3,00,000 *
(NOTE) 40% * ½)
e. Therefore, WDV as on 45,10,000 2,40,000
01.04.2023

NOTE:
Computation of Depreciation on Plant and Machine:
PARTICULARS Amount (₹) Amount (₹)
Normal Depreciation:
On Plant and Machine purchased
during the P.Y. and put to use for
less than 180 days
(₹8,00,000 * 15% * 1/2) 60,000
On Balance WDV (₹50,00,000 * 7,50,000 8,10,000
15%)
Additional Depreciation:
On New Plant and Machine put to
use for less than 180 days
50
(₹8,00,000 * 10%) 80,000
On New Plant and Machine put to
use for 180 days and more
(₹20,00,000 * 20%) 4,00,000 4,80,000
Total Depreciation 12,90,000

DEPRECIATION (Section 32) v/s SHORT TERM CAPITAL GAINS on SALE


of DEPRECIABLE ASSET (Section 50):

- Where Sale Proceeds exceeds the “Opening WDV + Additions


“of the block, it gives rise to Short Term Capital Gains. {There
cannot be negative WDV}. (Demonstrated in case 2 in
illustration 3)
- Further, where, the block of asset ceases to exist as all the
assets in the block are sold, there shall either be STCG or STCL
depending on the Sale Proceeds. If Sale Proceeds exceeds
“Opening WDV + Additions” of the block, it gives rise to STCG
(Demonstrated in case 3 in illustration 3). However, if it is
lower, it gives rise to STCL. (Demonstrated in case 4 in
illustration 3).
Illustration 3:
Following is the BLOCK of Asset of Computers (40%) in 4 cases:
FIGURES IN ₹
PARTICULARS Nos. Case 1 Nos. Case 2 Nos. Case 3 Nos. Case 4
Opening WDV 20 10,00,000 20 10,00,000 20 10,00,000 20 10,00,000
(+) Purchases 10 10,00,000 10 10,00,000 10 10,00,000 10 10,00,000
Sub Total 30 20,00,000 30 20,00,000 30 20,00,000 30 20,00,000
Less: Sales 25 17,00,000 26 21,00,000 30 22,00,000 30 17,50,000
WDV 5 3,00,000 (1,00,000) NIL (2,00,000) NIL 2,50,000
i.e.N.A. BUT N.A.
Depreciation 1,20,000 N.A. N.A. N.A.
Sec 32/ Sec 50 32 50 50 50
STCG/(STCL) N.A. 1,00,000 2,00,000 (2,50,000)

51
Notes:
- Nos. indicates number of assets.
- All new purchases of computer were put to use for 180 days or
more during the P.Y.

Computation of Short-Term Capital Gains/ (Short Term Capital Loss)


in case 2, 3 & 4: Figures in ₹
PARTICULARS Case 2 Case 3 Case 4
a. Full Value of
Consideration/ Net
Sale Consideration 21,00,000 22,00,000 17,50,000
b. Less: Cost of
Acquisition/ Cost of
Improvement
(Opening WDV +
Additions) (20,00,000) (20,00,000) (20,00,000)
c. Therefore,
STCG/(STCL) 1,00,000 2,00,000 (2,50,000)

EXPENDITURE ON SCIENTIFIC RESEARCH: {Section 35}


- It means activities for extension of knowledge in fields of natural
or applied science including agriculture, animal husbandry or
fisheries.
- Broadly, Deduction is allowed to the extent of 100% in following
cases:
A. Expenditure made by Company engaged in business of bio-
technology or any business of manufacture or production of
any article.
B. Expenditure incurred by any assessee on scientific research
related to business {Revenue Expenditure or Capital
expenditure – Other than on land}.
C. Amount paid to Notified approved
university/College/Research Association/ Other institution for

52
Social Sciences or Statistical research, Approved National
Laboratory/ University/ IIT.

INVESTMENT LINKED TAX INCENTIVES FOR SPECIFIED BUSINESS


(Section 35AD):
- Deduction is allowed to the extent of 100% for Capital
Expenditure in case of specified businesses.
- Following are SPECIFIED BUSINESSES: -
1. Setting up and Operating Cold Chain facilities for specified
products.
2. Setting up and Operating warehousing facilities for storing
agricultural produce.
3. Laying and operating a cross country natural gas or crude or
petroleum oil pipeline network for distribution including
storage facilities being an integral part of such network.
4. Building and operating a hotel of two star or above category,
anywhere in India.
5. Building and operating a hospital, anywhere in India, with at
least 100 beds for patients.
6. Developing and building a housing project under a notified
scheme for slum redevelopment or rehabilitation framed by
Central Government or a State Government.
7. Developing and building a housing project under a notified
scheme for affordable housing framed by Central Government
or State Government.
8. Production of fertilizer in India.
9. Setting up and operating an inland container depot or a
container freight station notified or approved under the
Customs Act.
10. Bee-keeping and production of honey and beeswax.
11. Setting up and operating a warehousing facility for storage of
sugar.
12. Laying and operating a slurry pipeline for transportation of
iron ore.
53
13. Setting up and operating a semiconductor wafer fabrication
manufacturing unit.
14. Developing or maintaining and operating or developing,
maintaining and operating a new infrastructure facility.
- The above businesses should not be set up by splitting up or
reconstruction of business already in existence. Also, it should not
be set up by transfer to a specified business of machinery or plant
previously used for any purpose.
- It should be noted that deduction is NOT ALLOWED for Capital
Expenditure incurred on acquisition of GOODWILL, LAND or
FINANCIAL INSTRUMENT.
- Also, any expenditure incurred in respect of which payment or
aggregate of payment made to a person of amount exceeding
₹10,000 in a day otherwise than by an Account Payee Cheque,
Bank Draft, ECS would not be eligible for deduction.
- Expenditure incurred prior to commencement of business would
be allowed as deduction in that Previous Year in which the
assessee commences the business. Only condition to be satisfied
is that amount incurred prior to commencement of business
should be capitalized in the books of account on date of
commencement of operations.
- No deduction u/s 10AA or Chapter VI-A under the heading C-
deductions in respect of certain incomes should have been
claimed. (Once the assessee has claimed benefit of deduction u/s
35AD for a particular year in respect of specified business, he
cannot claim the benefit u/ch VI-A or section 10AA for the same
or any other year and vice-versa)
- No deduction under any other section for expenditure for which
deduction is allowed under this section.
- Any loss from specified business can be set off only against the
profit of another specified business.

54
ILLUSTRATION 4:
MNP Ltd. commenced operations of the business of a new four-star
hotel in Chennai on 01.04.2022. The company incurred capital
expenditure of ₹40 lakhs during the period January 2022 to March 2022
exclusively for the above business and capitalized the same in its books
of account as on 1st April, 2022.
Further, during the Previous Year 2022-23, it incurred capital
expenditure of ₹2.5 crore (out of which ₹1 crore was for acquisition of
land) exclusively for above business.
Compute the income under the head PGBP for the Assessment Year
2023-24 assuming that MNP Ltd. has fulfilled all the conditions specified
for claim of deduction under section 35AD and has not claimed any
deduction under chapter VI-A under the heading C- Deductions in
respect of certain incomes.”
The profits from the business of running this hotel (before claiming
deduction under section 35AD) for the Assessment Year 2023-24 is ₹80
lakhs.
Assume that the company also has another existing business of running
a four-star hotel in Kanpur which commenced operations 6 years back,
the profits from which was ₹130 lakhs for the Assessment Year 2023-
24.

SOLUTION:
Computation of Income from Business or Profession for the
Assessment Year 2023-24 (Relevant to Previous Year 2022-23):
PARTICULARS Amount Amount
(₹ in lakhs) (₹ in lakhs)
Income from Four Star Hotel in Chennai 80
Less: Eligible deduction under section
35AD
(a) Expenditure prior to
commencement 40
55
(b) Capital expenditure incurred in
P.Y. 2022-23 (250 lakh – 100 lakh) 150 (190)
Total Loss from Four-star hotel in
Chennai (110)
Income from Four-star hotel in 130
Kanpur
Therefore, Income from business
or profession 20

Notes:
1. As per section 73A of Income Tax Act, loss from specified business
can be set off only against income from specified business.
2. Expenditure relating to acquisition of land is not allowed as
deduction under section 35AD.

AMORTISATION OF PRELIMINARY EXPENDITURE {SECTION 35D}


- It is allowed for Indian Companies and other resident non-
corporate taxpayers for the establishment of business concerns
of the expansion of business of existing concerns.
- Preliminary expenditure incurred shall be amortized over a period
of 5 years. In short, 1/5th of such expenditure shall be allowed
for each of the 5 successive previous years beginning with the
previous year in which the business commences or the previous
year in which the extension of undertaking is completed.
- Eligible expenses – Expenditure in connection with –
a. The preparation of feasibility report.
b. The preparation of project report
c. Conducting market survey or any other survey necessary for
the business of the assessee.
d. Engineering services relating to the assessee’s business.
e. Legal charges for drafting any agreement between the
assessee and any other person for any purpose relating to the
setting up to conduct business of assessee.

56
f. Further, if the assessee is company – then also – expenditure
incurred by way of legal charges for drafting the Memorandum
and Articles of Association of Company; on printing of MOA,
AOA, fees for registering the company under the Companies
Act and in connection with issue for public subscription of
shares or debentures of the company like underwriting
commission, brokerage.
- Overall limits:
In case of Indian companies:
5% of cost of project or; 5% of Capital employed (whichever is
higher)
In case of resident non-corporate assessee:
5% of the cost of project.
- No other deduction under any other provision of the Act

AMORTISATION OF EXPENDITURE INCURRED under VOLUNTARY


RETIREMENT SCHEME {Section 35DDA}:
- It is allowed to any assessee to the extent of 1/5th of the amount
paid for that P.Y. and the balance in 4 equal installments in the
four immediately succeeding previous years.
- No deduction under any provisions of this Act.

Other Deductions - Section 36


GENERAL DEDUCTIONS/ RESIDUARY EXPENSES – Section 37
- Revenue expenditure incurred for purpose of carrying in
business, profession but not allowable specifically in any of the
above sections.
- It should be incurred by the assessee during the previous year.
- It must have been incurred after the business was set up.
- It should not be in the nature of any personal expenses of the
assessee.
- It should not be in nature of capital expenditure.

57
- It should not have been incurred by the assessee for any purpose
which is an offence or is prohibited by law.
- It should be noted that CSR Expenditure is disallowed under this
section. {As it is an application of income. But, if CSR expenditure
satisfied the conditions of deductions in section 30-36, it will be
allowed in that respective section}.
- Advertisement in souvenir of political parties: Not allowed as
deduction while computing PGBP. {But, it will be allowed u/s
80GGB & 80GGC, as the case may be from Gross Total Income
while computing Total Income.}

INADMISSIBLE EXPENSES:

- 40(a)(i): Non compliance of TDS provisions w.r.t. payment to


Non-Residents

- 40(a)(ia): Non compliance of TDS provisions w.r.t. payment to


Residents : 30% of the such expenses are disallowed.

- 40(A)(2): Excessive payments to relatives

- 40(A)(3): Payment of expenses in excess of ₹10,000/₹35,000 in


mode other than Account Payee Cheque, Bank Draft, ECS, UPI,
BHIM, Adhaar Pay, NEFT, RTGS.

- 43B: Certain deductions to be made only on actual payment


basis
{In short, if the amounts as mentioned in section 43B remains
unpaid upto due date of filing of ITR u/s 139(1), then such amount
shall be disallowed while computing PGBP.}

58
SECTION 40(b)

Allowability of Partner’s Remuneration and Interest on


Partner’s Capital to a Partnership Firm:

PARTNER’S REMUNERATION:

- It is allowed only if it is as per the partnership deed.


- It is allowed to the extent of lower of 2:

a. Remuneration as debited towards working partners


Or;
b. Ceiling Limit as follows:

BOOK PROFITS Amount (₹)


a. NEGATIVE i.e. Loss 3,00,000
b. POSITIVE
90% of Book Profits or;
Upto ₹3,00,000 ₹1,50,000, whichever is
higher
Above ₹3,00,00
On first ₹3,00,000 90% of Books Profits
On Book Profits
exceeding ₹3,00,000 60% of Book Profits.

Here, Book Profits are computed as follows:

a. Net Profit as per P&L A/C xxxxx


b. Add: Expenses debited but not allowed as deduction xxxxx
c. Less: Items credited but not taxable under this head xxxxx
d. Add: Partner’s Remuneration debited xxxxx
e. Therefore, Book Profits xxxxx

59
Interest on Partner’s Capital:

- It is allowed only if it is as per the deed.


- It is allowed to the extent of 12% of partner’s capital.

PGBP of the Firm:


Book Profits of the firm (-) Allowable Partner’s Remuneration
Notes:
1. It should be noted that whatever amount of remuneration and
interest on partner’s capital is allowed as deduction to the firm is
taxable in hands of partners under the head PGBP.
2. Share of profit of firm given to partners is exempt in the hands of
partner u/s 10(2A) of Income Tax Act.

Illustration 5:
Rao and Jain, a partnership firm consisting of two partners reports a net
profit of ₹7,00,000 before deduction of following items:
1. Salary of ₹20,000 each per month payable to two working
partners of the firm (as authorized by the deed of partnership
firm).
2. Depreciation on Plant and Machine under section 32 (computed):
₹1,50,000.
3. Interest on Capital at 15% per annum (as per the deed of
partnership), the amount of capital eligible for interest:
₹5,00,000.
Compute:
1. Book Profit of the firm under section 40(b) of Income Tax Act.
2. Allowable working partner salary for Assessment Year 2023-24 as
per section 40(b) of Income Tax Act.
3. Profits and Gains from Business or profession of the firm.

60
SOLUTION:
Computation of Book Profits, allowable partner’s remuneration and
PGBP of the firm for the Assessment Year 2023-24
(Relevant to Previous Year 2022-23):

PARTICULARS Amount (₹) Amount (₹)


a. Net Profit of the firm before
deduction of depreciation and
interest on partner's capital 7,00,000
b. Less: Depreciation as per Income
Tax Act (1,50,000)
c. Less: Interest on Partner’s Capital as
allowed {₹5,00,000 * 12%} (60,000)
d. BOOK PROFITS 4,90,000
e. Less: Remuneration
(Lower of following 2)
As per deed {₹20,000 * 2 * 12} 4,80,000
Or;
Ceiling limit u/s 40(b)
On first ₹3,00,000 @ 90% =
2,70,000
On balance in excess of
₹3,00,000 @ 60% =
₹1,14,000 3,84,000
(₹1,90,000 * 60%)
Therefore, allowable remuneration (3,84,000)
f. Therefore, PGBP 1,06,000

COMPULSARY MAINTENANCE OF ACCOUNTS: {Section 44AA}

61
TAX AUDIT: {Section 44AB}

APPLICABILITY:

- In case of person carrying on business:


a. If Turnover/Sales exceed ₹1 crore in relevant PY.
b. In case of Individual/HUF/Partnership Firm carrying on
business whose sales/turnover during PY exceed ₹2 crore.
c. For person whose aggregate cash receipts is not exceeding 5%
of total receipts and; aggregate cash payments are not
exceeding 5% of total payments then the threshold turnover
limit for applicability of tax audit shall be ₹10 crore.
- In case of person carrying on profession:
a. If the Gross receipts of assessee exceeds ₹50 lakh.
b. In case, the Gross Receipts is not exceeding ₹50 lakh and
assessee declares income less than presumed income of 50%
and his income exceeds basic exemption limit in that P.Y.
- In case of assessee covered u/s 44AE i.e. assessee engaged in
transport business:
Applicable if assessee declares income less than presumed
income.
- Submission of Audit Report:
Due date of filing of Income Tax Return in case of assessee who
are required to get their accounts audited is 31st October of the
relevant assessment year.
The specified date for Tax Audit Report is one month prior to the
due date of furnishing of ITR i.e. 30th September of the relevant
A.Y.

Illustration 6:

Comment on applicability of Tax Audit under section 44AB of the


Income Tax Act in the following cases for PY 2023-24:

1. Company assessee having turnover in business ₹120 lakhs,


PGBP declared: ₹20 lakhs.
62
Solution:
Here, as Turnover of Company assessee engaged in business
exceeds ₹1 crore, Tax Audit is applicable irrespective of PGBP
declared.
It should be noted that provisions of presumptive taxation of
section 44AD are NOT APPLICABLE in this case as assessee is
not individual, HUF or Firm. Hence, Section 44AD is N.A. even
if T/O is not exceeding ₹2 crore.

2. What would be your answer in the above case, if the assessee


is not Company but an individual assessee?

Solution:
Here, provisions of section 44AD are applicable assessee being
INDIVIDUAL engaged in business, turnover NOT exceeding ₹2
crore. PGBP declared by the assessee is equal to or more than
8% of ₹120 lakhs i.e. ₹9.6 lakhs. Hence, no need to maintain
books of accounts and no need of audit u/s 44AB.

3. A Partnership Firm (Not being LLP) has turnover in business


of ₹3 crores, PGBP declared is ₹30 lakhs.

Solution:
Here, provisions of section 44AB w.r.t. Tax Audit shall apply as
Turnover in business exceeds ₹1 crore.
Note that provisions of section 44AD shall not apply in this case
as Turnover of assessee in business exceeds ₹2 crore.

4. Mr.A, a Chartered Accountant, engaged in Practice of


Accountancy has Gross Receipts of ₹60,00,000. He declares
PGBP of ₹32,00,000.
Solution:

63
Here Tax Audit u/s 44AB is mandatory as Gross Receipts in case
of assessee engaged in Profession exceeds ₹50,00,000 during
the P.Y.

5. What would be your answer in the above case, if the Gross


Receipts were ₹48,00,000 and PGBP declared is ₹20,00,000.

Solution:
Here section 44ADA is applicable as Gross receipts for assessee
engaged in profession is not exceeding ₹50,00,000.
PGBP declared by such assessee from profession is not at least
50% of the Gross Receipts & therefore, it is mandatory to
maintain books of Accounts as per section 44AA and get them
audited u/s 44AB.

6. What would be your answer in Question 5, if PGBP declared


is ₹25,00,000 instead of ₹20,00,000?

Solution:
Here, section 44ADA provisions are applicable as Gross
Receipts in case of assessee engaged in profession is not
exceeding ₹50,00,000. Further, he has declared PGBP of 50%
or more of the Gross Receipts & therefore, there is no need to
maintain books of accounts and also no need for audit u/s
44AB.

7. What would be your answer in Q.1, if the amount of ₹118


lakhs was received via banking mode out of turnover of ₹120
lakhs? Given also that out of expenses incurred, about 98%
were incurred via banking mode.

Solution:
Here, the limit applicable for tax audit shall be ₹10 crore as
95% or more of the turnover is received by way other than

64
cash and also 95% or more of expenses are by way of other
than cash.
Hence, provisions of section 44AB w.r.t. Tax Audit shall not
apply.

8. An individual engaged in business is having turnover of ₹12


crores, declared PGBP of ₹2 crores.

Solution:
Here, TAX AUDIT is applicable as Turnover in business exceeds
₹1 crore. It should be noted that how much of turnover is
received in Bank / how much of expenses is done via bank does
not matter as Turnover exceeds ₹10 crore.
Note: Provisions of section 44AD do not apply as Turnover in
business exceeds ₹2 crores.

PRESUMPTIVE TAXATION:

SECTION 44AD:

- Applicable for Resident individual, HUF or Partnership Firm (not


being LLP) engaged in business.
- Turnover in this business should be not exceeding ₹200 lakhs
during the P.Y.
- Presumptive Income:
Minimum income to be declared:
8% of Gross Receipts
6% of Gross Receipts in respect of Turnover which is received by
way of Account Payee cheque, Bank Draft, ECS, Credit Card, Debit
Card, NEFT, RTGS, UPI, Aadhaar Pay during the P.Y. or before due
date of filing of ITR.
- No need to maintain books of Accounts and get them audited if
income is declared as per section 44AD. However, if after
declaring profits on presumptive basis u/s 44AD, say, for A.Y.
2023-24, non-declaration of profits on presumptive basis for any
of 5 successive A.Y.s thereafter (i.e. from A.Y. 2024-25 to A.Y.
65
2028-29), say, for A.Y. 2025-26, would disentitle the assessee
from claiming profits on presumptive basis for 5 successive A.Y.
relevant to P.Y. of such non-declaration (i.e. from A.Y. 2026-27 –
2030-21). In such a case, assessee would have to maintain books
of accounts and get them audited u/s 44AB if the total income
exceeds basic exemption limit in those years.

SECTION 44ADA:
- Applicable to Resident individual or partnership firm (but not LLP)
engaged in any profession.
- Gross receipts should not be exceeding ₹50 lakhs during a P.Y.
- Presumptive Income:
Minimum Income to be declared in 50% of Gross Receipts.
- No need to maintain books of accounts if profits declared is as per
44ADA. However, if profits declared is lower than 44ADA & total
income exceeds basic exemption limit, then maintain books of
Accounts and get them audited as per section 44AB.
Common notes for section 44AD & 44ADA:
- Deductions u/s 30 – 38 shall be deemed to have been given full
effect to and no further deduction shall be allowed. {Even Interest
on Partner’s Capital and Partner’s Remuneration of section 40(b)
will not be allowed}.
- Eligible assessee opting for provisions of these presumptive
taxation shall pay advance tax only in one instalment i.e. 100% of
the advance tax is payable by 15th March of the Previous Year.

SECTION 44AE:
- An assessee engaged in business of leasing, hiring, plying of goods
carriages and owning NOT MORE THAN 10 goods carriages at any
point of time during the Previous Year is eligible for section 44AE.
- Presumed Income to be declared:
For other than heavy goods vehicle: ₹7,500 per month or part
thereof during which such vehicle is owned.
66
For heavy goods vehicle: {Such vehicle of which gross weight
exceeds 12,000 kgs}: ₹1,000 per ton per month or part thereof.
- Deductions u/s 30 – 38 shall be deemed to have been given full
effect to and no further deduction shall be allowed. However,
Interest on Partner’s Capital and Partner’s Remuneration of
section 40(b) will be allowed.
- No need to maintain books of accounts if profits declared is as per
44AE. However, if profits declared is lower than 44AE & total
income exceeds basic exemption limit, then maintain books of
Accounts and get them audited as per section 44AB.
- Eligible assessee has to pay advance tax in 4 installments.

Illustration 7:
A Partnership Firm not being LLP engaged in business having Turnover
of ₹150 lakhs has received amount of ₹110 lakhs by way of Account
payee cheque upto 31.03.2023. Further ₹20 lakhs is received by way of
Account Payee cheque upto 31.07.2022. What is the deemed income
to be declared as per section 44AD?
Solution:
Deemed Income u/s 44AD is at 8% of turnover and in case of amount
received by way of Account Payee cheque/ Bank Draft/ ECS upto due
date of filing of ITR shall be at 6% of the amount so received from the
turnover.
Computation of deemed income under section 44AD for A.Y. 2023-24
(Relevant to P.Y. 2022-23):
PARTICULARS Amount
(₹ in
Lakhs)
a. Turnover amount received by way of account
payee cheque upto due date of filing of ITR
(110 lakh + 20 lakh) = 130 lakh
b. Deemed Income/ Presumed Income/ PGBP as per
section 44AD @ 6% on above 7.8
67
c. Balance Turnover {150 lakhs (-) 130 lakhs} = 20
lakhs
d. Deemed Income @ 8% of turnover 1.6
e. Therefore, Total Deemed Income/ Presumed
Income 9.4

Note:
If such amount or more than this is declared as income then no need to
maintain books of accounts and even no need for tax audit u/s 44AB.

ILLUSTRATION 8:
Mr. Prakash is in the business of operating goods vehicle. As on
01.04.2022, he had the following vehicles:
Vehicle Weight Date of Put to use
purchase during FY 2021-
22
A 8,500 02.04.2021 Yes
B 13,000 15.05.2021 Yes
C 12,000 04.08.2021 No. Under
Repair

During Previous Year 2022-23 he purchased the following vehicles:


Vehicle Weight Date of Put to use
purchase during FY 2022-
23
D 11,000 30.04.2022 10.05.2022
E 15,000 15.05.2022 18.05.2022
Compute his income under section 44AE of Income Tax Act for
Assessment Year 2023-24.
SOLUTION:
Computation of deemed/presumed income of Mr. Prakash for the
Assessment Year 2023-24 (Relevant to Previous Year 2022-23):

68
Vehicle Period Computation Amount (₹)
(Months)
A 12 ₹7,500 * 12 90,000
B 12 ₹1,000 * 13 * 12 1,56,000
C 12 ₹7,500 * 12 90,000
D 12 ₹7,500 * 12 90,000
E 11 ₹1,000 * 15 * 11 1,65,000
Total Income u/s 44AE 5,91,000

INCOME OF BUSINESS INCOME IN CASES WHERE INCOME IS PARTLY


AGRICULTURAL AND PARTLY BUSINESS IN NATURE {It will be covered
as part of Agricultural income topic}

69
CAPITAL GAINS
Charging section:
Section 45:

CAPITAL
TRANSFER of ASSET

What is “Capital Asset”? {Section 2(14)}


- Property of any kind held by an assessee whether or not
connected with his business or profession
- Any securities held by FIIs which has invested in such securities in
accordance with SEBI regulations.
- It does not include: {Following are not Capital Assets}
a. Stock-in-trade
b. Personal Effects {Movable Property held for personal use}
Exclusions from Personal Effects (Therefore Capital Assets):
Jewellery
Archaeological collections
Drawings
Paintings
Sculptures
Work of Art.
c. Rural Agricultural land in India.
Agricultural Land described below is Urban Agricultural Land
and therefore would fall within definition of Capital Asset:
1. Agricultural land situated in any area within the jurisdiction
of municipality or cantonment board having population of
not less than 10,000, or;
2. Agricultural land situated in any area within such distance,
measured aerially, in relation to range of population as
shown hereunder –

70
Shortest aerial distance Population according to
from the local limits of a the last preceding census
municipality or of which relevant figures
cantonment board referred have been published
to in item (1) before the first day of
Previous Year.
Upto 2 kms More than 10,000
Exceeding 2 kms but upto 6
kms More than 1,00,000
Exceeding 6 kms but upto 8
kms More than 10,00,000
d. Specified Gold Bonds
e. Special Bearer Bonds, 1991
f. Gold Deposit Bonds
WHAT IS “TRANSFER”? {Section 2(47)}
It includes:
- Sale, Exchange or relinquishment of the asset;
- Extinguishment of any rights therein;
- Compulsory Acquisition;
- Conversion of Capital Asset into Stock-in-trade;
- Maturity or; redemption of a Zero-Coupon Bond (ZCB);
- Part performance of a contract
- Transactions having effect of enabling or transferring the
enjoyment of immovable property.

SHORT TERM CAPITAL ASSET and LONG-TERM CAPITAL ASSET:


- It depends on period of holding
- For asset to be short term, the period of holding is as follows for
different assets:

71
Held for not more Held for not more Held for not more
than 12 months than 24 months than 36 months
- Security - Unlisted - Unit of debt-
(other than Shares oriented
Unit) listed in - Land or fund.
a Recognised building or - Unlisted
stock both. Securities
exchange. other than
- Unit of Equity shares.
Oriented - Other Capital
Fund/Unit of Assets.
UTI.
- Zero Coupon
Bond.

In case, period of holding is more than above in case of given


assets, then it is Long Term Capital Assets.

Format for computation of Capital Gains:

Amount
PARTICULARS
(₹)
A. Full Value of Consideration xxxxx
B. Less: Expenditure on Transfer (e.g.:
Brokerage on sale, legal expenses) (xxxxx)
C. Therefore, Net Sale Consideration xxxxx
D. Less: Cost of Acquisition/Indexed Cost of
Acquisition, in case of LTCA (xxxxx)
E. Less: Cost of Improvement/ Indexed Cost of
Improvement, in case of LTCA (xxxxx)
F. Therefore, Short Term Capital Gains (STCG)/
Long Term Capital Gains (LTCA), as the case
may be xxxxx
G. Less: Exemptions under section
54/54B/54D/54F/54EC (xxxxx)
H. Therefore, Taxable LTCG/STCG, as the case
may be. xxxxx

72
Concept of INDEXATION – Logic and Formula with illustration:
Say in Financial Year 2001-02, an assessee purchased a land for
₹1,00,000, which he sold in Financial Year 2022-23 for ₹10,00,000.
Then the difference between sale Price and purchase price of land =
₹9,00,000.
But; Long Term Capital Gains shall be after considering indexed cost of
acquisition.
2001-02 2022-23
Cost of Acquisition 1,00,000 x = ? (ICOA)
CII 100 331
Here, x= 1,00,000 * 331/100 = ₹3,31,000
Therefore, Long Term Capital Gains = ₹10,00,000 (-) ₹3,31,000 =
₹6,69,000.
In short, deduction is allowed of 2 things:
a. Cost of Acquisition: ₹1,00,000
b. Inflation amount: ₹2,21,000
Formula for Indexed Cost of Acquisition (ICOA):
Cost of Acquisition X CII for the P.Y. in which the asset is transferred
CII for the P.Y. in which the asset is acquired
Similarly, Indexed Cost of Improvement (ICOI) shall be:
Cost of Improvement X CII for the P.Y. in which the asset is transferred
CII for the P.Y. in which improvement took
place
Let’s learn Capital Gains by dividing the topic in DIFFERENT CASES:
1. Transfer of such capital asset which was acquired prior to
01.04.2001:
- The base year for CII is 2001-02 for which the CII is 100. In case,
asset is acquired prior to 2001-02, then CII for the P.Y. in which

73
asset is acquired will not be determinable i.e. there will be no
CII available for the denominator.
- In such case, CII for the denominator is to be taken as "100".
- However, Cost of Acquisition can be taken as original cost of
acquisition or FMV of the Asset as on 01.04.2001, whichever is
HIGHER.
{Actually, the provision states that assessee should take either
of the two at his option. But always assessee would like COA to
be higher, hence the conclusion.}
- In case of cost of acquisition of land or building or both acquired
prior to 01.04.2001, it shall be original cost of acquisition or;
FMV as on 01.04.2001 (subject to Stamp Duty Value as on
01.04.2001}, whichever is higher.
- Any Cost of improvement incurred prior to 01.04.2001 shall be
ignored.

Illustration 1:
Gold Jewellery purchased by Mr. E in 1990-91 for ₹50,000 was
sold in P.Y. 2022-23 for ₹12,00,000. It’s FMV as on 01.04.2001
was ₹1,20,000. Compute Capital Gains.
Solution:
Computation of Long-Term Capital Gains for the Assessment
Year 2023-24
(Relevant to Previous Year 2022-23)
PARTICULARS Amount (₹)
a. Full Value of Consideration 12,00,000
b. Less: Indexed Cost of Acquisition
(₹1,20,000 * 331/100) (3,97,200)
c. Therefore, Long Term Capital Gains 8,02,800
Note:
Cost of Acquisition is taken as Original Cost of Acquisition i.e.
₹50,000 or FMV as on 01.04.2001 i.e. ₹1,20,000, whichever is
higher.

74
Illustration 2:
Original cost of acquisition of land as on 14th February, 1992 is
₹1,00,000. FMV as on 01.04.2001 is ₹1,50,000 and its Stamp Duty
Value as on 01.04.2001 is ₹1,75,000. What is the Cost of
Acquisition?

Solution:
Higher of the following shall be the Cost of Acquisition:

1. Original Cost of Acquisition


1,00,000
Or;
2. FMV as on 01.04.2001 or; Stamp Duty Value
(whichever is lower)
i.e. ₹1,50,000 or; ₹1,75,000, whichever is lower
1,50,000

Therefore, Cost of Acquisition shall be ₹1,50,000.

Illustration 3:
On 01.05.1985 assessee acquired a property being plot of land
for ₹1,00,000 in a cream area in the city. In the year 1995-96,
he constructed a house for ₹1,50,000 on the said plot. Further,
in Financial Year 2010-11, he builds 1st floor to the house by
incurring ₹20,00,000. Further in Financial Year 2016-17, he
renovated ground floor as well as first floor by incurring
₹15,00,000. In Financial Year 2022-23, he sold the entire
property for ₹1 crore. Compute Capital Gains for the
Assessment Year 2023-24.
FMV of the property as on 01.04.2001 was ₹4,00,000 {Stamp
Duty Value as on that date was ₹3,75,000}.
CII for P.Y. 2010-11: 167, P.Y. 2016-17: 264, P.Y. 2022-23: 331
Brokerage paid for transfer of this property is 0.5% while equal
are the legal charges.

75
Solution:
Computation of Long-Term Capital Gains for the Assessment
Year 2023-24 (Relevant to Previous Year 2022-23) on transfer
of plot of land with constructed house:
PARTICULARS Amount (₹)
a. Full Value of Consideration 1,00,00,000
b. Less: Expenditure on transfer
Brokerage {1,00,00,000 * 0.5%} 50,000
Legal Expenses 50,000 (1,00,000)
c. Therefore, Net Sale Consideration 99,00,000
d. Less: Indexed Cost of Acquisition
{₹3,75,000 * 331/100} (12,41,250)
e. Less: Indexed Cost of Improvement
Incurred during 1995-96
{To be neglected as incurred
prior to 01.04.2001} -
Incurred during 2010-11 for
Construction of first floor
(₹20,00,000 * 331/167) 39,64,072
Incurred during 2016-17 for
Renovation of both floors
(₹15,00,000 * 331/264) 18,80,682 (58,44,754)
f. Therefore, Long Term Capital Gains 28,13,996

2. Conversion of Capital asset into stock in trade:


- It is regarded as transfer.
- In such case, Full Value of Consideration shall be taken as “Fair
Market Value” as on date of conversion of capital asset into stock
in trade.
- These Capital Gains shall become taxable in that Previous Year in
which such stock in trade is actually sold.
- In the P.Y in which such stock is sold, along with Capital gains,
there shall also arise PGBP. For computing such PGBP, purchase
cost shall be taken as FMV as on date of conversion of Capital Asset
into Stock in trade.

76
Illustration 4:
In 2005-06, assessee purchased plot of land for ₹15,00,000 which is
his Capital Asset. In 2015-16, he decided to make plots of this land
and sell it to various buyers as plots in a residential gated
community. The development cost incurred on this plot is
₹35,00,000. In Financial Year 2022-23, finally all 7 plots from the
scheme were sold, each plot for ₹15,00,000. Further Advertisement
and other administration expenses incurred by the assessee were
₹5,00,000.
What is the income tax implication in the hands of assessee.
{FMV as on date of conversion of Capital Asset into Stock in Trade
was ₹25,00,000.}
CII for F.Y. 2005-06: 117, F.Y. 2015-16 is 254 & F.Y. 2022-23 is 331.

Solution:

Computation of Long-Term Capital Gains for the Assessment Year


2023-24
(Relevant to Previous Year 2022-23)
{As the converted stock in trade is actually sold in P.Y. 2022-23}
PARTICULARS Amount (₹)
a. Full Value of Consideration {FMV as on date
of conversion of Capital Asset into Stock in 25,00,000
Trade}
b. Less: Indexed Cost of Acquisition (32,56,410)
{₹15,00,000 * 254/117}
c. Therefore, Long Term Capital Gains (7,56,410)

Computation of Profits and Gains from Business or Profession for


the Assessment Year 2023-24 (Relevant to Previous Year 2022-23)
as stock in trade i.e. plots are sold in this P.Y.

77
PARTICULARS Amount (₹) Amount (₹)
a. Sale Price of all 7 plots of land
(₹15,00,000 x 7) 1,05,00,000
b. Less: Expenses incurred in
business:
Deemed purchase cost of this
plot of land 25,00,000
Development cost & other
expenses (35,00,000 +
5,00,000) 40,00,000 (65,00,000)
c. Therefore, Profits and Gains
from Business or Profession 40,00,000

3. COMPULSARY ACQUISITION:

- Compulsory acquisition is regarded as transfer.


- It is to be taxed in that Previous Year in which compensation or
part thereof is first received. However, benefit of indexation is
to be taken only upto the P.Y. in which asset is compulsorily
acquired.
- Here Full Value of Consideration shall be amount of
compensation.
- Further, if assessee is not satisfied with the amount of
compensation and say he gets the compensation enhanced,
then such enhanced compensation is also taxable under the
head Capital Gains in that P.Y. in which such compensation is
received. However, Cost of Acquisition and cost of improvement
shall be Nil while computing Capital Gains in case of Enhanced
compensation. It should be noted that Expenses on transfer in
such case could be considered say legal expenses for filing the
suit for getting the compensation enhanced.
- In case there is delay in release of Compensation or enhanced
compensation, then Government may pay interest on such
compensation or enhanced compensation. Such interest will be
taxable under the head Income from Other Sources. It should be

78
noted that deduction is allowed to the extent of 50% u/s 57
while computing such income to tax.
Illustration 5:
Mr.G purchased plot of land on 3rd May, 2003 for ₹5,00,000. He
incurred ₹50,000 on its improvement in Financial Year 2015-16. In
Financial Year 2018-19, it is compulsorily acquired by the State
Government, fixing the compensation at ₹25,00,000. Legal Expenses
incurred by Mr.G is ₹20,000. This amount was received as ₹5,00,000
on 1st May, 2022 and ₹20,00,000 on 30th June, 2023. Assessee not
being satisfied with the quantum of compensation had filed a legal
suit to get the compensation enhanced. Fees for the same ₹30,000.
The compensation was enhanced by ₹15,00,000 & it was received
entirely on 6th August, 2024. Along with it a lumpsum interest of
₹3,00,000 was received for delay in payment of compensation as well
as enhanced compensation. Discuss its taxability in the hands of Mr.G
for the relevant Assessment Years.
CII for F.Y. 2018-19 – 280, F.Y. 2003-04 – 109, F.Y. 2015-16 - 254 & F.Y.
2022-23 – 331.
SOLUTION:
Computation of Taxable Long Term Capital Gains for Mr.G on
compulsory acquisition of land for the Assessment Year 2023-24
{Relevant to Previous Year 2022-23} as compensation or part thereof
on acquisition of land is first received in P.Y.2022-23
PARTICULARS Amount (₹)
A. Full Value of Consideration 25,00,000
B. Less: Expenditure on Transfer (Legal Expenses) (20,000)
C. Therefore, Net Sale Consideration 24,80,000
D. Less: Indexed Cost of Acquisition
{₹5,00,000 x 280/109) (12,84,404)
E. Less: Indexed Cost of Improvement
(₹50,000 x 280/254) (55,118)
F. Therefore, Long Term Capital Gains 11,40,478

79
Note: Here, Previous Year in which asset is transferred is the Previous
Year in which asset is compulsorily acquired i.e. P.Y. 2018-19.

Computation of Long-Term Capital Gains for Assessment Year 2025-


26 {Relevant to Previous Year 2024-25} as enhanced compensation is
actually received in Previous Year 2024-25
PARTICULARS Amount (₹)
a. Full Value of Consideration {Being Enhanced
Compensation} 15,00,000
b. Less: Expenditure on Transfer {Being legal fees} (30,000)
c. Therefore, Net Sale Consideration 14,70,000
d. Less: Indexed Cost of Acquisition & Indexed
Cost of Improvement
{It shall be NIL being for the purpose of
Enhanced Compensation} -
e. Therefore, Taxable Long Term Capital Gains 14,70,000

Note:
Since the original asset for which the compensation is received is a
LONG TERM CAPITAL ASSET, even the enhanced compensation
received shall be a long term capital gains.

Computation of Income from Other Sources {Interest on


compensation & enhanced compensation} for the Assessment Year
2025-26 {Relevant to Previous Year 2024-25} as such interest amount
is received on 06.08.24
PARTICULARS Amount (₹)
a. Total Interest received 3,00,000
b. Less: Deduction u/s 57 of the Act @ 50% (1,50,000)
c. Therefore, amount of interest taxable under the
head Income from Other sources 1,50,000

Note:
Interest on Compensation or enhanced compensation is taxable in
that P.Y. in which such interest is actually received.

80
4. Capital Gains on transfer of depreciable asset:
Already covered while doing topic of PGBP.
Illustration 6:
Singhania and Company, a sole proprietorship owns 6 machines,
put in use for business in March, 2021. The depreciation on these
machines is charged at 15%. The Opening balance of these
machines after providing depreciation for Previous Year 2021-22
was ₹8,50,000. Three of the old machines were sold on 10th June
2022 for ₹11,00,000. A second-hand plant was bought for ₹8,50,000
on 30th November, 2022.
You are required to:
a. Determine the claim of depreciation for Assessment Year
2023-24.
b. Compute the Capital Gains liable to tax for the Assessment
Year 2023-24.
c. If Singhania and Company had sold the three machines in
June 2022 for ₹21,00,000, will there be any difference in
your above workings? Explain.

SOLUTION:
If 3 of the old machines were sold for ₹6,00,000:
Computation of Depreciation for the Assessment Year 2023-24
(Relevant to Previous Year 2022-23):
PARTICULARS Amount (₹)
A. Opening Written Down Value i.e. WDV
as on 01.04.2022 8,50,000
B. Add: Additions at Actual Cost during the
Previous Year 8,50,000
C. Therefore, Sub Total 17,00,000
D. Less: Sale proceeds from assets from the
block sold during the year (11,00,000)

81
E. Therefore, WDV for depreciation 6,00,000
F. Hence, Depreciation @ ½ of 15% as
₹6,00,000 is out of purchases during the
year put to use for less than 180 days 45,000
G. Therefore, Opening WDV as on 5,55,000
01.04.2023

Note: There shall be No Capital Gains in this case as sale


proceeds of the block of assets does not exceed the total of
“Opening WDV + Additions”. Also, the block of asset
continues to exist.

If 3 machines were sold for ₹21,00,000:


{Here, Sale Proceeds exceed “opening WDV + Additions” &;
therefore, there shall arise Short Term Capital Gains under section
50 of Income Tax Act}
COMPUTATION OF STCG on sale of Machines for the
Assessment Year 2023-24
(Relevant to Previous Year 2022-23):
PARTICULARS Amount (₹)
A. Full Value of Consideration/ Net Sale
Consideration 21,00,000
B. Less: Cost of Acquisition & Cost of
Improvement {Being Opening WDV +
Additions i.e. ₹8,50,000 + ₹8,50,000}
(17,00,000)
C. Therefore, Short Term Capital Gains
4,00,000

What would be your answer in the original question if all the


assets were sold for ₹15,00,000?
{Here, Block of Assets cease to exist and sale proceeds are lower
than “Opening WDV + Additions”, therefore, there is STCL}
82
COMPUTATION OF SHORT-TERM CAPITAL LOSS on sale
of Machines for the Assessment Year 2023-24
(Relevant to Previous Year 2022-23):
PARTICULARS Amount (₹)

a. Full Value of Consideration/ Net Sale


Consideration 15,00,000
b. Less: Cost of Acquisition and Cost of
Improvement (17,00,000)
c. Therefore, Short Term Capital Loss
(2,00,000)

5. SLUMP SALE
✓ Slump sale means entire business unit/division is
transferred for a LUMPSUM CONSIDERATION without
assigning individual values to any assets/liabilities.

✓ Full value of Consideration: Lumpsum consideration


received or; FMV of capital assets transferred, whichever
is higher.

✓ In case, the Unit transferred is in existence for more than 36


months, then it gives rise to Long Term Capital under
section 50B. However, benefit of Indexation is not
available while computing Capital Gains under section 50B.

✓ However, if the unit is in existence for not more than 36


months before its transfer, then there shall arise Short Term
Capital Gains.

83
✓ Cost of Acquisition and Cost of Improvement shall be
taken as "NET WORTH" of the undertaking/Unit
transferred. Here, Net Worth means "ASSETS (-) OUTSIDE
LIABILITIES".
With respect to Depreciable Assets, WDV as per section
43(6) of Income Tax Act should be considered.

Further w.r.t. Other than Depreciable Assets, their BOOK


VALUE excluding the effect of REVALUATION shall be
considered.
And; Liabilities shall be considered at its BOOK VALUES.

ILLUSTRATION 7:
Mr. A is a proprietor of Akash Enterprises having 2 units.
He transferred on 01.04.2022 his Unit 1 by way of slump sale
for a total consideration of ₹25,00,000. The Fair Market Value
of the Capital Assets of Unit 1 on 01.04.2022 was ₹30,00,000.
Unit 1 was started in 2005-06. The expenses incurred on the
transfer were ₹28,000. His Balance Sheet as on 31.03.2002 is
as under:
Figures in ₹
Liabilities Total Assets Unit 1 Unit 2 Total
Own Capital
15,00,000 Building 12,00,000 2,00,000 14,00,000

Revaluation
Reserve (For 3,00,000 Machinery 3,00,000 1,00,000 4,00,000
building of Unit 1)
Bank Loan (70% for
2,00,000 Debtors 1,00,000 40,000 1,40,000
unit 1)
Trade Creditors Other
1,50,000
(25% for Unit 1) Assets 1,50,000 60,000 2,10,000

Other Information:
1. Revaluation reserve is created by revising upward the value
of the building of Unit 1.
2. No individual value of any asset is considered in the transfer
deed.
84
3. Other assets of Unit 1 include patents acquired on 01.07.2020
for ₹50,000 on which no depreciation has been charged.
Compute Capital Gain for the Assessment Year 2023-24.
SOLUTION:
Computation of Capital Gains on Slump Sale for the
Assessment Year 2023-24
(Relevant to Previous Year 2022-23):
PARTICULARS Amount (₹)
A. Full Value of Consideration {Higher of
FMV of Capital Assets of Unit 1 on
30,00,000
01.04.2022 or; FMV of monetary
consideration received.}
B. Less: Expenses on Transfer (28,000)
C. Therefore, Net Sale Consideration 29,72,000
D. Less: Cost of Acquisition and Cost of
Improvement {Net Worth – Note} (12,50,625)
E. Therefore, Long Term Capital Gains 17,21,375
Notes:
1. Computation of Net Worth of Unit 1 of Akash Enterprises:

PARTICULARS Amount (₹) Amount (₹)

A. Building {excluding ₹3
lakh on account of 9,00,000
revaluation}
B. Machinery 3,00,000
C. Debtors 1,00,000
D. Patents (Note) 28,125
E. Other Assets (₹1,50,000 (-)
₹50,000) 1,00,000
Total Assets 14,28,125
Less:
Creditors (₹1,50,000 * 25%) 37,500
Bank Loan (₹2,00,000 * 70%) 1,40,000 (1,77,500)
Therefore, Net Worth 12,50,625
85
2. Written Down Value of Patents as on 01.04.2022:
PARTICULARS Amount (₹)
a. Cost as on 01.07.2020 50,000
b. Less: Depreciation for P.Y. 2020-21 @
25% (12,500)
c. WDV as on 01.04.2021 37,500
d. Less: Depreciation for 2021-22 @ 25%
(9,375)
e. Therefore, WDV as on 01.04.2022 28,125

3. The problem has been solved assuming that the balance sheet
values of ₹3,00,000 and ₹9,00,000 represent the WDV as per
section 43(6) of Income Tax Act of machinery and building
respectively of Unit 1.

4. Since the Unit 1 is in existence for more than 36 months,


Capital Gains arising would be Long Term Capital Gains.
However, Indexation benefit is not available in case of slump
sale.

6. Transfer of such capital asset which is acquired by way of gift


or will or inherited:
• In such cases, the Cost of Acquisition is to be taken as "cost
to the previous owner".
• Further, for determining the period of holding for the
purposes of ascertaining whether the Capital gains is short
term or long term, the period of holding of previous owner
shall also be considered.
• Moreover, according to the decision given by Bombay HC
in the case of Manjula Shah, the CII for the P.Y. in which
asset is acquired by the previous owner can be considered
in the denominator for computing Indexed Cost of
Acquisition. {In short, benefit of indexation can be taken
from the year asset was acquired by the previous owner.}
86
ILLUSTRATION 8:
Mr. A purchased a residential property for ₹40,00,000 on
01.02.2004. In Financial Year 2012-13, he incurred some
expenses of ₹10,00,000 for renovation of this property. On
3rd March, 2016, he gifted the said property to his son Mr.B.
On 14th March, 2023, Mr.B sold this property for ₹1.75 crore.
He incurred legal expenses of 1% on selling this property.
Further, brokerage is 1.5% of sale consideration.
Earlier in F.Y.2018-19, Mr. B spent ₹8,00,000 on further
renovation of the property.
Compute Capital Gains in hands of Mr.B.
{CII for P.Y.: 2003-04: 109, P.Y 2012-13: 200, P.Y. 2018-19: 280,
P.Y.2022-23: 331}

SOLUTION:
Computation of Long Term Capital Gains for Mr.B on sale
of residential property acquired by way of gift from his
father for Assessment Year 2023-24 {Relevant to Previous
Year 2022-23}:

PARTICULARS Amount (₹)

a. Full Value of Consideration 1,75,00,000


b. Less: Expenditure on transfer
Legal Expenses 1,00,000
Brokerage 1,50,000 (2,50,000)
c. Therefore, Net Sale Consideration 1,72,50,000
d. Less: Indexed Cost of Acquisition
{₹40,00,000 x 331/109}
Note: Manjula Shah decision is
(1,21,46,789)
followed.
e. Less: Indexed Cost of
Improvement:
87
Improvement incurred by Mr.A (In
F.Y. 2012-13)
₹10,00,000 * 331/200 (16,55,000)
Improvement incurred by Mr.B (In
F.Y. 2012-13)
₹8,00,000 * 331/280 (9,45,714)
Therefore, Long Term Capital
25,02,497
Gains

7. Transfer of such capital asset w.r.t. which advance


money was received earlier and forfeited as negotiations
of transfer failed.
If advance was received If advance was received
and forfeited before and forfeited on or after
01.04.2014 01.04.2014
Advance forfeited to be Advance forfeited to be
deducted while determining taxed under section 56 as
Cost of Acquisition for Income from Other Sources
computing Capital Gains.
Taxability is therefore Tax liability is attracted in
postponed to the year of the year of forfeiture of
actual transfer of Capital advance.
asset.

ILLUSTRATION 9:
On 1st January, 2011, an advance of ₹2,50,000 was received
by Mr.A for his property. However, the negotiations failed
and the advance was forfeited. On 12th February, 2023, the
property was sold for ₹90,00,000. Legal expenses on transfer
are 1% while brokerage is 2% of sale consideration. The
original purchase cost of the property was ₹10,00,000,
purchased in 2001-02. Further, in F.Y. 2016-17, improvement
cost incurred is ₹6,00,000. Compute Capital Gains for the
A.Y. 2023-24.

88
SOLUTION:
Computation of Long-Term Capital Gains for the
Assessment Year 2023-24
{Relevant to Previous Year 2022-23} for Mr. A on transfer
of his property.
PARTICULARS Amount (₹)
A. Full Value of Consideration 90,00,000
B. Less: Expenses on Transfer
Legal Expenses 90,000
Brokerage 1,80,000 (2,70,000)
C. Therefore, Net Sale Consideration 87,30,000
D. Less: Indexed Cost of Acquisition
{10,00,000 (-) 2,50,000} * 331/100 (24,82,500)
E. Less: Indexed Cost of Improvement
{₹6,00,000 * 331/264} (7,52,273)
F. Therefore, Long Term Capital
54,95,227
Gains

Note:
Since Advance received and forfeited is before 01.04.2014, it
is reduced from the cost of acquisition while computing
Capital Gains in that P.Y. in which the property is actually
transferred.
What would be your answer in the above case, if the said
advance was received and forfeited in F.Y.2018-19?
Then amount of advance received and forfeited shall be
taxable under the head Income from Other Sources in
P.Y.2018-19 i.e. the P.Y. in which such advance is received
and forfeited. {It is taxable in such a way as the advance
received and forfeited is on or after 01.04.2014.}
Further, while considering Cost of Acquisition at the time of
transfer in the P.Y.2022-23, there shall be no adjustment for
such advance received and forfeited as it is already taxed.
89
Therefore, Cost of Acquisition considered shall be
₹10,00,000.
Capital Gains taxable for A.Y. 2023-24 (Relevant to P.Y.
2022-23) shall be computed as follows:
PARTICULARS Amount (₹)
a. Net Sale Consideration {Same as
87,30,000
computed above.}
b. Less: Indexed Cost of Acquisition
{10,00,000 * 331/100} (33,10,000)
c. Less: Indexed Cost of
Improvement (7,52,273)
d. Therefore, Long Term Capital
46,67,727
Gains

8. Transfer of Immovable Property – Determination of Full


Value of Consideration:
Section 50C
➢ If Stamp Duty Value is greater than 110% of Actual
Consideration, then, Full Value of Consideration shall
be STAMP DUTY VALUE.
➢ However, if Stamp Duty Value is not exceeding 110%
of Actual Consideration, then Actual Consideration
itself is the FVC.

ILLUSTRATION 10:
Say ₹100 lakhs is Actual sale price of an immovable property
while its Stamp Duty Value as on date of Transfer is ₹105
lakhs, then what shall be the Full Value of Consideration?
SOLUTION:
Here, 110% of Actual Consideration = ₹110 lakhs. Therefore,
Stamp Duty Value is lower than 110% of Actual
Consideration.
90
Therefore, Actual Consideration itself can be taken as FVC
i.e. ₹100 lakhs.
What would be your answer if the Stamp Duty Value was
₹112 lakhs instead of ₹105 lakhs?
SOLUTION:
FVC shall be taken as Stamp Duty Value i.e. ₹112 lakhs as
Stamp Duty Value exceeds 110% of the Actual
Consideration.

91
EXEMPTIONS UNDER SECTION 54, 54EC & 54F:

POINT SECTION 54 SECTION 54EC SECTION 54F

Exemption is House (LTCA) to House Land/Building (LTCA) to Any LTCA other than
about? Bonds House to House
Who can claim? Individual & HUF Any Assessee Individual & HUF
Asset Residential House Property Long Term Capital Asset Any Long-Term Capital
Transferred? being Long Term being Land or building or Asset other than
both residential house.
Amount Invested Purchasing House/ National Highway Investment is done in a
In? Constructing House in Authority of India (NHAI) Residential House.
India {Only 1 Residential Bonds or Rural
House where LTCG Electrification Corporation
exceeds ₹2 crore.} (REC) Bonds.
Time Limit for In case of purchase of within 6 months from the same as given in section
investment? house: within 1 year before date of transfer. 54
the date of transfer or
within 2 years after date of
transfer.
In case of Construction:
within 3 years from date of
transfer of house.
Quantum of Amount of LTCG or Amount invested or LTCG, If entire amount of Net
Exemption? Amount invested in House whichever is lower. Sale Consideration is
Property/ies, whichever is However, the aggregate of invested, then the entire
lower. amount invested in such amount of Long Term
Bonds during the P.Y. in Capital Gains.
which asset is transferred However, if amount less
and in the subsequent P.Y. than NSC is invested,
cannot exceed ₹50 lakhs. then exemption shall be
on PROPORTIONAL
BASIS#.
Lock in Period 3 years from the date of it’s 5 years from the date of 3 years from the date of
purchase. purchase of the bonds. it’s purchase.
Also, within these 5 years
loan cannot be taken
keeping these bonds as
security.
Consequence if To compute Capital Gain Amount earlier claimed as LTCG earlier claimed as
transferred on sale of such property, exemption shall become exemption shall become
within lock in the COA = Original Cost of taxable in the P.Y. in which taxable in that P.Y. in
period Acquisition (-) LTCG the condition is violated. which such new house
earlier claimed as is transferred.
exemption.

92
NOTE FOR “SECTION 54EC”:
Whenever any assessee in business transfers an asset which is
depreciable asset being building after holding it for more than 24
months, it is a Long-Term Capital Asset even though the resultant
capital gains are short term capital gains under section 50.
Therefore, on transfer of such an asset, such assessee can claim
exemption under section 54EC if other conditions are satisfied.

NOTE FOR “SECTION 54F”:


# Proportional basis:
Long Term Capital Gains X Amount Invested
Net Sale Consideration

Additional Conditions for section 54F:


1. Assessee should not own more than one house as on date of
transfer of LTCA.
2. He should not purchase another house within 2 years from
the date of transfer or construct another house within 3 years
from the date of transfer. If purchased/constructed, then the
LTCG earlier claimed as exemption shall become taxable.

CAPITAL GAINS DEPOSIT ACCOUNT SCHEME {CGDA}:


▪ Time Limit for investment in House u/s 54, 54F of Income
Tax Act is 2 years from the date of transfer in case of
purchase of house while 3 years from the date of transfer in
case of construction of house. Hence, it may happen that for
capital gains arising from property transferred in a
particular P.Y., exemption is to be claimed u/s 54 or section
54F but investment in house is not done upto due date of
filing of ITR. In such cases, the assessee's intention of

93
investing in house in future can be displayed by depositing
the amount in CGDA on or before due date of filing of ITR.

▪ Exemption allowed shall be equal to amount deposited in


CGDA or amount of LTCG, whichever is lower.
▪ However, in future, the amount from CGDA should be
utilized necessarily for purchase/construction of house &
that too within time limits prescribed u/s 54 / section 54F.
▪ In case, the amount is withdrawn and utilized for purpose
other than investment in house, then such amount gets
taxable under the head capital gains in that P.Y. in which it
is so utilized. Further, if the amount remains unutilized in
CGDA for 3 years from the date of transfer, then on expiry
of 3 years, it will become taxable under the head Capital
Gains.

ILLUSTRATION 11:
Mr. Sunil entered into an agreement with Mr. Dhaval to sell his
residential house located at Navi Mumbai on 16.08.2022 for
₹80,00,000. The sale proceeds was to be received in following
manner:
a. 20% through account payee bank draft on the date of
agreement.
b. 60% on the date of possession of property.
c. Balance after the completion of registration of the title to the
property.
Mr. Dhaval was handed over the possession of the property on
15.12.2022 and the registration process was completed on
14.01.2023. He paid the sale proceeds as per the sale agreement.
The value determined by stamp duty authority on 16.08.2022 was
₹90 lakhs whereas on 14.01.2023 it was ₹91,50,000.

94
Mr. Sunil had acquired the property on 01.04.2001 for ₹10,00,000.
After recovering the sale proceed from Mr. Dhaval, he purchased
a residential house property for ₹35,00,000.
Compute the income under the head Capital Gains for the
A.Y.2023-24. Also compute tax liability.
CII for 2001-02: 100, CII for 2002-03: 105 & CII for 2022-23: 331.

SOLUTION:
Computation of Long Term Capital Gains of Mr.Sunil for the
Assessment Year 2023-24 {Relevant to Previous Year 2022-23} on
transfer of residential house property and tax liability thereon:
PARTICULARS Amount (₹)
a. Full Value of Consideration/ Net Sale
Consideration 90,00,000
b. Less: Indexed Cost of Acquisition
{₹10,00,000 * 331/100} (33,10,000)
c. Therefore, Long Term Capital Gains
56,90,000
d. Less: exemption under section 54 (35,00,000)
e. Therefore, Long Term Capital Gains 21,90,000
f. Tax Liability
(₹21,90,000 (-) basic exemption limit:
₹2,50,000) * 20% 3,88,000
Add: Health and Education Cess @ 4% 15,520
Therefore, Total Tax Liability 4,03,520

NOTES:
1. Since, part of the consideration was received on the date of
agreement by way of account payee bank draft, the Stamp
Duty Value as on date of agreement shall be considered.
Therefore, Stamp Duty Value as on 16.08.2022 i.e. ₹90,00,000
is considered. As per section 50C of the Income Tax Act,
since Stamp Duty Value exceeds 110% of actual
95
consideration i.e. ₹90 lakhs is greater than ₹88 lakhs (₹80
lakhs * 110%), Stamp Duty Value shall be taken as Full Value
of Consideration.
2. Exemption under section 54 of Income Tax Act is available
in this case as on transfer of one residential property, an
assessee is investing in another one residential property in
India and it is assumed that such investment is made within
the prescribed time limits under the said section.
The quantum of exemption is taken as long-term capital
gains i.e. ₹56,90,000 or amount invested i.e. ₹35,00,000,
whichever is lower. Hence, Exemption is ₹35,00,000.

ILLUSTRATION 12:
Mr. Kumar is owner of residential house which was
purchased in September, 2003 for ₹7,00,000. He sold the said
house on 5th August, 2022 for ₹30,00,000. Valuation as per
stamp valuation authority of said plot of land was
₹44,00,000. He invested ₹8,00,000 in NHAI Bonds on 12th
January, 2023. He purchased a residential house on 8th
September, 2022 for ₹12,00,000. He gives other particulars as
follows:
a. Interest on Bank Deposit: ₹32,000
b. Investment in Public Provident Fund: ₹12,000
You are requested to compute taxable income for the A.Y.
2023-24.
CII for 2003-04: 109 & for P.Y. 2022-23: 331.

SOLUTION:
Computation of Taxable Income for Mr. Kumar for the
Assessment Year 2023-24
{Relevant to Previous Year 2022-23}

96
PARTICULARS Amount (₹)

a. Full Value of Consideration/ Net Sale


Consideration 44,00,000
b. Less: Indexed Cost of Acquisition
{7,00,000 * 331/109} (21,25,688)
c. Long Term Capital Gains 22,74,312
d. Less: Exemption u/s 54 (12,00,000)
e. Less: Exemption u/s 54EC (8,00,000)
f. Therefore, Taxable LTCG 2,74,312
g. Add: Income from Other Sources
(Interest on Bank Deposit) 32,000
h. Gross Total Income 3,06,312
i. Less: Deductions under chapter VI-A
u/s 80C (Deposit in PPF) (12,000)
j. Therefore, Total Income 2,94,312

Note:
Since, Stamp Duty Value is GREATER than 110% of Actual
Consideration, Stamp Duty Value itself is taken as Full
Value of Consideration - Section 50C of Income Tax Act.
{₹44,00,000 is greater than 110% of ₹30,00,000 i.e.₹33,00,000.}

ILLUSTRATION 13:
Mr.S purchased a plot of land on 3rd February, 2000 for
₹10,00,000. It's Fair Market Value as on 1st April, 2001 was
₹9,80,000. He incurred ₹50,000 on improvement in F.Y.2020-
21. Further in the year F.Y.2022-23, he sold this plot of land
for ₹97,00,000 by incurring expenditure on transfer
amounting ₹2,00,000. Further, on 5th May, 2023 he
purchased a residential house for ₹50,00,000. Compute
taxable Capital Gains for the Assessment Year 2023-24. It is

97
given that he does own any house as on the date of transfer
of the plot of land.
SOLUTION:
Computation of Long-Term Capital Gains for the
Assessment Year 2023-24 {Relevant to Previous Year 2022-
23} on transfer of plot of land:
PARTICULARS Amount (₹)
a. Full Value of Consideration 97,00,000
b. Less: Expenditure on Transfer (2,00,000)
c. Therefore, Net Sale Consideration 95,00,000
d. Less: Indexed Cost of Acquisition
(₹10,00,000 * 331/100) (33,10,000)
e. Less: Indexed Cost of Improvement
{₹50,000 * 331/301} (54,983)
f. Therefore, Long Term Capital Gains 61,35,017
g. Less: Exemption u/s 54F
{61,35,017 * 50,00,000/95,00,000} (32,28,956)

h. Therefore, Taxable LTCG 29,06,060

Cases in Capital Gains {Continued…}


9. CAPITAL GAINS ON TRANSFER OF SHARES:

98
Shares acquired as
1. Original Shares

2. Bonus Shares

3. Right Shares

Cost of Acquisition: -
1. Original Shares: Cost of Acquisition (COA) shall be the purchase
price of these shares.

2. For Bonus Shares:

Allotted before • COA = FMV of these


01.04.2001 shares as on
01.04.2001

Allotted on or
after • COA = NIL
01.04.2201

3. Right Shares: It shall be the amount paid by assessee to acquire


the share.

99
TYPES OF CAPITAL GAINS ON TRANSFER OF
SHARES:

LISTED UNLISTED
SHARES SHARES

STCG: TAXABLE U/S STCG: TAXABLE at


111A @ 15% normal rates.

LTCG: TAXABLE U/S


112A @ 10% on LTCG: Taxable @
LTCG exceeding 20% u/s 112.
₹10 lakhs.

COST OF ACQUISITION IN CASE OF SHARES WHERE THE TAXABILITY


IS u/s 112A:
a. With respect to shares purchased on or after 01.04.2018:
Original Cost of Acquisition.
b. With respect to shares purchased before 01.04.2018:

1. Consider - Original Cost of Acquisition (COA)

2. Take - FMV of such share as on 31.01.2018 &; Full


Value of Consideration, whichever is lower.

3. COA = HIGHER of 1 & 2

WHAT IS FMV in such cases?


➢ In case where share is listed on any Recognised Stock Exchange
as on 31.01.2018:
- Trading in such asset as on 31.01.2018 has taken place:
100
Highest Price of such share on that date.
- No Trading has taken place on 31.01.2018:
Highest Price on day preceding 31.01.2018 when such asset
was traded.

ILLUSTRATION 14:
PARTICULARS 1 2 3 4
a. Original COA of
share before
01.04.2018 100 100 100 100
b. FMV as on
31.01.2018 200 200 50 200
c. FVC w.r.t.
transfer of this
share 250 150 150 50
SOLUTION:
HIGHER OF
1. COA 100 100 100 100
2. FMV but
subject to
FVC {i.e.
FMV not to 200 150 50 50
exceed FVC}
3. Therefore, 200 150 100 100
COA
4. Hence,
Resultant 50 0 50 (50)
LTCG

*In short, Cost of Acquisition shall be higher of Original Cost of


Acquisition or; FMV as on 31.01.2018 (FMV restricted to FVC).

101
Illustration 15:

1. Mr. Mithun purchased 100 equity shares of M/s Goodmoney


Co. Ltd. as on 01.04.2006 at ₹1,000 per share in public issue of
the company by paying Securities Transaction Tax (STT).
Company allotted bonus shares in ratio of 1:1 on 01.12.2021.
He has also received dividend of ₹10 per share on 01.05.2022.
He sold all the shares on 01.10.2022 at ₹4,000 per share
through a recognised stock exchange and paid brokerage of 1%
and Securities Transaction Tax of 0.02% to celebrate his 75th
birthday.
Compute his total income and tax liability for the assessment
year 2023-24 assuming he is having no income other than
given above. Fair Market Value of shares of M/s Goodmoney
Co. Ltd. on 31.01.2018 is ₹2,000.

Solution:

Computation of Total Income and Tax Liability of Mr. Mithun


for the Assessment Year 2023-24
{Relevant to Previous Year 2022-23}:
PARTICULARS Amount (₹)
A. Capital Gains
Long Term Capital Gains:
Full Value of Consideration on transfer
of 100 shares – originally acquired
{₹4,000 * 100} 4,00,000
Less: Expenditure on transfer
{Brokerage} (4,000)
{1% of ₹4,00,000}
Therefore, Net Sale Consideration 3,96,000
Less: Cost of Acquisition {Note}
{₹2,000 per share * 100 shares} (2,00,000)
Therefore, Taxable Long Term Capital
Gains 1,96,000
Short Term Capital Gains:

102
Full Value of Consideration on transfer
of 100 bonus shares {₹4,000 * 100} 4,00,000
Less: Expenses on Transfer {Brokerage}
{1% of ₹4,00,000} (4,000)
Net Sale Consideration 3,96,000
Less: Cost of Acquisition {NIL, as the
bonus shares are acquired on or after (Nil)
01.04.2001}
Therefore, Taxable Short Term Capital
Gains 3,96,000
B. Income from Other Sources
Dividend from Domestic Company {Fully
Taxable in hands of the assessee i.e.
recipient. Therefore, ₹10 per share *
2,000 shares} 2,000
C. Therefore, Gross Total Income 5,94,000
D. Tax liability
On Dividend Income {₹2,000 (-)
Exemption of ₹2,000 utilised from basic
exemption limit of ₹3,00,000, assessee
being resident senior citizen.} -
On Short Term Capital Gains, Tax u/s
15% u/s 111A {₹3,96,000 (-) ₹2,98,000,
being unexhausted basic exemption 14,700
limit} * 15%
On Long Term Capital Gains u/s 112A @
10% on such LTCG as exceeds ₹1,00,000
{₹1,96,000 (-) ₹1,00,000} * 10% 9,600
Therefore, Total Tax 24,300
Add: Health and Education Cess @ 4% 972
Therefore, Tax liability 25,272
Rounded off to nearest of ₹10 u/s 288B 25,270
Notes:
1. Securities Transaction Tax is not allowed as deduction.
2. Cost of Acquisition of listed shares acquired before
01.02.2018 shall be taken as higher of following 2: -
a. Original Cost of Acquisition ₹1,000 per share

103
Or;
b. Lower of:
FMV as on 31.01.2018: ₹2,000
And; ₹2,000 per share
FVC of the share: ₹4,000

Hence, Cost of Acquisition shall be ₹2,000 per share,


being higher of a and b.

OTHER POINTS IN CAPITAL GAINS:

1. Taxability of Capital Gains in case of Specified


Agreements:
• Applying definition of transfer in certain cases,
under the development agreements, the
transfer took place in the year in which the
owner of the immovable property, handed over
the said property to the developer.
• Consequently, the capital gains tax liability in the
hands of owner would arise in the year in which
possession of immovable property is handed
over to the developer for development of
project, in spite of the fact that the consideration
thereof will be received later, say after couple of
years.
• To minimise the genuine hardship to assessee in
paying capital gains tax in the year of transfer,
section 45(5A) provides that
- in case of assessee being individual or HUF,
- who enters into a specified agreement for
development of a project,
- the capital gains arising from such transfer shall
be chargeable to Income Tax as income of the
previous year in which the certificate of

104
completion for the whole or part of the project
is issued by the competent authority.

Illustration 16:
A Plot of land belonging to Mr.X is handed over to a Construction
Company in a Joint Development Agreement on 15.12.2022. The
agreed consideration is 2 Flats in the said new scheme and ₹5 crores.
Here, the tax liability on transfer of "Plot of Land" under Joint
Development Agreement would not arise immediately in F.Y. 2022-23
but; it will arise in that P.Y. in which the construction is complete and
Occupancy Certificate/Completion Certificate is given by the Authority.
In that year, FVC for computing CG shall be "₹5 crores + SDV of 2 Flats".
In future, if one of such flat is sold by Mr.X, then it's COA shall be FVC
i.e. SDV of this flat on basis of which earlier he had charged Capital Gains
to Tax.

2. BUY BACK OF SHARES


{Section 115QA of Income Tax Act}:
• Where Domestic Company buys back shares,
then w.r.t. amount distributed as profits to the
shareholders, the company is liable to pay tax.
• Such tax is at 20% of amounts distributed as
profits to the shareholders + SC @ 12% + H&EC @
4%.
• Such amount received on buy back of shares is
FULLY EXEMPT in the hands of Shareholders
under section 10(34A).
Illustration 17:
ABC Pvt. Ltd. bought back its 1,00,000 shares @ ₹90 each. They were
issued at ₹20 each 3 years back. Compute the amount of tax payable on
buy back of shares in the hands of the company under section 115QA
105
of Income Tax Act, 1961. Also state what shall be tax implications in the
hands of shareholders.

SOLUTION:
Computation of tax liability of the Company ABC Pvt. Ltd. on buy back
of its own shares:
PARTICULARS Amount (₹)
a. Amount paid on buy back of 1,00,000 shares @ ₹90
each 90,00,000
b. Less: Issue price of such shares {₹1,00,000 x 20} (20,00,000)
c. Therefore, Profit distributed on buy back of shares 70,00,000
d. Therefore, Tax @ 20% u/s 115QA 14,00,000
Add: Surcharge @ 12% 1,68,000
Sub Total 15,68,000
Add: Health and Education Cess @ 4% 62,720
Therefore, Total Tax 16,30,720
Note:
In hands of shareholders, such amount received on buy back of shares
shall be fully exempt u/s 10(34A) of Income Tax Act, 1961.

3. COST OF ACQUISITION OF INTANGIBLE ASSETS:

Goodwill in Business Goodwill in Profession


Purchased Goodwill
(Self Generated) (Self Generated)
• COA = NIL • COA: Cannot be • COA = Amount paid
• Therefore, Taxable. determined{B.C. to the seller of
Srinivasa Shetty - business from whom
Hon. SC decision} Goodwill is
• Therefore, No Capital purchased.
Gains.

106
INCOME FROM OTHER SOURCES
Charging Section: Section 56
It taxes specific incomes under this head: -
a. Dividend
b. Interest on Securities
c. Winnings from Lotteries, Horse Races, Card Games, any other
casual income.
d. Gifts {Receipts without consideration or for inadequate
consideration}
Income from Other Sources is also a residuary head of income i.e.
incomes which could not be taxed in first four heads of income will be
taxed under this head.
Conditions:
a. It should be Income.
b. It should not be exempt.
c. It should not be taxable under any other heads of income.
Examples: -
• Salary received by Member of Parliament/ Member of Legislative
Assembly {There is no employer – employee relation between
payer and payee. Therefore, it cannot be taxed under the head
salaries. Hence taxed under the head Income from Other
Sources}.
• Interest on Fixed Deposit
• Interest on Savings Bank Account
• Income from Agricultural land in the foreign country
• Income from Sub Letting of property
• Income from letting of Vacant Land
• Family Pension i.e. Pension amount received by Legal Heir from
employer of deceased employee.
• Royalty Income

107
GIFTS
GIFTS

Received from
Received from
other than
employer
employer

On ocassion of In Kind:
In Cash:
MARRIAGE of Any other Gift. Not Taxable upto
assessee. FULLY TAXABLE
₹5,000 p.a.

Received from Received from


NOT TAXABLE.
RELATIVE @ NON RELATIVE

IN KIND i.e. of
NOT TAXABLE IN CASH
PROPERTY

Exceeds ₹50,000
Does not exceed MOVABLE IMMOVABLE
in aggregate
₹50,000 PROPERTY* PROPERTY**
during PY

Fully Taxable NOT TAXABLE

*GIFT OF MOVABLE PROPERTY received from other than


Relative:

FULLY TAXABLE IF FMV


Without Consideration
> ₹50,000
*Gift of Movable
Property from other
than relative Taxable =
With Inadequate
Consideration {FMV (-) Consideration}
if it is > ₹50,000

108
**GIFT OF IMMOVABLE PROPERTY RECEIVED FROM
OTHER THAN RELATIVE: -
GIFT OF IMMOVABLE

Taxable Amount =
Received without Consideration Stamp Duty Value (SDV)
PROPERTY

(If it exceeds ₹50,000)

Taxable Amount = SDV (-) Consideration


Not being Residential Unit If it exceeds higher of 2:
a. ₹50,000 or; b. 10% of consideration
Received for Inadequate Consideration.

Instead of 10%, consider 20%


Being Residential Unit
(Conditions - Ref.Note)

Conditions to be satisfied for “Residential” unit:

1. It should be Stock-in-Trade for the seller.


2. The consideration amount should not be exceeding ₹2 crore.
3. The transfer should be between 12.11.2020 to 30.06.2021.
4. This is the first-time purchase of residential unit for the buyer.

VERY IMPORTANT NOTE for CONSIDERING the Stamp Duty Value in case of
IMMOVABLE PROPERTY:

Buyer is purchasing house and the date of Agreement fixing the price and the
actual date of registration i.e. transfer are different.

In such a case, SDV shall be considered as on date of agreement.

Condition for taking SDV as on date of agreement:


Some amount of consideration or whole consideration should be paid
by way of AC Payee Cheque/ Bank Draft, etc. on or before the date of
agreement.

109
Definition of Relative:
(Relevant as any gift received by assessee from a relative is NOT
TAXABLE}

110
Definition of Property:
A Capital Asset of the assessee namely:
- Immovable property being land or building or both,
- Shares and securities,
- Jewellery,
- Archaeological collections
- Drawings,
- Paintings,
- Sculptures,
- Any work of art,
- Bullion.

ILLUSTRATION 1:
Discuss about taxability of receipts without consideration or for
inadequate consideration in following separate cases:
a. Mrs. X received gold Jewellery (Fair Market Value: ₹1,20,000)
from friend on occasion of her marriage.

Solution:
Not Taxable as gift is received on occasion of marriage of
assessee.

b. Mr. T received a plot of land (Stamp Duty Value: ₹5,00,000) from


his wife’s brother.

Solution:
Not Taxable as gift is received by Mr. T from his relative.

c. Mr. A receives ₹10,000, ₹25,000 and ₹20,000 as gift from L, M &


N respectively during F.Y. 2022-23.

111
Solution:
Taxable Amount = ₹55,000, as the aggregate of gifts in cash from
non-relative exceeds ₹50,000 during P.Y. 2022-23, hence entire
such amount is taxable.

d. Mrs. B receives a diamond ring worth ₹1,50,000 from her fiancé.

Solution:
It is fully taxable to the extent of ₹1,50,000 as Fair Market Value
of the movable property exceeds ₹50,000.

e. Mr. P purchased shares (FMV: ₹1,00,000) from his friend for


₹20,000.
Solution:
Taxable Amount = ₹1,00,000 (-) ₹20,000 = ₹80,000.
The difference between the Fair Market Value and property
received from non - relative exceeds ₹50,000 and hence such
difference shall be taxable.

f. Mr. D is gifted Plot of Land at Goa by Mr.S. The Stamp Duty Value
of this land is ₹6,00,000.

Solution:
Here, Taxable amount shall be ₹6,00,000 being the Stamp Duty
Value of land received from non-relative as it exceeds ₹50,000.

g. Mr. E purchased an immovable property from Mr. Q for


₹25,00,000. The Stamp Duty Value of this property is ₹30,00,000.

Solution:
Difference between Stamp Duty Value and Consideration is
₹30,00,000 (-) ₹25,00,000 = ₹5,00,000.
It will be compared with, higher of following:

112
1. ₹50,000
Or;
2. 10% of the Consideration i.e. 10% of ₹25,00,000 = ₹2,50,000

Since, ₹5,00,000 i.e. the difference between SDV and


consideration exceeds ₹2,50,000, such difference shall be
taxable.
Therefore, Taxable amount = ₹5,00,000.

h. What would be your answer in above case, if the consideration is


₹27,50,000 instead of ₹25,00,000?

Solution:
Stamp Duty Value (-) Consideration
= 30,00,000 (-) 27,50,000 = ₹2,50,000.
It will be compared with higher of:-
1. ₹50,000
Or;
2. 10% of Actual Consideration i.e. 10% of ₹27,50,000 =
₹2,75,000.

Since, the difference does not exceed ₹2,75,000, it shall not be


taxable.

i. Mrs. N gets an I-phone (FMV: ₹1,10,000) from her friend on


occasion of her birthday.

Solution:
NOT TAXABLE as Mobile Phone does not fall within ambit of
property.
It should be noted than only the receipt of “Property” without
consideration or for inadequate consideration is taxable.

113
DIVIDEND:

- Fully Taxable under the head Income from Other Sources. It


might be dividend from Companies {Domestic Company or
Foreign Company} of from Co-operative societies.
- Deemed Dividend: -
Section 2(22)
(a) Distribution of accumulated profits, entailing the release of
company's assets.
(b) Distribution of debentures, deposit certificates to
shareholders and bonus shares to preference shareholders
(c) Distribution on liquidation
(d) Distribution on reduction of capital
(e) Advance or Loan by a "closely held company" {i.e. company
in which public is not substantially interested} to its
shareholder.
It is deemed as dividend to the extent of accumulated
profits, if given to –
✓ such shareholder who holds 10% or more in the
shares carrying voting power of the company.
✓ any concern (entity) in which such shareholder has
substantial interest (20% or more in share of
profit/20% or more in shares if that concern is
company)
✓ any person for benefit of such shareholder
Illustration 2:
ABC Pvt. Ltd. advances loan of ₹10,00,000 to Mr. during the Financial
Year 2022-23. The amount in Profit and Loss Account of the company is
₹15,00,000. What shall be treatment in hands of Mr. assuming he holds
10% of shares of the company?
Solution:
Here deemed dividend as per section 2(22)(e) of the Income Tax Act,
1961 shall be ₹10,00,000 being advanced as Loan to a shareholder

114
whose holding in the company is 10% or more of the shares carrying
voting power. Further deemed dividend shall be to the extent of
amount given as loan ie. ₹10,00,000 or; amount of accumulated profits
ie. ₹15,00,000, whichever is lower.

Illustration 3:
LMN Pvt. Ltd. gave Loan of ₹20,00,000 to L&Co., a partnership Firm in
which Mr. L has profit share of 25%. Given that Mr. L’s holding of equity
shares in the Company is 15% of shares carrying voting power. The
accumulated profits of the company are ₹15,00,000. What is the
treatment under Income Tax?
Solution:
₹15,00,000 shall be deemed as dividend in the hands of Mr.L
{₹20,00,000 being the amount of loan to the firm in which such
shareholder has substantial interest OR; ₹15,00,000 being accumulated
profits, whichever is LOWER.}
Exceptions to section 2(22)(e) - Cases where the Loan given to such
shareholder will not be deemed as dividend:
1. In case company is engaged in business of Money Lending/
Banking Company.
2. Advance is given in ordinary course of business i.e. such advance
is for the benefit of the company/ to be spent on behalf of
Company.

TAXABILITY OF CERTAIN INCOMES UNDER THIS HEAD:


1. CASUAL INCOMES:
- Winnings from lotteries, Horse Races, Card Games, Games of
any other sort, Gambling, Betting.
- It is all taxable fully i.e. without any deductions either normal
or of chapter VI-A.

115
- It is taxable at flat (i.e. without considering slabs) 30% u/s
115BB.
- Even TDS is at 30% u/s 194B, 194BB.

2. Consideration received in excess of FMV of shares issued by a


closely held company to be treated as income of such company,
where shares are issued at a premium: {section 56(2) (viib)}

Illustration 4:
Sr. Name of No. of Face Value FMV of Issue Applicability of section 56(2)
No. Co. Shares of Shares shares price of (viib)
shares
1. A (P) Ltd. 10,000 100 120 130 ₹10 per share shall be
taxable as the shares are
issued at premium and
amount higher than FMV.
Taxable Amount per share =
Issue Price of Share (-) FMV
of share i.e. ₹130 (-) ₹120 =
₹10. Total Taxable amount =
₹10,000 * 10 = ₹1,00,000
2. B (P) Ltd. 20,000 100 120 110 Provisions of this section
shall not apply as even
though the shares are issued
at premium, the shares are
issued at lower than FMV.
3 C (P) Ltd. 30,000 100 90 98 Provisions of this section
shall not apply as the shares
are issued at a discount,
even if the issue price is
greater than FMV.
4 D (P) Ltd. 40,000 100 90 110 ₹20 per share shall be
taxable as shares are issued
at premium and also issue
price exceeds FMV. Taxable
amount = ₹20 per share {110
– 90}. Total Taxable = 20 *
40,000 = ₹8,00,000.

116
3. Interest received on compensation/ enhanced compensation is
deemed to be income in the year of receipt and taxable under
the head Income from Other Sources
Deduction - allowed is 50% of such interest received {section 57}

4. Advance received and forfeited on or after 01.04.2014 for sale


of capital asset will be taxed under the head Income from other
sources
Note: In case, it is advance received and forfeited for sale of
capital asset before 01.04.2014, then it will be reduced from Cost
of Acquisition while computing Capital Gains and thus Capital
Gains shall increase to the extent of such advance received and
forfeited.

5. Family Pension:
- It is taxable under the head Income from Other Sources.
- Deduction is allowed to the extent of 1/3rd of such pension
amount received or; ₹15,000 p.a., whichever is lower.

Illustration 5:
Mrs. J received ₹1,20,000 as Family Pension from P Ltd. (former
employer of her deceased husband). What shall be tax implication?
Solution:
Amount of family pension received is taxable under the head Income
from Other Sources.
Deduction is allowed to the extent of lower of following:
a. 1/3rd of family pension amount received
i.e. 1/3rd of ₹1,20,000 = ₹40,000
or;
b. ₹15,000 p.a.
Hence, deduction shall be ₹15,000.
Therefore, Taxable amount of family pension shall be ₹1,20,000 (-)
₹15,000 = ₹5,000.
117
CLUBBING OF INCOME
• It is income of other person included in assessee’s total income.

• Illustration to understand the logic:

Situation for Previous Year 2022-23 is given below: -


Mr.X Mrs.X Master X
(Minor Child)
PGBP: Salary: No Income originally
₹40,00,000 ₹25,00,000
He wants to Interest Income on FD made in his name
do FD of by Mr.X: ₹20,00,000 * 5% = ₹1,00,000.
₹20,00,000 This is the only income that the child has.
which will Therefore, if we seek to tax ₹1,00,000 as
earn IFOS in hands of child, it will be fully
Interest @ exempt as his Total Income does not
5% p.a. exceed basic exemption limit of
He does FD ₹2,50,000.
in name of
his child. Thus effectively, Mr.X can shift his
income legally upto ₹5,00,000 {after
considering Rebate u/s 87A} in hands of
child &; such income will go untaxed.

If taxed in hands of Mr.X, it would have


been taxed @ 30% + SC, if any + H&EC.

In short, Mr.X is trying to do Tax


Avoidance.
BUT;
Income Tax Act has introduced ANTI-TAX
AVOIDANCE measure in this regard &;
that is PROVISIONS OF CLUBBING OF
INCOME.

So, effectively here, Income of minor


child will get clubbed in hands of Mr.X
&; thus avoiding tax would not be
possible for Mr.X.
118
Cases in which the income is to be clubbed: -
INCOME TO BE CONTENTS
CLUBBED
Income transferred When a person transfers the income accruing to an
without transfer of asset without the transfer of asset itself, such income
asset. is to be included in the total income of the transferor,
whether such asset is revocable or irrevocable.
Income arising Such income is to be included in the hands of
from revocable transferor.
transfer of assets. A transfer is deemed to be revocable, if it –
a. Contains any provision for re-transfer of the
whole or any part of income or assets to the
transferor or
b. gives right to re-assume power over the whole
or any part of the income or the asset.
Income arising to Such income arising to spouse is to be included in the
spouse by way of total income of the individual.
remuneration
from a concern in However, if remuneration received is attributable to
which the the application of technical or professional knowledge
individual has and experience of spouse, then, such income is not to
substantial be clubbed.
interest.
Income arising to Income arising from an asset (other than House
spouse from assets Property) transferred otherwise than for adequate
transferred consideration or not in connection with an agreement
without adequate to live apart, from one spouse to another shall be
consideration. included in total income of the transferor.

This provision does not apply in case of house property


as the transferor-spouse would be deemed owner as
per section 27.
Income arising to Income arising from an asset transferred otherwise
son’s wife from an than for adequate consideration, by an individual to
asset transferred his or her son’s wife shall be included in total income
without adequate of the transferor.
consideration.
Income arising All income arising to any person or association of
from transfer of persons from assets transferred without adequate
assets for the consideration is includible in the income of transferor,
benefit of spouse to the extent such income is used by the transferee for
or son’s wife. the immediate or deferred benefit of the transferor’s
spouse or son’s wife.

119
Income of minor All income arising to a minor child (including a minor
child. married daughter) shall be included in the total
income of his or her parent.
The income of the minor child shall be included with
the income of that parent whose total income, before
including minor’s income is higher.
Where the marriage of parent does not subsist, the
income of minor will be includible in the income of
that parent who maintains the minor child in the
relevant Previous Year.
The parent in whose total income, the income of
minor child or children are included, shall be entitled
to exemption of such income subject to maximum of
₹1,500 per child under section 10(32).
The following income of minor child shall, however,
not be clubbed in the hands of his or her parent: -
a. Income from manual work done by him or
activity involving application of minor’s skill,
talent or specialized knowledge and experience
and;
b. Income of minor child suffering from any
disability specified in Section 80U.
Note: In case of transfer of House property to minor
without consideration or for inadequate
consideration, these provisions of clubbing
shall not apply but provisions of deemed
ownership under section 27 shall apply and
transferor parent shall be deemed owner of the
house and accordingly income will be taxed in
hands of parent. Therefore, in such case,
exemption of upto ₹1,500 u/s 10(32) shall not
be applicable.
However, the provisions of deemed ownership
are not applicable to house transferred to
minor married daughter &; therefore, in that
case, provisions of clubbing shall continue to
apply.
Conversion of self Where, an individual, who is a member of the HUF,
acquired property converts his individual property into property of the
into the property HUF of which he is a member, directly or indirectly, to
of HUF. the family otherwise than for adequate consideration,
the income from such property shall continue to be
included in the total income of the individual.

120
Illustration 1:

Mr. Sharma has children consisting 2 daughters and 2 sons. The annual
income of 2 daughters were ₹9,000 and ₹4,500 and of sons were
₹6,200 and ₹4,300 respectively. The daughter who has income of
₹4,500 is suffering from disability specified under section 80U.
Compute the amount of income earned by minor children to be
clubbed in hands of Mr. Sharma.

Solution:

Computation of Income of minor children to be clubbed in hands of


Mr. Sharma for the Assessment Year 2023-24
(Relevant to Previous Year 2022-23)

PARTICULARS Son 1 Son 2 Daughter 1 Total


Annual Income 6,200 4,300 9,000 19,500
Less: Exemption u/s
10(32) (1,500) (1,500) (1,500) (4,500)
Income to be clubbed 4,700 2,800 7,500 15,000

Note:
Income of daughter whose income is ₹4,500 and suffering from
disability specified under section 80U shall not be clubbed in hands of
Mr. Sharma.

Illustration 2:

Mr. Vaibhav started a Proprietary business on 01.04.2021 with a


capital of ₹5,00,000. He incurred a loss of ₹2,00,000 during the year
2021-22 of ₹2,00,000. To overcome the financial position, his wife
Mrs. Vaishali, a software engineer gave a gift of ₹5,00,000 on
01.04.2022, which was immediately invested in the business by Mr.
Vaibhav. He earned a profit of ₹4,00,000 during the year 2022-23.
Compute the amount to be clubbed in hands of Mrs. Vaishali for the
Assessment Year 2023-24.

121
If Mrs. Vaishali gave the said amount as Loan, what would be the
amount to be clubbed?
Solution:

- The amount of profit to the extent of gifted amount to the total


capital on the first day of the previous year must be clubbed in the
hands of assessee.
- Income accruing or arising from transferred assets will alone be
clubbed.

Relevant computations:

Particulars Amount (₹)


a. Capital as on 01.04.2021 5,00,000
Less: loss for F.Y. 2021- (2,00,000)
22
Therefore, Total Capital
as on 31.03.2022 3,00,000
b. Add: Amount of gift
invested as capital in
business 5,00,000
c. Therefore, Capital as on
01.04.2022 8,00,000
d. Profit of P.Y. 2021-22 4,00,000
e. Amount to be clubbed ₹4,00,000 * 5,00,000/8,00,000
= ₹2,50,000

Notes:

1. Gift received from spouse is NOT TAXABLE being received from


relative.
2. If the amount is received as Loan, then clubbing provisions
would not be attracted.
Illustration 3:
Mr. A is an employee of Larsen Ltd. and has substantial interest in
the company. His salary is ₹25,000 per month. Mrs. A also is
working in that company at salary of ₹10,000 per month without
any professional qualification (Ignore Standard Deduction).
122
Mr. A also receives ₹30,000 as income from securities. Mrs. A owns
a House Property which she has let out. Rent received from such
house property is ₹12,000 per month.
Mr. A and Mrs. A have 3 minor children – two twin daughters and
one son. Income of twin daughters is ₹2,000 p.a. and that of his son
is ₹1,200 p.a. Compute the income of Mr. A and Mrs. A.

Solution:
Computation of Total Income of Mr. A and Mrs. A for the
Assessment Year 2023-24
(Relevant to Previous Year 2022-23):
PARTICULARS Mr. A (₹) Mrs. A (₹)
a. Income from Salaries 3,00,000
(₹25,000 *
12)
b. Add: Salary paid to Mrs. A,
clubbed in hands of Mr. A {As Mr.
A has substantial interest in the
company and Mrs. A does not
have any professional 1,20,000
qualification.} (₹10,000 *
12)
c. Income from House Property - 1,00,800
d. Income from Other Sources
(Interest on Securities) 30,000 -
e. Therefore, Total Income {Before
including Minor’s Income}
4,50,000 1,00,800
f. Income of twin daughters
2,000
Less: Exempt upto ₹1,500
each (2,000) -
Income of minor son 1,200
Less: Exempt (1,200) - Nil
g. Therefore, Total Income 4,50,000 1,00,800

123
Note:
1. Total Income of minor child shall be clubbed in hands of parent
whose total income is greater before such clubbing and hence
is seek to be clubbed in hands of Mr.A.

2. Computation of Income from House Property for Mrs. A


PARTICULARS Amount (₹)
a. Gross Annual Value {₹12,000 * 12} 1,44,000
b. Less: Municipal Taxes -
c. Therefore, Net Annual Value 1,44,000
d. Less: Deductions under section 24
(a) Standard Deduction @ 30% of NAV (43,200)
(b) Interest on Borrowed Capital -
e. Therefore, Income from House 1,00,800
Property

CROSS TRANSFERS – Discussed through illustration:


Mr. Vasudevan gifted a sum of ₹6,00,000 to his brother’s wife on 14.06.2022.
On 12.7.2022, his brother gifted a sum of ₹5,00,000 to Mr. Vasudevan’s wife.
The gifted amounts were invested as Fixed Deposits in Banks by Mrs.
Vasudevan and wife of Mr. Vasudevan's brother on 01.08.2022 @ 9%
interest.
Examine the consequences of above under the provisions of Income Tax Act
in hands of Mr. Vasudevan and his brother.
Solution:
- These are CROSS TRANSFERS.
- Income from assets transferred would be assessed in hands of the
deemed transferor because the transfers are so intimately
connected to form part of a single transaction and each transfer
constitutes consideration for the other by being mutual or
otherwise.
- If two transactions are inter-connected and are part of the same
transaction in such a way that it can be said that the circuitous
method was adopted as a device to evade tax, the implication of

124
clubbing provisions would be attracted. (CIT v/s Keshavji Morarji
– SC)
- Accordingly, the interest income arising to Mrs. Vasudevan in form
of interest on Fixed Deposits would be included in Total Income of
Mr. Vasudevan and interest income arising in hands of his brother’s
wife would be taxable in hands of Mr. Vasudevan’s brother as per
section 64, to the extent of amount of cross transfers i.e.
₹5,00,000.
- This is because both Mr. Vasudevan and his brother are the indirect
transferors of the income to their respective spouses with an
intention to reduce their burden of taxation.
- However, the interest income earned by his spouse on FD of ₹5
lakhs alone would be included in hands of Mr. Vasudevan’s brother
and not the interest income on the entire FD of ₹6 lakhs as the cross
transfer is only to the extent of ₹5 lakhs.

125
SET OFF AND CARRY FORWARD OF
LOSSES

INTER SOURCE AND INTER HEAD SET OFF OF LOSSES:


PROVISION EXCEPTIONS
Inter- Source set off of losses under 1. Loss from speculation business can be set off
the same head of Income: only against profits from another speculation
Any loss in respect of one source business.
shall be set off against income from 2. Loss from specified business under section
any other source under the same 35AD can be set off only against profits from
head of income. any other specified business.
Example: 3. Long Term Capital Loss can be set off only
- Loss from textile business against Long Term Capital Gains.
can be set off against profit 4. Loss from activity of owning and maintaining
from printing business. race horses can be set off only against
- Loss from House A can be set income from activity of owning and
off against income from maintaining race horses.
House B.
- Short Term Capital Loss
(STCL) can be set off against
both Short Term Capital
Gains as well as Long Term
Capital Gains.
Inter-head adjustment: 1. Loss under the head “PGBP" cannot be set
Loss from one head of income can off against salaries.
be set off against income under any 2. Loss under the head Capital Gains cannot
other head. be set off against any other head of
Example: income.
Business loss can be set off against 3. Speculation losses, losses from specified
income from house property. business u/s 35AD and loss from activity
of owning and maintaining race horses
cannot be set off against income from any
other head.
4. Loss from House Property can be set off
against income under any other head
only to the extent of ₹2,00,000. The
remaining loss can be carried forward for
set off against income from house
property of the succeeding year(s).

126
Losses which cannot be set off or carried forward:
- Loss from gambling, betting, card games, etc.
- Loss from exempt source {E.g.: - Share of loss of partnership firm
cannot be set off against any other business income.}

PROVISIONS OF CARRY FORWARD OF LOSSES:


Nature of loss Income against which Maximum period for
to be carried the brought forward carry forward of
forward loss can be set off losses {from the end
of relevant
Assessment Year}
Unabsorbed Income under any head Indefinite period
Depreciation other than salaries.
Loss from Income from House 8 Assessment Years
House Property Property
Business Loss PGBP 8 Assessment Years
Loss from Income from any 4 Assessment Years
Speculation speculation business.
Business
Loss from Profit from any Indefinite period
specified specified business
business under
section 35AD
Long Term Long Term Capital 8 Assessment Years
Capital Loss Gains
Short Term Short Term/ Long Term 8 Assessment Years
Capital Loss Capital Gains
Loss from Income from activity of 4 Assessment Years
activity of owning and
owning and maintaining race
maintaining horses.
race horses

127
Order of set-off of losses:

1. Current Year depreciation/ Current year capital expenditure on


scientific research and current year expenditure on family
planning, to the extent allowed.
2. Brought forward loss from business/ profession.
3. Unabsorbed depreciation.
4. Unabsorbed capital expenditure on scientific research.
5. Unabsorbed expenditure on family planning.

Carry Forward of losses possible only if ITR is filed within due date
of section 139 (1): -
As per section 80, filing of loss return under section 139(3) within the
due date specified under section 139(1) is mandatory for carry
forward of above losses except loss from house property and
unabsorbed depreciation.
ILLUSTRATION 1:

Mr.X, a resident individual submits the following information,


relevant to the previous year ending March 31, 2023:
PARTICULARS Amount (₹)
a. Income from Salary (Computed) 2,22,000
b. Income from House Property
- House in Delhi 22,000
- House in Chennai (2,60,000)
- House in Mumbai (Self-Occupied) (20,000)
c. PGBP
- Textile Business 18,000
- Cosmetic business (22,000)
- Speculative Business 1 (74,000)
- Speculative Business 2 46,000
d. Capital Gains
- Short Term Capital Loss from sale of property (16,000)
- Long Term Capital Gains from sale of property 15,400
e. Income from Other Sources (Computed)
128
- Income from Betting 34,000
- Income from Card Games 46,000
- Loss on maintenance of race horses (14,600)

Determine the Gross Total Income of Mr.X for the assessment year
2023-24 and the losses to be carried forward assuming that he does
not opt to be taxed under section 115BAC.

SOLUTION:

Computation of Gross Total Income of Mr.X for the Assessment Year


2023-24 (Relevant to Previous Year 2022-23):
PARTICULARS Amount Amount (₹) Amount
(₹) (₹)
A. Income from 2,22,000
Salaries
B. Income from
House Property
- House in Delhi 22,000
- Loss from House in
Chennai (2,60,000)
- Loss from House in
Mumbai (20,000)
Maximum
deductible is (2,58,000) (2,00,000)
₹2,00,000
C. Profits and Gains
from Business or
Profession:
- Ordinary Business
Textile Business 18,000
Cosmetic Business (22,000) 4,000
- Speculative
Business
First (Loss) (74,000)
Second 46,000 (28,000) Nil
D. Capital Gains
129
- Short Term Capital
Loss (16,000)
- Long Term Capital
Gains 15,400 (600)
E. Income from Other
Sources
Income from 34,000
Betting
Income from Card
games 46,000 80,000
F. Gross Total Income 1,02,000

Following losses will be carried forward:

PARTICULARS PERIOD FOR CARRY Amount


FORWARD OF LOSS (₹)
House Property Loss Next 8 A.Y. 58,000
Normal Business Loss Next 8 A.Y. 4,000
Speculative Business Next 4 A.Y. 28,000
Loss
Short Term Capital Loss Next 8 A.Y. 600
Loss on maintenance of
race horses Next 4 A.Y. 14,600

130
DEDUCTIONS UNDER CHAPTER VI-A

Based on Investments, • 80C, 80CCC, 80CCD, 80D, 80DD,


80DDB, 80E, 80EE, 80EEA, 80EEB,
payments. 80G, 80GG, 80GGA, 80GGB, 80GGC.

• 80TTA, 80TTB, 80QQB, 80RRB.


Based on Incomes.

• 80U, 80JJAA.
Others

SECTION 80C:
DEDUCTION IN RESPECT OF INVESTMENTS/ CONTRIBUTIONS:
- Eligible assessee: Individual/ HUF.
- Investments/ Contributions eligible for deduction: -
a. Premium paid in respect of Life Insurance Policy:
It is allowed when paid for individual assessee himself, his
spouse or his children.
Inner Ceiling limits w.r.t. Life Insurance Premium:
Policy Issue date Inner Limit
a. Upto 31.03.2012 20% of sum assured
b. On or after 01.04.2012 till 10% of sum assured
31.03.2013
c. On or after 01.04.2013

131
If it is on life of person 15% of sum assured
suffering with disability
On any other person 10% of sum assured
b. Contribution to SPF, RPF & PPF
c. Deposit in Sukanya Samriddhi Account
d. Subscription to National Savings Certificate (NSC)
e. 5 Years Fixed Deposit in Bank/ Post Office
f. Payment of Tuition Fees for education within India for upto 2
children.
g. Repayment of Housing Loan {Principal element}, Stamp Duty,
Registration Fees &; other expenses.

Illustration 1
Compute the eligible deduction under section 80C for the Assessment
Year 2023-24 in respect of Life Insurance premium paid by Mr. Ganesh
during the Previous Year 2022-23, the details of which are given
hereunder: -
Sr. Date of Issue Person Insured Actual Insurance Premium
No. of Policy Capital sum paid during P.Y.
assured (₹) 2022-23 (₹)
1. 30.03.2012 Self 6,00,000 48,000
2. 01.05.2017 Spouse 1,50,000 20,000
3. 01.06.2020 Handicapped
Son (section 80U
disability) 4,00,000 80,000

Solution:
Sr. Date of Person Actual Insurance Deduction Restriction
No. Issue of Insured Capital Premium u/s 80C to % of
Policy sum paid for the sum
assured during A.Y. 2023- assured
P.Y. 24
2022-23
(₹)
132
1 30.03.2012 Self 6,00,000 48,000 48,000 20%
2 01.05.2017 Spouse 1,50,000 20,000 15,000 10%
3 01.06.2020 Handicapped
son 4,00,000 80,000 60,000 15%

SECTION 80CCC
DEDUCTION IN RESPECT OF CONTRIBUTION TO CERTAIN
PENSION FUNDS

SECTION 80CCD:
DEDUCTION IN RESPECT OF CONTRIBUTION TO PENSION
SCHEME NOTIFIED BY CENTRAL GOVERNMENT i.e. NEW
PENSION SCHEME:
- Quantum of deduction:
a. For employee: 10% of Salary
b. For employee employed by Central Government before
01.04.2004 or any other individual: 20% of Gross Total
Income
- ADDITIONAL LIMIT of deduction w.r.t. contribution in this
pension scheme u/s 80CCD(1B): - ₹50,000 {which is over and
above the overall ceiling limit of ₹1,50,000}
- Amount contributed by employer towards NPS for employee is
allowed as deduction u/s 80CCD(2).{It will be first included as
perquisite in Net Taxable Salary and thus Gross Total Income &;
then will be allowed as deduction under this section}
Limit:
a. If Contribution is by Central Government as employer: 14% of
salary*
b. If contribution by any other employer:- 10% of salary*

Salary* = Basic + Dearness Allowance {Terms of Employment/


Retirement Benefits}
133
Illustration 2:

The basic salary of Mr.A is ₹1,00,000 per month. He is entitled to


dearness allowance, which is 40% of basic pay. 50% of the Dearness
Allowance forms part of pay for retirement benefits. Both Mr. A and his
employer contribute 15% of basic salary to the pension scheme
referred to in section 80CCD. Explain the tax treatment in respect of
such contribution in hands of Mr.A.

Solution:

Salary for the purpose of deduction under section 80CCD for Mr. A
would be –
PARTICULARS Amount (₹)
a. Basic Salary {₹1,00,000 per month * 12} 12,00,000
b. Dearness Allowance {40% of ₹12,00,000 * 50%} 2,40,000
c. Therefore, Salary for the purpose of section 14,40,000
80CCD
d. Deduction under section 80CCD(1) is restricted
to 10% of ₹14,40,000 (as against actual
contribution of ₹1,80,000, being 15% of basic
salary of ₹12,00,000) 1,44,000
e. Deduction as per section 80CCD (1B), upto an
additional limit of ₹50,000. Therefore,
₹1,80,000 (-) ₹1,44,,000 36,000

Alternatively:

First claim entire ₹50,000 u/s 80CCD(1B), the limit of which is


exclusively for this purpose. Then, under section 80CCD(1), claim
₹1,80,000 (-) ₹50,000 = ₹1,30,000. This is more beneficial for the
assessee as under section 80CCE, the overall limit of deduction u/s 80C,
80CCC & 80CCD(1) is ₹1,50,000. If only ₹1,30,000 is claimed u/s
80CCD(1), that leaves scope of ₹20,000 for other deductions to
accommodate in the same. Thus, giving overall higher deduction to the
assessee.
134
Employer’s Contribution to NPS for employee’s benefit:

Such amount contributed is ₹12,00,000 * 15% = ₹1,80,000.

It will first be included as PERQUISITE in the taxable salary and


consequently in Gross Total Income.
Then, deduction will be allowed to the extent of 10% of salary.
Here, Salary = 10% of ₹14,40,000 = ₹1,44,000.

Hence, deduction u/s 80CCD(2) shall be ₹1,44,000. {Being lower of


₹1,80,000 or ₹1,44,000}.

Section 80D:
Mediclaim/ Health Insurance Premium/ Contribution to
Central Government Health Scheme/ Amount spent
towards Preventive Health Check-up:
- Eligible assessee: Individual/ HUF.
- Monetary ceiling Limit:
PARTICULARS Is not a Is a senior
Senior citizen
Citizen
a. Paid for assessee himself + his
spouse + dependent children ₹25,000 ₹50,000
b. ADDITIONAL LIMIT – Paid for
parents {May or may not be
dependent} ₹25,000 ₹50,000

Notes:

1. In case of Preventive Health Check-up, the limit is ₹5,000


{Amount for all together}. Further, it is an inner ceiling limit i.e.
not over and above ₹25,000/ ₹50,000.

135
2. Mediclaim/ Health insurance premium is allowed as deduction
only if paid in mode other than cash.
3. In case, Senior Citizen is not covered by medical insurance,
then deduction allowed u/s 800D shall be with respect to
actual amount spent towards medical expenditure or;
₹50,000, whichever is less.

Illustration 3:
Mr. A, age 40 years, paid medical insurance premium of ₹20,000 during
the Previous Year 2022-23 to insure his health as well as health of his
spouse. He also paid medical insurance premium of ₹47,000 during the
year to insure health of his father, aged 63 years, who is not dependent
on him. He contributed ₹3,600 to the Central Government Health
Scheme during the year. He has incurred ₹3,000 in cash on preventive
health check-up of himself and his spouse and ₹4,000 by cheque on
preventive health check-up of his father.
Compute the deduction allowable under section 80D for the
Assessment Year 2023-24.
Solution:
Deduction allowable under section 80D for the Assessment Year 2023-
24 (Relevant to Previous Year 2022-23)
PARTICULARS Amount (₹)
a. For Himself + Spouse + Dependent Children
(Paid by way of A/C Payee cheque in case of
health insurance premium while in any mode
in case of preventive health check up
- Mediclaim 20,000
- CGHS 3,600
- Preventive Health Check up {₹3,000 or the
remaining limit of ₹1,400 within the limit of
₹25,000, whichever is lower} 1,400
Total (a) 25,000
b. For his parents
136
- Mediclaim for father who is senior citizen 47,000
- Preventive Health Check up {₹4,000 or;
unexhausted limit of ₹3,000 from the overall
limit of ₹50,000, whichever is lower.} 3,000
Total (b) 50,000
Therefore, Total Deduction u/s 80D {A+B} 75,000

SECTION 80DD:
Deduction with respect to maintenance of disabled
dependent:
- Eligible Assessee: Individual/HUF.
- Deduction is NOT DEPENDEDNT on amount spent.
- Flat deduction of ₹75,000 p.a. in case of dependent being
disabled not severely. In case of severely disabled, deduction is
₹1,25,000.
- Severe disability = 80% or more of the disability as certified by
medical practioner.
- Dependent means:
For Spouse, Children, parents, brother or sister of
Individual the individual who is mainly or wholly dependent
on such individual and not claimed deduction
under section 80U in the computation of his
income.
For HUF A member of the HUF, wholly or mainly
dependent on such HUF and not claimed
deduction u/s 80U in the computation of his
income.

Illustration 4:

Mr. X is a resident individual. He deposits a sum of ₹50,000 with Life


Insurance Corporation every year for maintenance of his disabled
grandfather who is wholly dependent upon him.

137
The disability is one which comes under The Persons with Disabilities
(Equal Opportunities, Protection of Rights and Full Participation) Act,
1955. A copy of the certificate from the medical authority is submitted.
Compute the amount of deduction available under section 80DD for the
Assessment Year 2023-24.

Solution:
Since the amount deposited by Mr.X was for his grandfather, he will not
be allowed any deduction under section 80DD. The deduction is
available if the individual assessee incurs any expense for a
“dependent” disable person. Grandfather does not come within the
meaning of “dependent” defined under section 80DD.

Illustration 5:

What will be the deduction if Mr.X had made this deposit for his
dependent father?

Solution:
Since the expenditure was incurred for a dependent disabled person,
Mr. X will be entitled to claim a deduction of ₹75,000 under section
80DD, irrespective of the amount deposited.
In case his father has severe disability, the deduction would be
₹1,25,000.

SECTION 80DDB:
Deduction in respect of medical treatment w.r.t. specified
diseases:
- Eligible assessee: Resident individual/ HUF.
- Medical expenditure should be incurred on himself or dependent.
- Ceiling Limits for deduction:
a. For treatment of other than senior citizen: ₹40,000
b. For treatment of senior citizens: ₹1,00,000
- Therefore, amount allowed as deduction shall be amount spent
or the ceiling limits, whichever is lower.
138
SECTION 80E:
INTEREST ON HIGHER EDUCATION LOAN:
- Eligible Assessee: Individual
- Loan from Approved Charitable institution or any financial
institution.
- For HIGHER EDUCATION of the assessee himself or spouse or
children of individual or the student for whom individual is a legal
guardian.
- Higher Education = Any course pursued after Standard XII.
- Education may be pursued outside India.
- NO MONETARY CEILING LIMIT.
- Deduction is allowed for the Previous Year in which the loan
repayment starts + subsequent 7 P.Ys i.e. in total 8 P.Ys starting
with the P.Y. in which the loan repayment has started.

SECTION 80EE:
INTEREST ON LOAN BORROWED FOR ACQUISITION OF
HOUSE PROPERTY BY AN INDIVIDUAL:
- Conditions:
1.Value of House should be less than or equal to ₹50 lakhs.
2.Loan should be sanctioned during P.Y. 2016-17.
3.Loan sanctioned is less than or equal to ₹35 lakhs.
4.Assessee should not own any residential house as on date of
sanction of loan.
- Ceiling Limit for deduction: ₹50,000 p.a.
- Loan necessarily should be from a financial institution.

139
SECTION 80EEA:
Interest on Loan taken for residential house property:
- Eligible assessee: Individual
- Conditions:
a. Stamp Duty Value of the House should be less than or equal to
₹45 lakhs.
b. Loan should be sanctioned between 1st April, 2019 to 31st
March, 2022.
c. He should not be owner of any residential house as on date of
sanction of loan.
- Ceiling Limit for deduction: ₹1,50,000
- Loan should necessarily be from Financial Institution.

SECTION 80EEB:
INTEREST ON LOAN FOR PURCHASE OF ELECTRIC VEHICLE:
- Eligible Assessee: Individual.
- Loan should be from Financial Institution {Bank/ Specified NBFCs}
- Loan should be sanctioned between 01.04.2019 – 31.03.2023.
- Quantum of deduction: Amount of interest paid or; ₹1,50,000,
whichever is lower.

140
SECTION 80G
DONATIONS:
- Eligible assessee: ANY assessee.
- Following are the four categories of deduction u/s 80G:
CATEGORY I CATEGORY II CATEGORY III CATEGORY IV
Donation qualifying Donation Donation qualifying Donation
for 100% deduction qualifying for for 100% Qualifying for 50%
without any qualifying 50% deduction deduction, subject deduction,
limit without any to qualifying limit. subject to
qualifying limit. qualifying limit.
In this 24 funds are Following 4 Following Donation to any
covered. Important funds are Donations qualify institution in India
ones: covered here: for deduction: - established for
National Defence - The a. To the Charitable
Fund, National Jawaharlal Government purposes.
Children’s Fund, Prime Nehru or any
Minister’s National Memorial approved
Relief Fund, PM Cares Fund. local
Fund. - Prime authority,
Minister’s institution or
Drought association
Relief Fund for
- Indira promotion of
Gandhi family
Memorial planning.
Trust b. Sum paid by a
- Rajiv company as a
Gandhi donation to
Foundation the Indian
Olympic
Association
or any other
association
established in
India as may
be notified by
the Govt. for
141
the
development
of
infrastructure
for sports or
games or the
sponsorship
of sports and
games in
India.

Meaning for Qualifying amount for section 80G: -


10% of “Adjusted Gross Total Income”
Meaning of Adjusted Gross Total Income: -
Gross Total Income
(-) Deductions u/s 80C – 80U {Except 80G}
(-) STCG u/s 111A / LTCG
NOTE: Donation in excess of ₹2,000 shall be allowed only if paid in
mode other than cash.

Illustration 6:
Mr. Shiva, aged 58 years, has Gross Total Income of ₹7,75,000
comprising of income from salary and house property. He has made the
following payments and investments:
a. Premium paid to insure the life of his major daughter {Policy
taken on 01.04.2018} (Sum assured ₹1,80,000}: ₹20,000.
b. Medical Insurance Premium for self: ₹12,000, for spouse:
₹14,000.
c. Donation to Public Charitable Trust: ₹50,000 by way of cheque.
d. LIC pension fund: ₹60,000
e. Donation to National Children’s Fund: ₹25,000 by way of cheque.

142
f. Donation to Jawaharlal Nehru Memorial Fund: ₹25,000 by way of
cheque.
g. Donation to approved institution for promotion of family
planning: ₹40,000 by way of cheque.
h. Deposit in PPF: ₹1,00,000.
Compute the Total Income of Mr. Shiva for the Assessment Year
2023-24.
Solution:
Computation of Total Income of Mr. Shiva for the Assessment
Year 2023-24 (Relevant to Previous Year 2022-23):
PARTICULARS Amount (₹) Amount (₹)
a. Gross Total Income 7,75,000
b. Less: Deduction u/s 80C
Deposit in PPF 1,00,000
Life Insurance Premium
{Maximum 10% of the
assured value ₹1,80,000, as
the policy is taken after 18,000
31.03.2012}
1,18,000
Deduction u/s 80CCC in
respect of LIC Pension Fund 60,000
1,78,000
As per section 80CCE,
deduction u/s 80C, 80CCC &
80CCD is restricted to 1,50,000
Deduction u/s 80D
Medical Insurance Premium
in respect of self and spouse 26,000
Restricted to 25,000
Deduction u/s 80G (Note) 87,500
c. Total Income 5,12,500

143
Working Note: Computation of Deduction u/s 80G:
Sr. Particulars of Amount % of Donation
No Donation Donated deduction u/s 80G (₹)
(₹)
1. National
Children’s Fund 25,000 100% 25,000
2. Jawaharlal Nehru
Memorial Fund 25,000 50% 12,500
3. Approved 100%
Institution for subject to
promotion of qualifying
family planning 40,000 limit 40,000
4. Public Charitable 50%
Trust 50,000 subject to 10,000
qualifying
limit (Note)
87,500

Note:
Adjusted Total Income = Gross Total Income (-) Deductions u/s
80C – 80U except 80G i.e. ₹6,00,000 {₹7,75,000 (-) ₹1,75,000}
10% of Adjusted Total Income = ₹6,00,000 * 10% = ₹60,000
Firstly, donation of ₹40,000 to approved institution for family
planning qualifying for 100% deduction subject to qualifying limit
has to be adjusted against this amount.
Thereafter, donation to Public Charitable Trust qualifying for 50%
deduction, subject to qualifying limit is adjusted.
Hence, the contribution of ₹50,000 to Public Charitable Trust is
restricted to ₹20,000 {₹60,000 (-) ₹40,000}, 50% of which would
be deduction u/s 80G.
Therefore, deduction u/s 80G in respect of donation to Public
Charitable Trust would be ₹10,000, which is 50% of ₹20,000.

144
SECTION 80GG:
RENT PAID:
- Quantum of deduction:
Least of the following:
a. Rent paid (-) 10% of total income {Before allowing deduction
u/s 80GG}
b. 25% of Total Income
c. ₹5,000 per month.
- To claim deduction u/s 80GG, assessee should not be in receipt of
HRA from his employer which is eligible for exemption u/s
10(13A). Further, he or his spouse or minor child should not be
owning house in that city where he ordinarily carries out business
or is employed.
Illustration 7:
Mr. Ganesh, a businessman, whose Total Income (before allowing
deduction under section 80GG) for the Assessment Year 2023-24 is
₹4,60,000, paid house rent of ₹12,000 p.m. in respect of residential
accommodation occupied by him at Mumbai. Compute the deduction
allowable to him under section 80GG for the Assessment Year 2023-24.
Solution:
The deduction u/s 80GG will be computed as follows:
a. Actual Rent paid (-) 10% of Total Income
i.e. ₹1,44,000 (-) ₹46,000
= ₹98,000

b. 25% of Total Income


= 25% of ₹4,60,000
= 1,15,000

c. Amount computed at ₹5,000 per month


= ₹5,000 * 12
= ₹60,000
145
Therefore, Deduction allowable under section 80GG shall be
₹60,000, being least of the above.

SECTION 80GGA:
DONATIONS FOR SCIENTIFIC RESEARCH AND RURAL
DEVELOPMENT:
- Eligible assessee: Any assessee not having income chargeable
under the head Profits and Gains from business or profession. {As
those assessees having PGBP can claim deduction u/s 35 from Net
Profit}
- Making Donation for SCIENTIFIC RESEARCH and RURAL
DEVELOPMENT is allowable as deduction.
- Donation in excess of ₹2,000 shall be allowed only if paid in mode
other than cash.

SECTION 80GGB:
DONATION/ CONTRIBUTION TO POLITICAL PARTIES BY
DOMESTIC COMPANIES:
- No deduction is allowed w.r.t. any sum contributed by way of
cash.
- Contribution would also include expenditure incurred on
advertisement in journal, etc. published by a political party.

SECTION 80GGC:
DONATION/ CONTRIBUTION TO POLITICAL PARTIES BY
ANY PERSON:
- No deduction is allowed w.r.t. any sum contributed by way of
cash.

146
- The deduction is not allowed for any Artificial Juridical Person
(AJP) or Local Authority, which is wholly or partly funded by the
Government.
COMMON NOTE FOR Section 80GGB & Section 80GGC:
Section 37 DISALLOWS expenditure on advertisement w.r.t. amount
paid for advertising in journals of Political parties. Therefore, such
expenses are disallowed while computing PGBP/ Gross Total Income.
However, it is allowed u/s 80GGB/80GGC, as the case may be from
Gross Total Income.

SECTION 80JJAA:
DEDUCTION IN RESPECT OF EMPLOYMENT OF NEW
EMPLOYEES:
- This deduction is allowed to ANY assessee engaged in business to
which TAX AUDIT IS APPLICABLE.
- It should have employed NEW EMPLOYEES.
- Deduction = 30% of new employee cost, allowed for the Previous
Year in which such employees are employed and in next 2 years.
- Furnishing of details in REPORT BY a CA by 30th September of the
Assessment Year.
- Who is NOT an additional employee?
a. Employee whose total emoluments are more than ₹25,000
p.m.
b. Employee who does not participate in Recognised Provident
Fund (RPF).
c. Employees employed for less than 240 days during a P.Y.
{150 days in case of employee employed in Apparel, footwear
or leather products manufacturing}
Illustration 8:
Mr. A has commenced the business of manufacture of computers on
01.04.2022. He employed 350 new employees during the Previous Year
2022-23, the details of whom are as follows: -
147
Sr. No. of Date of Regular/ Total Monthly
No. employees employment Casual emolument per
employee (₹)
1. 75 1.4.2022 Regular 24,000
2. 125 1.5.2022 Regular 26,000
3. 50 1.8.2022 Casual 24,500
4. 100 1.9.2022 Regular 24,000

The regular employees participate in Recognised Provident Fund while


the casual employees do not.
Compute the deduction, if any, available to Mr. A for Assessment Year
2023-24, if profits and gains derived from manufacture of computers
that year is ₹75 lakhs and his total turnover is ₹10.16 crores.
What would be your answer if Mr.A has commenced the business of
manufacture of footwear on 01.4.2022?
Solution:
Mr. A is eligible for deduction under section 80JJAA since he is subject
to tax audit under section 44AB for Assessment Year 2023-24 and has
employed additional employees during the Previous Year 2022-23.
1. If Mr. A is engaged in the business of manufacture of computers:

Additional Employee Cost = ₹24,000 * 12 * 75 = ₹2,16,00,000


(Refer Working Note)

Deduction u/s 80JJAA = 30% of ₹2,16,00,000 = ₹64,80,000.

Note: Deduction of ₹64,80,000 each will also be allowed in


subsequent 2 Assessment Years.

Only employees in Sr.1 are treated as Additional Employees.


Others are not treated as in Sr.2, the emoluments per month are
exceeding ₹25,000. In Sr. No. 3, the employees are not
participating in Provident Fund. Sr. No.4 employees are employed

148
for less than 240 days in Previous Year 2022-23. {In P.Y. 2023-24,
they would be treated as additional employees, if in that year
they continue to be employees for minimum 240 days.
Accordingly, in A.Y. 2024-25, deduction of 30% of such employee
cost will be allowable}.

2. If Mr. A in engaged in the business of manufacture of footwear:

In this case, he would be entitled to deduction under section


80JJAA in respect of employee cost of regular employees
employed on 01.09.2022 i.e. Sr. No.4 as they have been employed
for more than 150 days in the previous year 2022-23.

Additional Employee Cost = ₹2,16,00,000 + (₹24,000 * 100 * 7) =


₹3,84,00,000

Deduction u/s 80JJAA = ₹3,84,00,000 * 30% = ₹1,15,20,000.

Note: In the succeeding 2 A.Y.s i.e. 2024-25 & 2025-26, also


deduction of ₹1,15,20,000 each would be allowable.

SECTION 80QQB
DEDUCTION IN RESPECT OF ROYALTY INCOME OF
AUTHORS OF CERTAIN BOOKS OTHER THAN TEXT BOOKS:
- Eligible assessee: Resident Individual.
- Ceiling Limit: ₹3,00,000
- Inner Limit:- In case Royalty is received as certain percentage of
sale value of books, then royalty allowed as deduction cannot
exceed 15% of value of books sold.
- Where assessee earns income from outside India, he should bring
such income in convertible foreign exchange within a period of 6
months from the end of the previous year in which such income
is earned.

149
SECTION 80RRB:
DEDUCTION WITH RESPECT TO ROYALTY ON PATENTS:
- Eligible Assessee: Resident Individual.
- Quantum of deduction: Amount earned as royalty from patents
or; ₹3,00,000, whichever is lower.

SECTION 80TTA:
DEDUCTION WITH RESPECT TO INTEREST ON SAVINGS
DEPOSIT:
- Eligible Assessee: Individual/ HUF
- Quantum of deduction: Actual amount of interest or; ₹10,000,
whichever is lower.
{A Resident Senior Citizen will claim deduction under section
80TTB and not under section 80TTA}

SECTION 80TTB:
INTEREST ON DEPOSITS IN CASE OF SENIOR CITIZENS:
- Eligible assessee: SENIOR CITIZEN i.e. resident whose age is 60
or more than 60 during the relevant previous year.
- Quantum of deduction: Actual amount of interest or; ₹50,000,
whichever is lower.
- It should be noted that deduction is allowed w.r.t. interest on
savings bank as well as Fixed Deposits with Banking Company,
Co-operative Society and Post Office.

150
SECTION 80U:
DEDUCTION IN CASE OF INCOME EARNED BY PERSON
WITH DISABILITY:
- Eligible Assessee: Resident individual who at any time during
previous year is certified by a medical authority as a person to be
with disability.
- Quantum of deduction: ₹75,000 and; ₹1,25,000 in case of severe
disability.
- The above amounts are allowed as FLAT DEDUCTION i.e.
irrespective of source of income, amounts invested.

151
ADVANCE TAX, TDS & TCS

MODES OF COLLECTING INCOME TAX FROM AN


ASSESSEE: -

SELF
ADVANCE TAX TDS TCS ASSESSMENT
TAX

Understanding Concept of Advance Tax, TDS & Self-


Assessment Tax with help of illustration:
Say Mr. A has tax liability of ₹4,00,000 for the Assessment Year 2023-24
{Relevant to Previous Year 2022-23}. During the P.Y. 2022-23, he has
paid Advance Tax of ₹2,50,000, his tax deducted at source by his
customers is ₹60,000. Therefore, Total Prepaid Taxes for the A.Y. 2023-
24 is ₹3,10,000. Here, during A.Y.2023-24, he will have to pay Self-
Assessment tax of ₹90,000. {₹4,00,000 (-) 3,10,000}.
However, say in the above case, TDS amount was ₹2,00,000 instead of
₹60,000, then there would be refund of ₹50,000 {Tax payable =
₹4,00,000 (-) Prepaid Taxes: ₹4,50,000}.
In both the above cases, Income Tax Department will be intimated by
Filing of Income Tax Return (ITR).

152
ADVANCE TAX:
When do provisions of Advance Tax become applicable?
Every assessee where Advance Tax payable is ₹10,000 or more for a
year.
*Advance Tax payable = Tax Payable (-) TDS.
NOTE:
Senior Citizen i.e. (Resident age 60 or more during P.Y.) who is not
having PGBP may not pay advance tax. {Therefore, such senior citizens
would pay their entire tax liability by way of TDS done by payers + Self-
Assessment Tax.}

What are the installments in which advance tax is to be


paid & when?
On or before Total amount of tax to be paid
(As % of advance tax liability)
15th June of P.Y. 15%
15th Sept of P.Y. 45%
15th Dec of P.Y. 75%
15th March of P.Y. 100%

NOTE:
Assessees opting for section 44AD & 44ADA presumptive taxation
need not pay advance tax in 4 installments, they may pay it only in
last installment.

What will be the consequences of not paying Advance Tax


within time?
For delay in payment of advance tax, there shall be interest chargeable
under section 234C
Rate of Interest: 1% per month or part thereof on the amount of
shortfall in the payment of advance tax.
153
j NOTE: Interest u/s 234C will apply for 1st installment only if advance tax paid
is less than 12% of advance tax payable &; Interest for 2nd installment shall
apply only if advance tax paid is less than 36% of advance tax payable.

Illustration 1:
Mrs. Lakshmi has tax payable of ₹10,00,000 for the Assessment Year
2023-24. She has paid advance tax as follows:
On 15th June, 2022: ₹1,00,000
On 14th September, 2022: ₹4,00,000
On 10th March, 2023: ₹4,50,000
Compute Interest payable under section 234C for deferment in
payment of advance tax installments, if any.
SOLUTION:
Computation of Interest payable under section 234C for the
Assessment Year 2023-24 {Relevant to Previous Year 2022-23}:
Installment Advance Advance Shortfall Interest (₹)
{On or before} Tax payable Tax paid (₹) (₹)
(₹)
15th June, 1,50,000 1,00,000 50,000 1,500
2022 {₹10,00,000 {₹50,000 * 1%
* 15%} p.m. * 3}
th
15 Sept, 4,50,000 5,00,000 NIL NIL
2022 {₹10,00,000 {₹1,00,000
* 45%} +
₹4,00,000}
th
15 Dec, 2022 7,50,000 5,00,000 2,50,000 7,500
{₹10,00,000 {₹2,50,000 *
* 75%} 1% p.m. * 3}
15th March, 10,00,000 9,50,000 50,000 500
2023 {₹10,00,000 {₹5,00,000 {₹50,000 * 1%
* 100%} + * 1}
₹4,50,000}

154
NOTE:
Interest is charged under section 234C in installment 1 as advance tax
paid is less than 12% of tax payable.

CONSEQUENCE OF TAX REMAINING PAYABLE AS ON 31st


March of the Previous Year:
INTEREST u/s 234B
This section will cover interest beginning 1st day of the Assessment Year
till the actual date of payment of balance tax as self-assessment tax.

Interest of section
234 B is NOT
PAYABLE if total
Advance Tax paid is
90% or more of the
tax payable.

Rate of Interest: 1% per month or part of the month for which delay
continues in payment of tax beyond 31st March of the Previous Year.
Illustration 2:
In above illustration of Mrs. Laxmi, total advance tax paid = ₹9,50,000
out of ₹10,00,000 i.e. 95%, hence, Interest u/s 234B will not be
chargeable.
Illustration 3:
Say in illustration of Laxmi, the last installment was ₹3,00,000 instead
of ₹4,50,000, then what would have been interest u/s 234B, given that
the remaining tax was paid on 15th August, 2023?

155
Solution:
Here, provisions of section 234B shall apply as tax paid is 80%
{8,00,000/10,00,000 * 100} of total tax payable.
Interest Computation shall be as follows:
a. Balance payable as on 31.03 of the Previous Year ₹2,00,000
th
b. Self-assessment tax of ₹2 lakhs paid on 15 Aug, 2023
c. No. of months for which interest applicable 5 months
{April – August, 2023}
d. Interest @ 1% per month or part thereof ₹10,000
{₹2,00,000 * 1% * 5 months}

WHAT IS THE INTEREST APPLICABLE FOR DELAY IN FILING OF INCOME


TAX RETURN?

It is at 1% per month or part thereof for which the delay continues


beyond due date of filing of Income Tax Return.

Interest is computed on amount of tax payable as on 31.03 of the


Previous Year.

Illustration 4:

Due date of filing of Income Tax Return of Mrs. Laxmi is say 31st July,
2023. Considering the original case, where the amount of tax payable
as on 31.03.2023 is ₹50,000, compute the interest payable under
section 234A given that she has filed her Income Tax Return on 8th
August, 2023.

Solution:
Delay in filing of return u/s 139(1) in no. of months including part
thereof: 1 month
Therefore, Interest @ 1% p.m. or part thereof
on ₹50,000 {₹50,000 * 1% * 1 month} ₹500

156
WHAT ARE THE SPECIAL PROVISIONS IN RELATION TO ADVANCE TAX
APPLICABLE IN CASE OF INCOME ARISING FROM CAPITAL GAINS AND
CASUAL INCOME?

PROVISO TO SECTION 234C

➢ Advance tax is payable by an assessee on his/her total income


which includes capital gains and casual income like lotteries,
crossword puzzles, etc.
➢ Since, it is not possible for assessee to estimate his capital gains
or income from lotteries, etc., it has been provided that if any
such income arises after the due date for any installment, then
the entire amount of tax payable (after considering tax
deducted at source) on such capital gains or casual income
should be paid in remaining installments of advance tax, which
are due.
➢ When no such installment is due, the entire tax should be paid
by 31st March of the relevant financial year.
➢ No Interest liability on late payment would arise if the entire
tax liability is so paid.

157
TAX DEDUCTED AT SOURCE (TDS)
FACTORS TO BE UNDERSTOOD TO UNDERSTAND TDS:

PAYER PAYEE

NATURE OF PAYMENT

THRESHOLD
NORMAL TIME OF DEDUCTION
LIMIT
RATE OF
TDS

Understanding and analysing above factors with help of a


table:
{Refer next page}

158
Sr. Section Nature of Threshold Payer Payee Normal Time of
No. Payment Limit (₹) Rate of Deduction
TDS
1. 192 Salary Basic Any Individual Average At the time of
exemption Employer employee rate of tax payment
limit as compute
Income Tax d on basis
payable by of rates in
employee = force.
TDS by
employer
2. 192A Premature Payment or Trustees Individual 10%, but At the time of
Withdrawal aggregate of the EPF employee if failure payment.
from payment of scheme or to furnish
employee more than any PAN, TDS
Provident or equal to authorise at
Fund ₹50,000 d person maximum
under the marginal
scheme rate of
tax.
3. 193 Interest on Interest on Any Any 10% At time of credit
securities 8% Savings Assessee Resident to the Account
Bond: of the payee or
₹10,000 in at the time of
a F.Y. making
Interest on payment,
debentures whichever is
issued by earlier.
Company in
which
Public are
substantiall
y
interested:
₹5,000 in a
F.Y.
In any
other case:
No
threshold.
159
4. 194 Dividend Dividend Principal Resident 10% Before making
paid or officer of a Shareholde payment or;
credited to domestic r distribution of
an company. dividend.
individual
shareholde
r by mode
other than
cash:
₹5,000
5. 194A Interest Interest Any Any 10% At the time of
other than paid or person Resident credit to the
interest on credited by other than account of the
securities a Banking individual payee or at the
Co., Co.Op or HUF. time of making
Society, NOTE: payment,
Post Office: Individual whichever is
₹40,000 in or HUF earlier.
a F.Y. shall also
In all above be
cases, if covered if
payee is their
Senior Gross
Citizen: Receipts
₹50,000. in
In all other preceding
cases: P.Y.
₹5,000. exceed ₹1
crore in
business/
₹50 lakhs
in
profession
6. 194B Winnings ₹10,000 Any Any Person. 30% At the time of
from Lottery, Person. payment.
Crossword
puzzles or
card games
or games of
any other
sort.
160
7. 194BB Winnings ₹10,000 Any Any Person. 30% At the time of
from Horse Person payment.
Races
8. 194C Payment to For single Same as Any If PAYEE is At the time of
Contractor payment: section resident IND./HUF credit to the
₹30,000 194A contractor. 1% Account of the
For payee or at the
aggregate Any other time of making
of case: 2% payment,
payments whichever is
during the earlier.
year:
₹1,00,000
9. 194D Insurance ₹15,000 in Any Any 5% At the time of
Commission a F.Y. Person Resident credit to the
Account of the
payee or at the
time of making
payment,
whichever is
earlier.
10. 194DA Any sum ₹1,00,000 Any Any 5% At the time of
under a life person Resident payment.
insurance
policy.
11. 194E Payment to No Any Non 20% + At the time of
NON Threshold. Person Resident H&EC @ credit to the
RESIDENT sportsman 4% of Account of the
sportsman or including 20% = payee or at the
sports an athlete 20.8%. time of making
association or payment,
of income entertainer whichever is
referred in or NR earlier.
section sports
115BBA institution.
12. 194EE Payment of ₹2,500 in a Any Individual 10% At the time of
Deposit Financial Person or HUF. payment.
under Year.
National
Savings
Scheme.
161
13. 194G Commission ₹15,000 in Any Any Person. 5% At the time of
on sale of a F.Y. Person. credit to the
lottery Account of the
tickets. payee or at the
time of making
payment,
whichever is
earlier.
14. 194H Commission ₹15,000 in Same as Any 5% At the time of
or Brokerage a Financial section Resident credit to the
Year. 194A Account of the
payee or at the
time of making
payment,
whichever is
earlier.
15. 194-I Rent ₹2,40,000 Same as Any Land and At the time of
in a section Resident Building: credit to the
Financial 194A. 10%. Account of the
Year Plant and payee or at the
other: 2% time of making
payment,
whichever is
earlier.
16. 194-IA Payment of More than Any Resident 1% At the time of
transfer of or equal toPerson Transferor credit to the
certain ₹50,00,000 {Here, it is Account of the
immovable . not payee or at the
property required time of making
other than for payer payment,
agricultural to have whichever is
land. TAN} earlier.
17. 194-IB Payment of More than Individual Any 5% At time of credit
rent by ₹50,000 /HUF resident of rent for the
certain per month {Other last month of
individuals or or part of than the previous
HUF. month. whose year or; last
total sales month of
from tenancy, if the
business/ property is
profession vacated during
162
exceed ₹1 the year, as the
crore/ ₹50 case may be, to
lakhs in the account of
last F.Y.} the payee or at
the time of
payment,
whichever is
earlier.
18. 194-IC Payment No Any Any 10% At the time of
under threshold person resident. credit to the
specified specified. paying Account of the
agreement any sum payee or at the
referred to in by way of time of making
section kind payment,
45(5A) under a whichever is
registered earlier.
agreemen
t wherein
land and
building
are
handed
over by
owner for
developm
ent of real
estate
project for
a
considerat
ion being
share in
Land or
Bldg. or
both in
such
project,
with
payment
of part
considerat
163
ion in
cash.
19. 194J Fees for Prof. ₹30,000 in Same like Any 10% At the time of
Services or a financial section resident. credit to the
Technical year for 194A. 2%: Account of the
Services or EACH where payee or at the
Royalty or CATEGORY payee is time of making
Non of income. engaged payment,
Compete {No only in whichever is
Fees or Threshold business earlier.
Director’s limit w.r.t. of
Rem. payment to operation
a director of call
of a centre.
company} Also in
case of
fees for
technical
services
or
royalty,
where
such
royalty is
in nature
of
considera
tion for
sale,
distributi
on or
exhibition
of
cinemato
graph
films.
20. 194K Income on ₹5,000 Any Any 10% At the time of
units other person resident. credit to the
than in responsibl Account of the
nature of e for payee or at the
paying time of making
164
Capital any payment,
Gains. income in whichever is
respect of earlier.
units of
Mutual
Funds.
21. 194LA Compensatio ₹2,50,000 Any Any 10% At the time of
n on Person resident. payment.
acquisition of responsibl
certain e for
immovable paying
property any sum in
other than nature of
agricultural compensa
land {i.e. tion or
compulsory enhanced
acquisition} compensa
tion on
compulso
ry
acquisitio
n of
immovabl
e
property.
22. 194M Payment to ₹50,00,000 Individual Any 5% At the time of
Contractors in a or HUF resident. credit to the
Commission Financial other than Account of the
on Year. those who payee or at the
brokerage, are time of making
Fees for required payment,
professional to deduct whichever is
services tax at earlier.
source u/s
194C,
194H and
194J.
23. 194N Cash ₹1 crore A banking Any Person 2% of At the time of
Withdrawals Company such sum. payment of such
or any sum.
bank or
165
banking In case,
institution recipient
A Co.op has not
society filed ITR
engaged for all the
in carrying 3
on the immediat
business ely
of banking preceding
or post P.Y.s for
office. which
time limit
u/s 139(1)
has
expired,
such sum
shall be
the
amount
or
aggregate
of
amounts
in cash
more
than ₹20
lakhs
during
the P.Y.,
TDS @ 2%
where
such
withdraw
al
exceeds
₹20 lakhs
but does
not
exceed ₹1
crore &; if
withdraw
166
al
exceeds
₹1 crore,
then @
5%.
24. 194-O Payment to More than ECO who E- 1% of At the time of
E-Commerce ₹5 lakhs facilitates commerce gross credit to the
Operator being gross sale of Participant amount Account of the
participant. amount of goods or of sale or payee or at the
sales or provision service or time of making
services or of services both. payment,
both in a of an ECO whichever is
F.Y. to an e- participan earlier.
commerce t through
participant, a digital or
being electronic
Individual/ facility or
HUF & such platform.
ECO E.g.:-
participant Amazon,
has Flipkart,
furnished OLA, etc.
PAN or
Adhaar
Number to
ECO.

No
threshold
limit in
other
cases.
25. 194P Deduction of At Specified Resident Tax
tax by applicable Bank. Specified payable
specified rates in senior by
bank in case force. {Tax citizen assessee
of specified payable by {Age: 75 or = Tax to
senior citizen assessee = more} be
tax to be deducted
deducted at source.
at source}
167
26. 194Q TDS on ₹50,00,000 Buyer Resident 0.1% of At the time of
purchase of whose purchases credit to the
goods. turnover exceeding Account of the
during the ₹50 lakhs. payee or at the
preceding time of making
P.Y. payment,
exceeds whichever is
₹10 crore. earlier.
27. 194R TDS on ₹20,000 Same like Any 10% of Before releasing
benefit or 194A resident value of the benefit of
perquisite in such perquisite.
respect of benefits.
business or
profession

Common Note:
As per section 206AA, if PAN is not furnished by Payee, then the rate
of tax to be deducted at source shall be rate as per the said section of
TDS or 20%, whichever is HIGHER. (Except for section 194 0 & 194Q,
where it is 5%)

Note for section 194C:


No TDS is required where amount is credited to person engaged in
business of plying, hiring or leasing of goods carriages, if he furnishes
his PAN to the deductor.
It should be noted that he should own not more than 10 goods
carriages at any time during the Previous Year.

Illustration 1:
Mr. X, a salaried individual, pays rent of ₹55,000 per month to Mr. Y
from June, 2022. Is he required to deduct tax at source? If so, when is
he required to deduct tax at source? Also, compute the amount of tax
to be deducted at source.

168
Would your answer change if Mr.X vacated the premises on 31st
December, 2022?
Also, what would be your answer if Mr. Y does not provide his PAN to
Mr.X?
Solution:
Since, Mr.X pays rent exceeding ₹50,000 per month in the F.Y. 2022-23,
he is liable to deduct tax at source at 5% of such rent for F.Y. 2022-23
under section 194-IB.
Thus, ₹27,500 {₹55,000 * 10 months * 5%} has to be deducted from
rent payable for March, 2023.
If Mr.X vacated the premises in December, 2022, then tax of ₹19,250
{₹55,000 * 7 months * 5%} has to be deducted from rent payable for
December, 2022.
In case Mr. Y does not provide his PAN to Mr.X, tax would be
deductible @ 20% instead of 5%
In case 1 above, this would amount to ₹1,10,000 {₹55,000 * 10 months
* 20%}, but the same has to be restricted to ₹55,000, being the rent for
March, 2023.
In case 2 above, this would amount to ₹77,000 {₹55,000 * 7 months *
20%}, but the same would be restricted to ₹55,000, being the rent for
December, 2022.

Illustration 2:
XYZ Ltd. makes a payment of ₹28,000 to Mr. Ganesh on 02.08.2022
towards fees for professional services and another payment of ₹25,000
to him on the same date towards fees for technical services. Discuss
whether TDS provisions under section 194J are attracted.
Solution:

169
Here, TDS provisions under section 194J would not get attracted as the
limit of ₹30,000 is applicable for fees for professional services and fees
for technical services, separately.
It is assumed that there is no other payment to Mr. Ganesh towards
fees for professional services and fees for technical services during P.Y.
2022-23.
Illustration 3:
Mr. Sharma, a resident Indian aged 77 years, gets pension of ₹52,000
per month from the UP State Government. The same is credited to his
savings account in SBI, Lucknow Branch. In addition, he gets interest @
8% p.a. on Fixed Deposits of ₹20 lakhs with the said bank. Out of
deposits of ₹20 lakhs, ₹2 lakhs represent 5 years term deposits made
by him on 1.4.2022. Interest on savings bank credited to his SBI Savings
Account for the P.Y. 2022-23 is ₹9,500.
1. From the above facts, compute the Total Income and tax liability
of Mr. Sharma for the Assessment Year 2023-24, assuming that
he has not opted for section 115BAC.
2. What would be the amount of tax deductible at source assuming
that the same is a specified bank? Is Mr. Sharma required to file
his return of income for the A.Y. 2023-24, if tax deductible at
source is fully deducted? Examine.
3. Is Mr. Sharma required to file his return of income for A.Y. 2023-
24, if the fixed deposit of ₹20 lakhs was with Canara Bank instead
of SBI, other facts remaining the same?
Solution:
1. Computation of Total Income of Mr. Sharma for the Assessment
Year 2023-24 (Relevant to Previous Year 2022-23):
PARTICULARS Amount (₹) Amount (₹)
a. Income from Salaries
Pension (₹52,000 * 12) 6,24,000
Less: Deduction u/s 16(ia) (50,000) 5,74,000
b. Income from Other Sources
Interest on Fixed Deposits
170
{₹20,000,000 * 8%} 1,60,000
Interest on Savings Account 9,500 1,69,500
Gross Total Income 7,43,500
c. Less: Deductions under chapter
VI-A
u/s 80C
Five years term deposit
{₹2,00,000 but restricted to 1,50,000
₹1,50,000}
u/s 80TTB {Interest on FD &
Savings Bank Account, restricted
to ₹50,000} 50,000 (2,00,000)
Total Income 5,43,500
Computation of Tax liability:
On first ₹3,00,000 Nil
On income between ₹3,00,000 to
₹5,00,000, tax at 5% 10,000
On income between ₹5,00,000 to
₹10,00,000, tax @ 20%
{₹43,500 * 20%} 8,700
Total 18,700
Add: Health and Education Cess
@ 4% 748 19,448
Rounded off to nearest of ₹10
u/s 288B 19,450

2. SBI, being a specified bank, is required to deduct tax at source u/s


194P and remit the same to Central Government. In such a case,
Mr. Sharma would not be required to file his return of Income u/s
139.

3. If the fixed deposit of ₹20 lakh is with a bank other than SBI, which
is the bank where his pension is credited, then, Mr. Sharma would
not qualify as a specified senior citizen. In this case, Mr. Sharma
would have to file his return of income u/s 139, since his total
income (without giving effect to deduction under chapter VI-A)
exceeds the basic exemption limit.
171
Illustration 4:
Mr. Gupta, a resident Indian, is in retail business and his turnover for
Financial Year 2021-22 was ₹12 crores. He regularly purchases goods
from another resident, Mr. Agarwal, a wholesaler, and the aggregate
payments during the Financial Year 2022-23 was ₹95 lakhs {20 lakhs on
1.6.22, 25 lakhs on 12.8.22, 22 lakh on 23.11.22 and 28 lakh on 25.3.22}
Assume that said amounts were credited to Mr. Agarwal’s Account in
books of Mr. Gupta on the same date. Mr. Agarwal’s turnover for the
Financial Year 2021-22 was ₹15 crore.
1. Based on above facts, examine TDS/TCS implications, if any under
the Income Tax Act, 1961.
2. Would your answer be different, if Mr. Gupta’s turnover for F.Y.
2021-22 was ₹8 crores, all other facts remaining the same?
3. Would your answer to (1) and (2) change, if PAN has not been
furnished by the buyer or seller as required?
Solution:
1.
• Since Mr. Gupta’s {Buyer} Turnover for F.Y. 2021-22 exceeds ₹10
crores
• and payments made by him to Mr. Agarwal, a resident seller
exceeds ₹50 lakhs in the P.Y. 2022-23, he is liable to deduct tax @
0.1% of ₹45 lakhs {being the sum exceeding ₹50 lakhs}.
• No tax is to be deducted on payments made on 1.6.2022 &
12.08.22 as the aggregate payments till that date is ₹45 lakhs i.e.
it has not exceeded the threshold limit of ₹50 lakhs.
• Tax of ₹1,700 (0.1% of ₹17,00,000) has to be deducted u/s 194Q
from the payment/ credit of ₹22 lakhs on 23.11.2022 {22 lakhs (-
) 5 lakhs, being the balance unexhausted threshold limit}
• Tax of ₹2,800 (0.1% of ₹28,00,000) has to be deducted u/s 194Q
from the payment/credit of ₹28 lakhs on 25.03.2023.

Note: In this case, since both section 194Q and 206C(1H) i.e of
TCS applies, tax has to be deducted u/s 194Q.
172
2.
• If Mr. Gupta’s turnover for F.Y. 2021-22 was only ₹8 crore, TDS
PROVISIONS u/s 194Q WOULD NOT GER ATTRACTED.
However, TCS provisions under section 206(1H) would be
attracted in hands of Mr. Agarwal, since the turnover exceeds ₹10
crores in Financial Year 2021-22 and the receipts from Mr. Gupta
exceed ₹50 lakhs.
• No tax is to be collected u/s 206C(1H) on 1.6.22 & 12.8.22 as the
aggregate receipts till that date i.e. ₹45 lakhs, has not exceeded
the threshold limit of ₹50 lakhs.
• Tax of ₹1,700 {0.1% of ₹17,00,000} has to be collected on
23.11.2022 (₹22 lakhs (-) ₹5 lakhs)
• Tax of ₹2,800 (0.1% of ₹28,00,000) has to be collected on
25.03.2023.

3.
If PAN is not furnished:

In case 1:
TDS @ 5% instead of 0.1%

In case 2:
TCS @ 1% instead of 0.1%.
Time limit for payment of Tax Deducted at Source to the
Government:
Case Due date of
payment
a. Sums deducted by an office of
Government On the same day
- Tax is to be paid without production
of Income Tax Challan
Within 7 days
- Where the tax is paid accompanied from the end of
by an income tax challan. the month in
173
which deduction
is made.
b. Sums deducted by deductor other than
Government Office:
- Where the income or amount is By 30th April
credited or paid in the month of
March.
Within 7 days
- In any other case from the end of
the month in
which deduction
is made.

Within 30 days
- Tax deducted u/s 194-IA, 194-IB, from the end of
194M month of
deduction.

Due dates of furnishing Quarterly statements of TDS:

Sr. Date of ending of Due Date


No. quarter of the financial
year
th
1. 30 June 31st July of the Financial Year
2. 30th September 31st October of the Financial Year
3. 31st December 31st January of the Financial Year
4. 31st March 31st May of the Financial Year
immediately following the financial
year in which the deduction is
made.

174
INTEREST LIABILITY FOR DELAY IN DEDUCTION OF TAX AT
SOURCE AND DELAY IN DEPOSIT OF TDS:

- Interest @ 1% per month or part of the month is chargeable


for delay in deduction of tax at source.
- Interest @ 1.5% per month or part of the month is
applicable for delay in deposit of TDS.

Illustration 5:

An amount of ₹40,000 was paid to Mr. X on 1.7.2022 towards fees for


professional services without deduction of tax at source. Subsequently,
another payment of ₹50,000 was due to Mr.X on 28.02.2023, from which
tax @ 10% {amounting to ₹9,000} on entire amount of ₹90,000 was
deducted. However, this tax of ₹9,000 was deposited only on
22.06.2023. Compute the interest chargeable.
Solution:
Computation of Interest:

PARTICULARS Amount (₹)


a. 1% on Tax deductible but not deducted i.e. 1%
on ₹4,000 for 8 months 320
b. 1.5% on tax deducted but not deposited i.e.
1.5% on ₹9,000 for 4 months 540
860

175
TAX COLLECTION AT SOURCE

1. Sellers of following goods are required to collect tax at source


from the buyers at specified rates as given below:
Nature of goods Percentage
Alcoholic Liquor for Human consumption 1%
Tendu Leaves 5%
Timber obtained for forest lease 2.5%
Timber obtained from any mode other than above 2.5%
Any other forest produce not being timber or tendu
leaves 2.5%
Scrap 1%
Minerals, being coal or lignite or iron ore 1%

In above cases, NO TCS if, such buyer furnishes a declaration in writing


that goods are to be utilised for the purpose of manufacturing or for
purpose of generation of power and not for trading purposes.

2. Lease of license of parking lot, toll plaza, mine or quarry:


TCS @ 2% to be collected from licensee (other than public sector
company.)

3. Sale of MOTOR VEHICLE OF VALUE EXCEEDING ₹10 lakhs:


TCS @ 1% of sale consideration from buyer.
4. Overseas remittance or an overseas tour package:

Rate of TCS in case of collection by an authorised dealer:


Sr. Amount and purpose of remittance Rate of
No. TCS
1. (a) where the amount is remitted for a Nil
purpose other than purchase of overseas
tour programme package; and
(b) the amount or aggregate of the amounts
being remitted by a buyer is less than ₹7
lakhs in a Financial Year.

176
2. (a) Where the amount is remitted for a 5% of the
purpose other than purchase of overseas amounts
tour program package; and or
(b) the amount of aggregate of amounts in aggregate
excess of ₹7 lakhs is remitted by the buyer in of
a financial year. amounts in
excess of
₹7 lakhs
3 (a) Where the amount being remitted out 0.5% of
is a loan obtained from any financial the
institution as defined u/s 80E for the amount or
purpose of pursuing any education; aggregate
and of
(b) The amount or aggregate of amounts amounts in
in excess of ₹7 lakhs is remitted by the excess of
buyer in a Financial Year. ₹7 lakhs

5. Sale of goods of value exceeding ₹50 lakhs:


• Every person being a seller
• Who receives any amount as consideration for sale of goods
• Of value exceeding ₹50 lakhs in a previous year
• Is required to collect tax at source at 0.1%
• Of the sale consideration in excess of ₹50 lakhs.
• NO TCS if the buyer is liable to deduct tax at source on
goods purchased by him from the seller and has deducted
such tax u/s 194Q.
Common Notes for TCS:
• If PAN is not furnished by the buyer, TAX is required to be
collected at HIGHER of 2: -
a. At twice the rate of TCS
Or;
b. 5% {1% in case of TCS for goods u/s 206C(1H)}

177
RESIDENTIAL STATUS & SCOPE OF TOTAL INCOME

WHY SHOULD WE UNDERSTAND RESIDENTIAL STATUS OF AN ASSESSEE??


RESIDENTIAL STATUS IS IMPORTANT TO BE DETERMINED AS ANY COUNTRY
LEVIES TAXES ON INCOME ON 2 BASIS:
1. On basis of RESIDENTIAL STATUS of assessee.
2. On the basis of source of income.
Therefore, Government of India also charges Income Tax on 2 basis: -
1. With respect to income of Resident assessee in India.
2. With respect to income of any assessee of which the source is in India.

Hence, knowing how to determine Residential Status of assessee becomes


important.

DETERMINING RESIDENTIAL STATUS FOR INDIVIDUAL:

INDIVIDUAL

RESIDENT NON RESIDENT

NOT
ORDINARY ORDINARY
RESIDENT RESIDENT
(R-OR) (R-NOR)

178
WHO WILL BE RESIDENT?
ONE WHO SATISFIES ANY ONE OUT OF TWO BASIC CONDITIONS:
1. HE IS IN INDIA DURING THE P.Y. FOR 182 DAYS OR MORE.
2. HE IS IN INDIA FOR 60 DAYS OR MORE DURING THE P.Y. + 365 DAYS
OR MORE DURING 4 P.Y. IMMEDIATELY PRECEEDING RELEVANT P.Y.
(2nd condition not to be checked for Indian citizen or person of Indian
origin or crew member of ship employed with NR shipping Co. leaving
India during P.Y) RESIDENT

ORDINARY RESIDENT
NOT ORDINARY RESIDENT
(R-OR)
(R-NOR)

WHEN?
1. He satisfies any 1 out of 2 basic conditions
and;
2. 2 out of 2 additional conditions:
a. He is Resident in India for 2 or more P.Y preceeding
relevant P.Y.
b. His stay in India is 730 days or more during the 7
P.Ys immediately preceeding the relevant P.Y.

179
For declaring Indian Citizen or person on
Indian Origin to be R-NOR, following
clauses have been introduced:
{As 60 days clause in basic conditions is Not
Applicable to them}
1. If Citizen of India/ Person of Indian
Origin satisfies the following
conditions: -
a. His Income from Indian sources is
exceeding ₹15 lakhs during the P.Y.
and;
b. His stay in India is 120 days or more
during the P.Y. + 365 days or more
during the 4 P.Y.s immediately
preceding the relevant P.Y.

2. In case citizen of India does not have


stay of 120 days or more in India
during the P.Y., still he will be R-NOR if
his Indian income during the P.Y. is
exceeding ₹15,00,000 &; he does not
have any foreign income based on his
domicile.

180
DETERMINING RESIDENTIAL STATUS FOR HUF:

HUF

RESIDENT NON RESIDENT


When?
Control and management wholly or
partly in India.

Ordinary Resident NOT ORDINARY


(R-OR) RESIDENT (R-NOR)

When?
When?
Karta of HUF satisfies both the Karta of HUF does not satisy both the
additional conditions of being conditions of being ordinary resident.
ordinary resident.

DETERMINING RESIDENTIAL STATUS OF COMPANY:

COMPANY

RESIDENT NON RESIDENT

When?
a. Indian Company
b. Any foreign company having
it's PLACE OF EFFECTIVE
MANAGEMENT (POEM) in India
at any point of time during P.Y.

181
DETERMINING RESIDENTIAL STATUS FOR FIRMS/ LOCAL
AUTHORITIES/ ARTIFICIAL JURIDICAL PERSON:

FIRMS/ LOCAL AUTHORITIES/ AJP

RESIDENT NON-RESIDENT
{Control and Management is wholly or {Control and
partly situated in India} Management is situated
wholly OUTSIDE INDIA}

182
SCOPE OF TOTAL INCOME:
SCOPE OF TOTAL INCOME
RESIDENT AND ORDINARY RESIDENT:
GLOBAL INCOME is taxable in their hands in India.

RESIDENT BUT NOT ORDINARY RESIDENT:


1. Income which is received or deemed to be
received, accrued or arising in India is taxable in
their hands.
2. Income which accrues or arises outside India
being derived from a business controlled in or
profession set up in India.

NON RESIDENT:
Income which is received or deemed to be
received, accrued or arising in India is taxable
in their hands.

Illustration 1:
An Australian Cricketer visits India for 100 days in every financial year.
This has been his practice for the past 10 financial years.
a. Find out his residential status for the assessment year 2023-24.
b. Would your answer change if the above facts relate to an Mr.X
who is Indian citizen residing in Australia and representing
Australian cricket team?
c. What would be your answer if Mr.X had visited India for 120 days
instead of 100 days every year, including P.Y. 2022-23?

183
SOLUTION:

1. Period of stay of Australian Cricketer in India in P.Y. 2022-23 is


100 days.
Further, in preceding 4 previous years it is 400 days {100 days * 4
years}.
So, he has been in India for a period of 60 days or more in the
previous year 2022-23 and 365 days or more in 4 immediately
preceding previous years.
Therefore, he satisfies one of the basic conditions and so, he is
resident for the assessment year 2023-24.
However, number of days stay in 7 immediately preceding
previous years is only 700 days {100 * 7} and therefore, he is not
satisfying the additional condition of staying 730 days or more in
7 preceding previous years.
Hence, he is NOT AN ORDINARY RESIDENT.
Therefore, his status is Resident but not ordinary resident (R-
NOR) for the assessment year 2023-24.

2. If the above facts relate to Mr.X, an Indian citizen, who is residing


in Australia, coming to visit to India, he would be treated as NON
RESIDENT in India irrespective of his total income (excluding
income from foreign sources), as his stay in India in the current
financial year is, in any case, less than 120 days.

3. In this case, since Mr.X’s total income (excluding income from


foreign sources) exceeds ₹15 lakhs, he would be treated as
Resident but Not Ordinary Resident in India for the P.Y. 2022-23,
since his stay in India is 120 days in the P.Y. 2022-23 and 480 days
(i.e. 120 days * 4 years) in the immediately 4 preceding previous
years.
However, if his total income (excluding income from foreign
sources) does not exceed ₹15 lakhs, he would be treated as Non-
Resident in India for the P.Y. 2022-23, since his stay in India is less
than 182 days P.Y. 2022-23.

184
ILLUSTRATION 2:

Mr. B, a Canadian citizen comes to India for the first time during
the Previous Year 2018-19. During the Financial Years 2018-19,
2019-20, 2020-21, 2021-22 & 2022-23, he was in India for 55 days,
60 days, 90 days, 150 days and 70 days respectively.
Determine the residential status for the Assessment Year 2023-
24.

SOLUTION:

During the previous year 2022-23, Mr. B was in India for 70 days
and during the 4 years preceding the previous year 2022-23, he
was in India for 355 days {55 + 60 + 90 + 150}.

Thus, he does not satisfy the basic condition under section 6.


Therefore, he is a NON-RESIDENT for the previous year 2022-23.

ILLUSTRATION 3:

The business of HUF is transacted from Australia and all the policy
decisions are taken there. Mr.E, the Karta of the HUF, who was
born in Kolkata, visits India during Previous Year 2022-23 after 15
years. He comes to India on 01/04/2022 and leaves for Australia
on 01/12/2022.
Determine the residential status of Mr.E and the HUF for A.Y.
2023-24.

SOLUTION:

a. During P.Y. 2022-23, Mr. E has stayed in India for 245 days.
Therefore, he is RESIDENT. However, since he has come to
India after 15 years, he does not satisfy the conditions of being
ordinary resident.
Therefore, the residential status of Mr. E for the P.Y. 2022-23
is RESIDENT BUT NOT ORDINARY RESIDENT.
185
b. Since the business of HUF is transacted from Australia and
policy decisions are taken there, it is assumed that control and
management is in Australia i.e. control and management is
WHOLLY OUTSIDE INDIA.
Therefore, the HUF is a NON-RESIDENT for the P.Y. 2022-23.

ILLUSTRATION 4:

Compute the total income in the hands of an individual aged 35 years,


being a resident and ordinary resident, resident but not ordinary
resident and non-resident for the A.Y. 2023-24:
PARTICULARS Amount (₹)
Interest on UK Development Bonds, 50% of Interest
received in India 10,000
Income from a business in Chennai {50% is received
in India} 20,000
Short Term Capital Gains on sale of shares of an
Indian Company, received in London 20,000
Dividend from British Company received in London 5,000
Long Term Capital Gains on sale of plant at
Germany, 50% of gains are received in India. 40,000
Income received from business in Germany which is
controlled from Delhi (₹40,000 received in India) 70,000
Profits from business in Delhi but managed entirely
from London 15,000
Income from House Property in London deposited
in a Bank at London, brought to India (Computed) 50,000
Income on debentures in an Indian Company,
received in London 12,000
Fees for technical services rendered in India but
received in London 8,000
Profits from business in Mumbai, managed from
London 26,000
Income from property in Nepal received there
(Computed) 16,000
Past foreign untaxed income brought to India
during the previous year 5,000
186
Income from Agricultural land in Nepal, received
there and then brought to India 18,000
Income from profession in Kenya which was set up
in India, received there but spent in India 5,000
Gift on occasion of his wedding 20,000
Interest on Savings Bank deposit in SBI 12,000
Income from business in Russia, controlled from 20,000
Russia
Dividend from Reliance Petroleum Ltd., an Indian
Company 5,000
Agricultural income from land in Rajasthan 15,000

SOLUTION:
Figures in ₹
PARTICULARS R-OR R-NOR NR
Interest on UK Development Bonds,
50% of Interest received in India 10,000 5,000 5,000
Income from a business in Chennai
{50% is received in India} 20,000 20,000 20,000
Short Term Capital Gains on sale of
shares of an Indian Company,
received in London 20,000 20,000 20,000
Dividend from British Company
received in London 5,000 - -
Long Term Capital Gains on sale of
plant at Germany, 50% of gains are
received in India. 40,000 20,000 20,000
Income received from business in
Germany which is controlled from
Delhi (₹40,000 received in India) 70,000 70,000 40,000
Profits from business in Delhi but
managed entirely from London 15,000 15,000 15,000
Income from House Property in
London deposited in a Bank at
London, brought to India
(Computed) 50,000 - -
Income on debentures in an Indian
Company, received in London 12,000 12,000 12,000
187
Fees for technical services rendered
in India but received in London 8,000 8,000 8,000
Profits from business in Mumbai,
managed from London 26,000 26,000 26,000
Income from property in Nepal
received there (Computed) 16,000 - -
Past foreign untaxed income
brought to India during the previous
year - - -
Income from Agricultural land in
Nepal, received there and then
brought to India 18,000 - -
Income from profession in Kenya
which was set up in India, received
there but spent in India 5,000 5,000 -
Gift on occasion of his wedding - - -
Interest on Savings Bank deposit in
SBI 12,000 12,000 12,000
Income from business in Russia,
controlled from Russia 20,000 - -
Dividend from Reliance Petroleum
Ltd., an Indian Company 5,000 5,000 5,000
Agricultural income from land in
Rajasthan - - -
Therefore, Gross Total Income 3,52,000 2,18,000 1,83,000
Less: Deduction u/s 80TTA {Interest
on Savings Bank Account – ceiling
Limit ₹10,000} (10,000) (10,000) (10,000)
Therefore, Total Income 3,42,000 2,08,000 1,73,000

188
AGRICULTURAL INCOME &
OTHER EXEMPTIONS UNDER INCOME TAX
What is Agricultural Income? {Section 2(1A) of Income Tax Act}

AGRICULTURAL
INCOME IS
EXEMPT under
section 10(1) of
Income Tax
Act.

189
Income derived
from such land
by:
a. agriculture
b.process
ordinarily
employed to
rended the
produce fit to be
Rent or Revenue
taken to market Income derived
derived from
{done by either from any farm
land situated in
cultivator or building required
India and used
receiver of rent for agricultural
for agricultural
in kind} operations.
purposes.
c. Sale by
cultivator or
receiver of rent
in kind of such
agricultural
produce raised
or received by
him.

190
SOME IMPORTANT POINTS TO BE CONSIDERED IN “AGRICULTURAL
INCOME”: -
• The term “Agriculture” CANNOT be extended to all activities
which have some distant relation to land like diary farming,
breeding and rearing of livestock, butter and cheese making and
poultry farming.

INCOME DERIVED FROM SAPLINGS OR SEEDLINGS GROWN IN A NURSERY


WOULD BE DEEMED TO BE AGRICULTURAL INCOME, WHETHER OR NOT THE
BASIC OPERATIONS WERE CARRIED OUT ON LAND.

• Income arising from use of farm building for any purpose


(including letting for residential purpose or for the purpose of
business or profession) other than agriculture would NOT BE
agricultural income.
• The income from farm building would be agricultural income
only if the following conditions are satisfied:
a. The building should be on or in immediate vicinity of the land;
b. The receiver of rent or revenue or cultivator, by reason of his
connection with such land require it as a dwelling house or as
a store house.
c. The land should either be assessed to land revenue in India or
be subject to local rate assessed and collected by the officers
of the Government as such
or;
where the land is not so assessed to land revenue in India or it
is not subject to local rate, then the land should be Rural
Agricultural land {as discussed in capital gains topic}.

191
APPORTIONMENT OF INCOME BETWEEN BUSINESS INCOME &
AGRICULTURAL INCOME {Already discussed in PGBP}

PARTIAL INTEGRATION OF AGRICULTURAL INCOME WITH NON-


AGRICULTURAL INCOME:

• Applicable to INDIVIDUALS/ HUF/ AOPs/ BOIs and Artificial


Juridical Persons.
• Conditions to be satisfied for partial integration:
a. The net agricultural income should exceed ₹5,000 during the
year.
b. Non – agricultural income should exceed the basic exemption
limit {i.e. ₹5,00,000 for resident very senior citizens, ₹3,00,000
for resident senior citizens and ₹2,50,000 for all others}.

TAX CALCULATIONS IN CASE OF PARTIAL INTEGRATION:


1. Take “Non-Agricultural income + Net Agricultural income” &
compute tax on the same.
2. Take “Net Agricultural income + Basic exemption limit” &
compute tax on the same.
3. Income tax under step (1) (-) Income tax under step (2)
4. Amount in step 3 will be increased by surcharge. It would be
reduced by rebate, if any, available u/s 87A.
5. Amount in step 4 to be increased by Health and Education Cess
@ 4%.

ILLUSTRATION 1:
Mr. X, a resident, has provided the following particulars of his income
for the Previous Year 2022-23
a. Income from Salaries (Computed): ₹2,80,000
b. Income from House Property (Computed): ₹2,50,000
c. Agricultural income from land in Jaipur: ₹4,80,000
d. Expenses incurred for earning agricultural income: ₹1,70,000
192
Compute his tax liability for the Assessment Year 2023-24 assuming
his age is –
a. 45 years
b. 70 years.
Assume that Mr. X does not opt for the provisions of section 115BAC.
SOLUTION:
Computation of Total Income of Mr.X for the Assessment Year 2023-
24 (Relevant to Previous Year 2022-23)
a. Computation of tax liability (age 45)

For the purposes of partial integration of taxes, Mr. X has


satisfied both the conditions i.e.

1. Net Agricultural income exceeds ₹5,000 and;


2. Non-Agricultural income exceeds the basic exemption limit of
₹2,50,000.
His tax liability is computed as follows: -
PARTICULARS Amount (₹) Amount (₹)
1. Total of Net Agricultural
income and total income & tax
liability on the same
a. Net Agricultural income
(₹4,80,000 (-) ₹1,70,000) 3,10,000
b. Total Income
Income from Salaries 2,80,000
Income from House 2,50,000
Property
Therefore, Gross Total
Income/ Total Income 5,30,000
c. Total 8,40,000
Tax Liability
On ₹2,50,000 -
On income between ₹2.5
lakhs to ₹5 lakhs, tax @ 5% 12,500
193
On income between ₹5
lakhs to ₹10 lakhs, tax @ 68,000
20%
Therefore, Total Tax 80,500
2. Tax on Basic Exemption limit +
Net Agricultural Income
Basic Exemption Limit 2,50,000
Net Agricultural income 3,10,000
Total 5,60,000
Tax liability
On income upto ₹2.5 lakhs -
On income between ₹2.5 lakhs
to ₹5 lakhs 12,500
On income between ₹5 lakhs to
₹10 lakhs, tax @ 20% 12,000
Therefore, Total Tax 24,500
3. Tax under step 1 (-) tax under
step 2 56,000
4. Add: Health and Education
Cess @ 4% 2,240
5. Therefore, Total Tax liability 58,240

Computation of tax liability (age: 70 years)


Here also, both basic conditions for partial integration are satisfied as
total income exceeds basic exemption limit of ₹3,00,000 and as already
discussed, net agricultural income exceeds ₹5,000.
PARTICULARS Amount (₹) Amount (₹)
1. Tax on total income + Net
Agricultural Income ie. tax on
₹8,40,000 (as computed
already):
On ₹3 lakhs -
On income between ₹3 lakhs -
₹5 lakhs, tax @ 5% 10,000
On income between ₹5 lakhs
to ₹8.4 lakhs , tax @ 20% 68,000
194
Total 78,000
2. Tax on Basic Exemption Limit +
Net Agricultural Income i.e. on
₹3,00,000 + ₹3,10,000 =
₹6,10,000
On first ₹3,00,000 -
On ₹3 lakhs - ₹5 lakhs 10,000
On income between ₹5 lakhs -
₹10,00,000 22,000
Total 32,000
3. Tax under step 1 (-) tax under
step 2 46,000
4. Add: H&EC @ 4% 1,840
5. Total tax liability 47,840

195
OTHER EXEMPTED INCOMES

1. Amounts received by a member from the income of HUF


{Section 10(2)}

2. Share Income of Partner {Section 10(2A)}

3. Interest on money standing to the credit of individual in his


NRE Account. {Section 10(4)(ii)}

4. REMENERATION RECEIVED BY INDIVIDUALS, WHO ARE NOT


CITIZENS OF INDIA: {Section 10(6)}
a. Remuneration received by officials of Embassies, etc of
foreign states {Section 10(6)(ii)}
b. Remuneration received for services rendered in India as an
employee of foreign enterprise {Section 10(6)(vi)} –
condition: employee’s stay in India should not exceed 90 days
c. Salary received by a non-citizen non-resident for services
rendered in connection with employment on foreign ship.
d. Remuneration received by Foreign Government employees
during their stay in India for specified training.

5. Compensation received on account of disaster. {Section


10(10BC)}
Received from Central Government, State Government or Local
Authority.

6. Payment from Sukanya Samriddhi Account {Section 10(11A)}

7. Educational Scholarships {Section 10(16)}

196
8. Payment to MPs/ MLAs {Section 10(17)}:
Exemption of Daily Allowance, Constituency allowance of MPs
and constituency allowance of MLAs.

9. Awards for literary, scientific and artistic works and other


awards by Government {Section 10(17A)}

10. Pension received by recipient of gallantry awards {section


10(18)}

11. Specified income of Sikkimese individual {Section 10(26AAA)}:


The following income, which arises or accrues to a Sikkimese
individual, would be exempt from Income tax –
a. Income from any source from state in Sikkim
b. Income by way of dividend or interest on securities.
However, this exemption will not be available to a Sikkimese
woman who, on or after 01.04.2008, marries a non-Sikkimese
individual.

12. TAX HOLIDAY FOR UNITS ESTABLISHED IN SPECIAL ECONOMIC


ZONES {Section 10AA}

• It should begin to manufacture or produce articles or things


or provide any service on or after 1.4.2005 but before
1.4.2021.
• Exemption is allowed for consecutive 15 years in respect of
profits derived from exports of such articles or things or
export of services.
• Amount of Exemption =
Profits of Unit in SEZ X Export Turnover of SEZ unit
Total Turnover of SEZ Unit
• Exemption allowed:
- 100% of such profits for first 5 years
- 50% in next 5 years
197
- 50% in last 5 years subject to transfer to SEZ Re-
investment Reserve Account.
ILLUSTRATION 2:
Rudra Ltd. has one unit at SEZ and another unit at Domestic Tariff Area
(DTA). The company provides the following details for the previous year
2022-23:
PARTICULARS Rudra Ltd. (₹) Unit in DTA (₹)
a. Total Sales 6,00,00,000 2,00,00,000
b. Export Sales 4,60,00,000 1,60,00,000
c. Net Profit 80,00,000 20,00,000
Calculate the eligible deduction under section 10AA of Income Tax Act,
1961 for the Assessment Year 2023-24 in following situations:
a. If both the units were set up and started manufacturing from
22.05.2014
b. If both Units were set up and started manufacturing from
14.05.2018.
SOLUTION:
ONLY HINT IS GIVEN -
a. If both the units were set up and started manufacturing from
22.05.2014:
₹60 lakhs X 300 lakhs X 50%
400 lakhs
= ₹22.50 lakhs
b. If both Units were set up and started manufacturing from
14.05.2018.
₹60 lakhs X 300 lakhs
400 lakhs
= ₹45 lakhs

198
Note: Turnover of SEZ Unit is computed as Total Turnover of
Rudra (-) Turnover of DTA unit

SPACE FOR OWN NOTES:

199
COMPUTATION OF TOTAL INCOME AND TAX LIABILITY
For Individuals:
Step 1: Determine the Residential Status

Step 2: Classify the Income in 5 heads of Income

Step 3: Compute Income under each head

Step 4: Club the income of spouse, minor child, etc.

Step 5: Set off current year losses and brought forward losses

Step 6: Get to Gross Total Income

Step 7: Claim Deductions under chapter VI-A

Step 8: Compute Total Income {Round off to nearest of ₹10 u/s


288A}

Step 9: Apply rates of tax on total income in case of an individual

Step 10: Apply Surcharge & Rebate

Step 11: Add Health and Education Cess @ 4% to sub-total of “Tax


+ Surcharge”

Step 12: Examine the applicability of Alternate Minimum Tax


(AMT)

Step 13: Examine whether or not to exercise the option under


section 115BAC for availing Alternative Tax Regime.

Step 14: Consider credit for Advance Tax, TDS & TCS

Step 15: Compute Tax payable {Total Tax payable (-) Tax Credit}
200
Rates of Tax:
TOTAL INCOME Rate of Tax
Upto ₹2,50,000
Upto ₹3,00,000 {Resident Individual whose age is
60 or more than 60 during the P.Y. but not 80}
Upto ₹5,00,000 {Resident individual whose age is NIL
80 or more than 80 during the P.Y.}
₹2,50,000/ ₹3,00,000, as the case may be to 5%
₹5,00,000
₹5,00,000 to ₹10,00,000 20%
Above ₹10,00,000 30%

SURCHARGE AND REBATE:


Total Income {assuming that the same
does not include dividend, LTCG u/s SURCHARGE
112A, 112 & STCG u/s 111A}
> ₹50 lakhs < ₹1 crore 10% of Income tax
> ₹1 crore < ₹2 crore 15% of Income tax
> ₹2 crore < ₹5 crore 25% of Income tax
> ₹5 crore 37% of Income tax

NOTE:

Enhanced rates of surcharge @ 25% and 37% will not apply in


respect of dividend income, long term capital gains taxable u/s
112A, 112 and Short-Term Capital Gains u/s 111A.

Concept of Marginal Relief {See Illustrations}

Rebate u/s 87A: Rebate of upto ₹12,500 for RESIDENT INDIVIDUALS


having total income upto ₹5,00,000.
Note: This rebate u/s 87A is NOT AVAILABLE in respect of tax
payable u/s 112A i.e. @ 10% on LTCG on shares of listed
companies.

201
ALTERNATE MINIMUM TAX

• Any person other than a company, who has claimed deduction under
section 10AA, 35AD or 80-IA to 80RRB (except 80P) would be subject
to AMT.
Note: Further individual will be subject to AMT if his Adjusted Total
Income (ATI) > ₹20 lakhs.

• Compute AMT @ 18.5% of Adjusted Total Income + SC, if applicable +


H&EC @ 4%.

• If AMT > tax computed as per normal provisions of income tax Act,
then pay such tax.

• Tax Credit to be carried forward if AMT is payable = AMT (-) Tax


payable as per normal provisions.

• Individual OPTING FOR TAX u/s 115BAC i.e. Alternative Tax Regime are
NOT LIABLE to AMT.

• How to compute Adjusted Total Income?


PARTICULARS Amount (₹) Amount (₹)
a. Total Income as per provisions
of Income Tax Act xxxxx
b. Add: Deduction u/s 35AD xxxx
Add: Deduction u/s 10AA xxxx
Add: Deductions u/s 80-IA to
80 RRB (except u/s 80P) xxxx xxxx
c. Sub Total xxxx
d. Less: Depreciation u/s 32
w.r.t. assets used in specified
business for which deduction
u/s 35AD is claimed (xxxx)
e. Therefore, Adjusted Total
Income xxxxx

202
ILLUSTRATION 1: {Solve after doing provisions of section 115BAC}

Mr. X, an individual set up a unit in Special Economic Zone (SEZ) in the


Financial Year 2018-19 for production of washing machines.
The unit fulfills all the conditions of section 10AA of the Income Tax
Act, 1961.
During the Financial Year 2021-22, he also set up a warehousing facility
in the district of Tamil Nadu for storage of agricultural produce. It
fulfills all the conditions of section 35AD. Capital expenditure in
respect of warehouse amounted to ₹75 lakhs (including cost of land:
₹10 lakhs). The warehouse became operational with effect from 1st
April, 2022 and the expenditure of ₹75 lakhs was capitalized in the
books on that date.

Relevant details for the Financial Year 2022-23 are as follows:


PARTICULARS Amount (₹)
Profit of unit located in SEZ 40,00,000
Export sales of above unit 80,00,000
Domestic Sales of above unit 20,00,000
Profits from operations of warehousing facility
{Before considering deduction u/s 35AD} 1,05,00,000
Compute Income Tax (including AMT u/s 115JC) liability of Mr.X for the
Assessment Year 2023-24 both as per regular provisions of the Income
Tax Act and as per section 115BAC for the Assessment Year 2023-24.
Advise Mr.X whether he should opt for section 115BAC.

SOLUTION:
Computation of Total Income and tax liability of Mr.X for the
Assessment Year 2023-24 (Relevant to Previous Year 2022-23) under
the regular provisions of the Income Tax Act:
PARTICULARS Amount (₹) Amount (₹)
Profits and Gains from Business
or Profession
Profit from unit in SEZ 40,00,000
Less: Deduction u/s 10AA (Note) (32,00,000) 8,00,000
Profits from operation of
Warehousing facility 1,05,00,000
203
Less: Deduction under section
35AD (Note) (65,00,000) 40,00,000
Therefore, Gross Total
Income/Total Income 48,00,000
Computation of tax liability as
per regular provisions
On income of ₹2.5 lakhs -
On income between ₹2.5 lakhs to
₹5 lakhs, tax @ 5% 12,500
On income between ₹5 lakhs -
₹10 lakhs, tax @ 20% 1,00,000
On income above ₹10 lakhs, tax
@ 30% 11,40,000
Total 12,52,500
Add: Health and Education Cess
@ 4% 51,100
Total Tax liability 13,02,600
Computation of Adjusted Total
Income
Total Income as per provisions of
Income Tax Act 48,00,000
Add: Deduction u/s 10AA 32,00,000
Add: Deduction u/s 35AD 65,00,000
Less: Depreciation u/s 32 on
₹65,00,000 @ 10% being building (6,50,000) 1,38,50,000
Alternate Minimum tax @ 18.5% 25,62,250
Add: Surcharge @ 15% {As the
ATI is exceeding ₹1 crore but not
exceeding ₹2 crore} 3,84,338
Sub Total 29,46,588
Add: H&EC @ 4% 1,17,864
Total 30,64,452
TAX PAYABLE {Higher of normal
tax or AMT} – Rounded off to
nearest of ₹10 u/s 288B 30,64,450
AMT Credit = 30,64,450 (-) 13,02,600 = ₹17,61,850
204
COMPUTATION OF TOTAL INCOME AND TAX LIABILITY OF MR.X FOR
THE ASSESSMENT YEAR 2023-24 (RELEVANT TO PREVIOUS YEAR
2022-23) UNDER PROVISIONS OF SECTION 115BAC:
PARTICULARS Amount (₹) Amount (₹)
Total Income as per normal
provisions of the Act 48,00,000
Add: Section 10AA deduction – not
allowed 32,00,000
Add: section 35AD deduction – not
allowed {So, allow section 32
depreciation on building} Therefore,
add back ₹65,00,000 (-) ₹6,50,000 58,50,000
TOTAL INCOME for computing tax u/s
115BAC 1,38,50,000
Tax u/s 115BAC
On income upto ₹2.5 lakhs -
On income between ₹2.5 lakhs - ₹5
lakhs, tax @ 5% 12,500
On income between ₹5 lakhs - ₹7.5
lakhs, tax @ 10% 25,000
On income between ₹7.5 lakhs - ₹10
lakhs, tax @ 15% 37,500
On income between ₹10 lakhs - ₹12.5
lakhs, tax @ 20% 50,000
On income between ₹12.5 lakhs - ₹15
lakhs, tax @ 25% 62,500
On income above ₹15 lakhs, tax @
30% 37,05,000
{1,23,50,000 * 30%}
Total 38,92,500
Add: SC @ 15% 5,83,875
Therefore, Total 44,76,375
Add: H&EC @ 4% 1,79,055 46,55,430
NOTE:
Individuals/HUF exercising for option of section 115BAC are not
liable to pay Alternate Minimum Tax u/s 115JC.
205
DECISION:
Since the tax liability of Mr.X under normal provisions of the Act is
lower than tax liability as computed u/s 115BAC, it would be beneficial
NOT TO OPT FOR SECTION 115BAC for A.Y. 2023-24 in this case.
Moreover, the benefit of AMT Credit is also available to the extent of
tax paid in excess of tax as per normal provisions.

NOTES:

1. Deduction u/s 10AA = ₹40,00,000 X 80,00,000


1,00,00,000
= ₹32,00,000

2. Deduction under section 35AD is NOT AVAILABLE on expenditure


incurred on acquisition of land. Therefore, allowable deduction
u/s 35AD = ₹75 lakhs (-) ₹10 lakhs.

Section 115BAC: Alternative Tax regime for individual/HUF:

Under this tax regime, income tax shall be computed at the option
of assessee as per rate given in the following table: -

Total Income Rate of Tax


Upto Rs.2,50,000 Nil
Rs.2,50,000 to Rs.5,00,000
5%
Rs.5,00,000 to Rs.7,50,000 10%
Rs.7,50,000 to Rs.10,00,000 15%
Rs.10,00,000 to 20%
Rs.12,50,000
Rs.12,50,000 to 25%
Rs.15,00,000
Above Rs.15,00,000 30%

Notes:

1. Exemption limit for any individual is Rs.2,50,000 under this


alternative tax regime. In short, the higher exemption limit of

206
Rs.3,00,000 in case of resident senior citizen and Rs.5,00,000 in
case of resident senior citizen is not applicable.

2. Rebate under section 87A is applicable for resident whose


taxable income does not exceed Rs.5,00,000. It is to the extent
of 100% of income tax or Rs.12,500, whichever is lower.

3. Special income will be taxed at special rates and not as per slabs
even if individual has opted for the tax regime under section
115BAC.

4. Surcharge applicable under the existing tax regime is also


applicable in the case of alternative tax regime under section
115BAC.

5. INCENTIVES NOT AVAILABLE if assessee opts for the alternative


tax regime under section 115BAC:

Assessee opting for tax regime under section 115BAC is not


entitled to take exemptions/deductions of the following while
computing total income:-
a. Leave Travel Concession under section 10(5)

b. House Rent Allowance

c. Special allowances of section 10(14)

d. Allowance to MP/MLA
e. Exemption upto Rs.1,500 available in case of clubbed income of
minor child
f. Special Economic Zone (Section 10AA)

g. Standard Deduction

h. Entertainment Allowance

i. Profession Tax

j. Interest on Housing Loan in case of one or two self-occupied


properties
k. Additional depreciation

l. Investment Allowance under section 32AD

207
m. Tea/Coffee/Rubber development account (Section 33AB)
n. Site restoration fund
o. Deduction for scientific research under section 35
p. Capital expenditure pertaining to specified business
q. Agriculture extension project
r. Standard deduction in case of family pension
s. Deductions under section 80C to 80U {Except employer’s
contribution towards NPS under section 80CCD (2),
deduction under section 80JJAA and deduction under section
80LA(1A)} .

6. The total income of individual/HUF is calculated without


adjusting brought forward loss (and/or additional depreciation)
from any earlier year if such loss/additional depreciation
pertains to any deduction under the aforesaid sections.

7. Brought forward loss/ depreciation as mentioned above shall be


deemed to have been given full effect to and no further
deduction for such loss/ depreciation shall be allowed for any
subsequent year.

8. Alternate Minimum Tax (AMT) under section 115JC is not


applicable for the assessee opting for alternative tax regime of
section 115BAC. Consequently, even AMT Credit of earlier years
cannot be adjusted against the tax liability which is computed
under section 115BAC.

9. For assessee not having business or profession income, the


option must be exercised along with Income Tax Return for
every Previous Year.
However, if assessee has business/profession income, once this
option is exercised for any previous year, it remains valid for
subsequent years. However, the assessee cannot withdraw the
option.
208
ILLUSTRATION: -

1. Mrs. Dhanashree is a salaried employee in Krishna Private Limited


earning Gross Taxable Salaries of Rs.8,00,000 during Previous
Year 2022-23. Profession Tax paid by her is Rs.2,500. Further, she
deposited Rs.50,000 in Public Provident Fund in said Previous
Year.

Considering the above, answer the following questions:

a. Compute her Net Total income assuming that she has


opted for paying tax under alternative regime of section
115BAC.

b. Compute her net taxable salaries assuming that she has


opted for paying tax under alternative tax regime of section
115BAC.

c. Compute her Net Total income assuming that she is paying


tax as per normal provisions of the Act.
d. Compute her net taxable salaries assuming that she is
paying tax as per normal provisions of the Act.

e. Compute her tax liability assuming that she has opted for
paying tax as per alternative tax regime under section
115BAC.

f. Compute her tax liability assuming that she has NOT


opted for paying tax as per alternative tax regime under
section 115BAC i.e. she is paying tax as per normal
provisions of the Act.

Hints for above {Solutions}:


1. Where alternative tax regime is opted of section 115BAC, no

deductions are allowed w.r.t. “Standard Deduction” and


209
“Profession Tax” while computing Salaries. Therefore, Net
Taxable Salary is Rs.8,00,000.
Further, even deduction under section 80C w.r.t. PPF is also
not allowed. Therefore, Gross Taxable Salary itself is equal
to Total Income i.e. Rs.8,00,000.
Tax on Rs.8,00,000 as per section 115BAC for Assessment
Year 2023-24:
PARTICULARS Amount (Rs.)
Rs.0 to Rs.2,50,000 Nil
Rs.2,50,000 to Rs.5,00,000, Tax @ 12,500
5%
Rs.5,00,000 to Rs.7,50,000, Tax @ 25,000
10%
Rs.7,50,000 to Rs.8,00,000, Tax @ 7,500
15%
Total 45,000
Add: Health and Education Cess @ 1,800
4%
Total Tax 46,800
2. Where, Alternative tax regime is not opted, deductions
could be claimed as usual and accordingly, total income
and tax liability shall be computed as under for Assessment
Year 2023-24:
PARTICULARS Amount (Rs.)
a. Gross Taxable Salary 8,00,000
b. Less: Deductions under section 16
(ia) Standard deduction (50,000)
(ii) Entertainment Allowance -
(iii) Profession Tax (2,500)
c. Net Taxable Salary 7,47,500
d. Less: Deductions under chapter VI-A
U/S 80C (PPF deposit) (50,000)
e. Therefore, Total Income 6,97,500
f. Tax on above
0 to Rs.2,50,000 -
Rs.2,50,000 to Rs.5,00,000, Tax @ 5% 12,500
Rs.5,00,000 to Rs.6,97,500, Tax @ 20% 39,500
Total 52,000
Add: Health and Education Cess @ 4% 2,080
Therefore, Total Tax liability 54,080

2. Mr.Rohan, a businessman has income from business


Rs.14,00,000 during previous year 2022-23. Besides, he has interest
on savings bank account of Rs.21,000. He annually contributes
Rs.1,50,000 towards Public Provident Fund.
210
Based on the above information, answer the following questions:

a. Compute total income of Mr. Rohan assuming that he


does not opt for alternative tax regime of section
115BAC.

b. Compute total income of Mr. Rohan assuming that he opts


for alternative tax regime of section 115BAC.

c. Compute tax liability of Mr. Rohan assuming that he does


not opt for alternative tax regime of section 115BAC.

d. Compute tax liability of Mr. Rohan assuming that he


opts for alternative tax regime of section 115BAC.

Solution {Hint}:

Computation of Total Income and tax liability of Mr. Rohan for


Assessment Year 2023-24 (Relevant to Previous Year 2022-23):
PARTICULARS Existing Tax Alternative Tax
Regime Regime
Amount (Rs.) Amount (Rs.)
a. Profits and Gains from business or
Profession 14,00,000 14,00,000
b. Income from Other Sources (Interest on
Savings Bank Account) 21,000 21,000
c. Gross Total Income 14,21,000 14,21,000
d. Less: Deductions under chapter VI-A
u/s 80C (Deposit in PPF) 1,50,000 Not Available
u/s 80TTA (Interest on Savings
Account restricted to Rs.10,000 p.a.) 10,000 Not Available
e. Total Income 12,61,000 14,21,000
f. Tax liability:
g. Upto Rs.2.5 lakhs Nil Nil
h. Rs.2.5 lakhs to Rs.5 lakhs, Tax @ 5% 12,500 12,500
i. 1,00,000 25,000
(Rs.5 lac to Rs.10 (Rs.5 lac to Rs.7.5
lac, lac,
Tax @ 20%) Tax @ 10%)
j. 78,300 37,500
(Above Rs.10 lac, (Rs.7.5 lacs to
Tax @ 30%) Rs.10 lacs, Tax @

211
15%)

k. 50,000
(Rs.10 lac to
Rs.12.50
lac, tax @ 20%
l. 42,750
(Rs.12.50 lacs to
Rs.14.21 lacs, tax
@25%)
m. Total 1,90,800 1,67,750
n. Add: Health and Education Cess @ 4% 7,632 6,710
o. Total Tax 1,98,432 1,74,460
p. Rounded off to nearest of Rs.10 u/s 288B 1,98,430 1,74,460

Illustration:
What is the basic exemption limit to be considered while
computing tax liability of resident senior citizen who has
opted for alternative tax regime of section 115BAC of
Income Tax Act?

Solution:
₹2,50,000

Illustration:
What is the basic exemption limit to be considered while
computing tax liability of resident super senior citizen who
has opted for alternative tax regime of section 115BAC of
Income Tax Act?

Solution:
₹2,50,000

212
FILING OF RETURNS

Section 139(1)
Assessees required to file return of income compulsorily:
i. Companies & firms {whether having profit or loss or nil income}
ii. A person being a resident other than not ordinary resident, having
any asset located outside India held as a beneficial owner or
beneficiary or who has signing authority in any account located
outside India, whether or not having income chargeable to tax.
iii. Individuals/HUFs/BOIs/AOPs and Artificial Juridical Persons whose
total income before giving effect to provisions of chapter VI-A and
section 54, 54B, 54D, 54EC or 54F exceeds the basic exemption limit.
iv. Any person who during the previous year:
a. Has deposited more than ₹1 crore in one or more current accounts
maintained with a banking company or a co-operative bank or;
b. Has incurred expenditure of more than ₹2 lakh for himself or any
other person for travel to a foreign country or;
c. Has incurred expenditure of more than ₹1 lakh towards
consumption of electricity or;
d. Fulfills such other conditions as may be prescribed.
v. the deposit in one or more savings bank account of the person, in
aggregate, is ₹50 lakhs or more during the previous year.

Due date of filing of return of income:


Case Due Date
st
a. A company 31 October of
b. A person (other than a company) whose the
accounts are required to be audited Assessment
c. A partner of a firm whose accounts are Year.
required to be audited.

213
d. In any other case 31st July of the
Assessment
Year

Interest u/s 234F:


Fees for default in furnishing return of income:

- Where a person who is required to furnish a return of income


under section 139(1), fails to do so within the prescribed time limit
under section 139(1), he shall pay, by way of fee, a sum of ₹5,000.

- However, if the total income of the person does not exceed


₹5,00,000, the fees payable shall not exceed ₹1,000.

RETURN OF LOSS {Section 139(3)}


An assessee can carry forward or set off his/ its losses provided he/it
files his/its ITR u/s 139(3) within the due date specified in section
139(1).
Exceptions:
Loss from house property and unabsorbed depreciation can be carried
forward for set off even though return has not been filed before the
due date.

BELATED RETURN {Section 139(4)}

A return of income for any previous year, which has not been
furnished within the time allowed under section 139(1), may be
furnished at any time before the:
a. Three months prior to the end of relevant assessment year {i.e.
31.12.2023 for P.Y. 2022-23} OR;
b. Completion of assessment,
whichever is earlier.

214
REVISED RETURN: {Section 139(5)}

If any omission or any wrong statement is discovered in a return


furnished u/s 139(1) or belated return u/s 139(4), a revised return may
be furnished by the assessee at any time before the:
a. Three months prior to the end of relevant assessment year {i.e.
31.12.2023 for P.Y. 2022-23} OR;
b. Completion of assessment,
whichever is earlier.

Thus, belated return can also be revised.

215
SOME IMPORTANT QUESTIONS!!!
{MIXED BAG}

SCOPE OF TOTAL INCOME


1. Mrs. Roma, an Indian Citizen, is a government employee working for the Indian
Government. She submits the following information for the previous year ending
31.03.2023:


1 Salary income received in Malaysia for services rendered there 2,00,000
2 Profit from business carried on in Orissa 80,000
3 Loss from business carried on in Baroda (20,000)
4 Profit from business carried on in Paris (income is earned and 42,000
received in Sydney and business is controlled from Paris)
5 Loss from business carried on in Canada (though profits are not (46,000)
received in India, business is controlled from Dehradun)
6 Unabsorbed depreciation of business in Canada 16,000
7 Profit from Indonesia business (controlled form Delhi) and 60% of
profit deposited in a bank in Indonesia and 40% received in India 70,000
8 Rent from house property situated in Canada and received in
Canada 1,92,000

Determine the gross total income of Roma for the A.Y. 2023-24 ignoring the provisions of
section 115BAC on the assumption that she is:
(1) Resident but not ordinarily resident in India

(2) Non-resident in India.

SOLUTION:
Computation of gross total Income of Mrs. Roma for the A.Y. 2023-24:
Particulars of income Resident Non-
but not Resident
ordinarily (₹)
Resident
(₹)
1 Salary income received in Malaysia for services 2,00,000 2,00,000
rendered there (Note 1)
Less: Standard deduction under section 16(ia) 50,000 50,000
1,50,000 1,50,000
2 Profit from business carried on in Orissa [Since it 80,000 80,000
accrues or arises in India]
3 Loss from business carried on in Baroda [Since it (20,000) (20,000)
accrues or arises in India]
4 Profit from business carried on in Paris (income is Nil Nil
216
earned and received in Sydney and business is
controlled from Paris) [Since it accrues or arises
outside India]
5 Loss from business carried on in Canada (business is (46,000) Nil
controlled from Dehradun)
6 Unabsorbed depreciation of business in Canada (16,000) Nil
7 Profit from Indonesia business (business is controlled 70,000 28,000
from Delhi)
8 Rent from property situated in Canada and received in Nil Nil
Canada
Gross Total Income 2,18,000 2,38,000

Note 1 - Income from “Salaries” payable by the Government to a citizen of India


forservices rendered outside India is deemed to accrue or arise in India as per
section 9(1)(iii). Standard deduction under section 16(ia) is allowable, irrespective of
residential status.

Note 2 – In case of a non-resident, only income received or deemed to be received


inIndia and income accruing or arising or deemed to accrue or arise in India is chargeable
to tax. However, in case of a resident but not ordinarily resident, income derived from a
business controlled in or profession set up in India is also taxable even though it accrues
or arises outside India.
Therefore, income referred to in S. No. 1, 2 and 3 are taxable in the hands of Mrs. Roma
in both cases if she is a resident but not ordinarily resident or if she is a non-resident.
Loss from business carried on in Canada, unabsorbed depreciation of business in Canada
and Profit from Indonesia business would be fully chargeable to tax in India ifshe is a
resident but not ordinarily resident as it derived from a business controlled in India.
However, Profit from Indonesia business is taxable in case of non-resident to the extent
of such profits received in India.

INCOME FROM HOUSE PROPERTY


2. Mr. Akash owns a residential house property whose Municipal Value, Fair Rent and
Standard Rent are ₹1,60,000, ₹1,70,000 and ₹1,90,000, respectively. The house has two
independent units. Unit I (25% of floor area) is utilized for the purpose of his
profession and Unit II (75% of floor area) is let out for residential purposes at a monthly
rent of ₹8,500. Municipal taxes @8% of the Municipal Value were paid during the year
by Mr. Akash. He made the following payments in respect of the house property during
the previous year 2022-23:
Light and Water charges ₹2,000, Repairs ₹1,45,000, Interest on loan taken for the repair
of property ₹36,000. Mr. Akash has taken a loan of ₹5,00,000 in July, 2016 for construction
of the above house property. Construction was completed on 30th June, 2019. He paid
interest on loan @12% per annum and every month such interest was paid. No
repayment of loan has been made so far.

217
Income of Mr. Akash from his profession amounted to ₹8,00,000 during the year
(without debiting house rent and other incidental expenditure including admissible
depreciation of ₹8,000 on the portion of house used for profession).
Determine the Gross total income of Mr. Akash for the A.Y. 2023-24 ignoring the
provisions of section 115BAC.

SOLUTION:
Computation of Gross total income of Mr. Akash for the A.Y. 2023-24

Particulars ₹ ₹
I Income from House Property
Unit-II (75% of floor area)
Gross Annual Value
(a) Actual rent received (₹8,500 x 12) ₹1,02,000
(b) Expected rent ₹1,27,500
[Higher of municipal value (i.e. ₹1,60,000) and fair rent
(i.e. ₹1,70,000) but restricted to standard rent (i.e.
` 1,90,000) ₹1,70,000 x 75%]
Higher of (a) or (b) is GAV 1,27,500
Less: Municipal taxes (₹1,60,000 x 8% x 75%) 9,600
NAV 1,17,900
Less: Deductions u/s 24
(a) 30% of NAV ₹35,370
(b) Interest on loan (See note) ₹96,750 1,32,120 (14,220)

II Profits & Gains of business &


profession
Income from Profession 8,00,000
of
Less: Light & Water Charges (25% ₹ 500
₹2,000)
Municipal taxes (25% of ₹12,800) ₹ 3,200
Repairs (25% of ₹1,45,000) ₹ 36,250
Interest on loan taken for repair (25% of ₹ 9,000
₹ 36,000)
Interest on loan taken for construction of ₹ 15,000
house property (25% of ₹60,000)
Depreciation ₹ 8,000 71,950 7,28,050
Gross Total Income 7,13,830

Note:

218
Computation of Interest on loan


Interest for the year (₹5,00,000 x 12%) 60,000
Pre-construction period Interest-
12% of ₹5,00,000 for 33 months = ₹1,65,000
To be allowed in 5 equal instalments from the year of completion 33,000
(₹1,65,000 x 1/5)
Interest on loan taken for repair (no restriction for let out property) 36,000
Total Interest deduction u/s 24(b) 1,29,000
Total Interest deduction u/s 24(b) for let out property 96,750
(75% X ₹1,29,000)

CAPITAL GAINS:
3. Mr. Suresh has a sole proprietary manufacturing unit. On 1st April, 2022, he owns Plant
A and Plant B (rate of depreciation 15%). Depreciated value of the block on 1 st April,
2022 is ₹10,00,000. Plant B is transferred on 15th October, 2022 for ₹19,00,000. Expenditure
on transfer of Plant B is ₹20,000. Plant C (rate of depreciation 15%) is purchased on 10th
March, 2023 for ₹22,00,000. However, Plant C is put to use on 2nd September, 2023. Business
income of Mr. Suresh before claiming any depreciation is ₹11,00,000.
On 1st March, 2023, Mr. Suresh transfers 900 equity shares in A Ltd. (unlisted) for
₹23,50,000. Mr. Suresh does not own any residential house property. These shares
were purchased on 2nd April, 2015 for ₹2,00,000. To avail of the benefit of exemption
under different sections, he made the following investments on 1st May, 2023.
(i) A residential house property at Kolkata: ₹19,00,000 (out of which stamp duty
expenditure is ₹30,000).
(ii) NHAI bonds: ₹3,00,000.
Find out the gross total income of Mr. Suresh for the A.Y. 2023-24.
CII – F.Y. 2022-23: 331; F.Y. 2015-16:254

SOLUTION:
Computation of gross total income of Mr. Suresh for the A.Y. 2023-24:

Particular Amount Amount


s (₹) (₹)
Profits and gains of business or profession
Business income before depreciation 11,00,000
Depreciated value of the block on April 1, 2022 10,00,000
Add: “Actual cost” of Plant C acquired on March 10, 22,00,000
2023
Less: Sale Consideration of Plant B 19,00,000

219
Written down value on March 31, 2023 13,00,000
Normal depreciation (not available as Plant C is not put Nil
to use during the P.Y. 2022-23)
Additional depreciation (not available as Plant C is not Nil
put to use during the P.Y. 2022-23)
Capital Gains
Long term capital gain on transfer of unlisted
equity shares [Since shares were held for more than
24 months]
Sale consideration 23,50,000
Less: Indexed Cost of Acquisition [2,00,000 x 331/254] 2,60,630
20,89,370
Less: Exemption under section 54EC Nil
[Deduction under section 54EC is allowable only in
respect of long term capital gain on transfer of land and
building]
Exemption under section 54F 16,89,278 16,89,278
[20,89,370 x 19,00,000/23,50,000]
4,00,092
Gross Total Income 15,00,092

SET OFF & CARRY FORWARD OF LOSSES:

4. Mr. Ram, a resident Individual aged 65 years, submits the following details of his income
for the assessment year 2023-24:
Particulars ₹
Loss from speculative business A 30,000
Income from speculative business B 1,50,000
Loss from specified business covered under section 35AD 20,000
Income from Salary (computed) 2,00,000
Loss from let out house property 1,90,000
Loss from cloth business 80,000
Long-term capital gain from sale of urban land 3,00,000
Long-term capital loss on sale of shares (STT not paid) 1,00,000
Long-term capital loss on sale of listed shares in recognized stock 1,50,000
Exchange (STT paid at the time of acquisition and sale of shares)
Income from betting (Gross) 80,000
Loss from gambling 8,000
Interest on saving bank deposits 12,000
Interest on fixed deposits with banks 40,000
Compute the total income of Mr. Ram and show the items eligible for carry forward,
assuming that he does not opt for the provisions of section 115BAC.

220
SOLUTION:
Computation of total income of Mr. Ram for the A.Y. 2023-24

Particulars Amount (₹) Amount (₹)


Salaries
Income from Salary 2,00,000
Less: Loss from house property set-off against salary 1,90,000
10,000
Profits and gains from business or profession
Income from speculative business B 1,50,000
Less: Loss of ₹30,000 from speculative business A 30,000
Less: Loss from cloth business [Loss from non-
speculative business can be set off against profits from
speculative business] 80,000
40,000
Capital Gains
Long-term capital gain from sale of urban land 3,00,000
Less: Long-term capital loss on sale of shares (STT not 1,00,000
paid)
Less: Long-term capital loss on sale of listed shares in
recognizes stock exchange (STT paid at the time of
acquisition and sale of shares) 1,50,000
50,000

221
Income from Other Sources
Income from betting 80,000
Interest on savings bank deposits 12,000
40,000 1,32,000
Interest on fixed deposits with banks
Gross Total Income 2,32,000
Less: Deduction under section 80TTB (Maximum being 50,000
` 50,000, since Mr. Ram is a senior citizen)
Total Income 1,82,000
Notes:
(i) Loss from specified business covered under section 35AD can be set off onlyagainst
profits and gains of any other specified business. Therefore, such losscannot be set
off against any other income. The unabsorbed loss of ` 20,000 has tobe carried forward
for set-off against profits and gains of any specified business in the following year.
(ii) Loss from gambling can neither be set off against any other income, nor can be carried
forward.

CLUBBING OF INCOME:
5. Examine the tax implication of each transaction and compute the total income
of Mr. Tushar and Mrs. Tushar and their minor son for the assessment year 2023-24,
assuming they do not wish to opt for section 115BAC.
(1) Mr. Tushar has a fixed deposit of ₹6,00,000 in State bank of India. He instructed
the bank to credit the interest on the deposit @9% from 1st April, 2022 to 31st
March, 2023 to the savings bank account of Mr. Raj, son of his brother, to help him
in his education.
(2) Mr. Tushar started a proprietary business on 1st May, 2022 with capital of
₹6,00,000. His wife, Mrs. Tushar, a software Engineer, gave cash of ₹5,00,000 on 1st
May, 2022, which was immediately invested in the business by Mr. Tushar. He
earned a profit of ₹4,00,000 during the previous year 2022-23.
(3) Mr. Tushar's minor son derived an income of ₹20,000 through a business activity
involving application of his skill and talent.

SOLUTION:
Computation of total income of Mr. Tushar and Mrs. Tushar and minor son for the A.Y. 2023-24

Particulars Amount Amount Amount


(₹) (₹) (₹)
Mr. Tushar Mrs. Tushar Minor Son
Interest on Mr. Tushar fixed Deposit 54,000
with State bank of India (` 6,00,000 x
9%)
As per section 60, in case there is a
transfer of income without transfer of
assets from which such income is
222
derived, such income shall be treated
as income of the transferor. Therefore,
the fixed deposit interest of ` 54,000
transferred by Mr. Tushar to Mr. Raj
shall be included in the total income of
Mr. Tushar
Profit for P.Y. 2022-23 to be 2,18,182 1,81,818
apportioned on the basis of capital
employed on the first day of previous
year i.e. as on 1st May, 2022, since
business started on 1.5.2022 (6:5)
Share of income of Mr. Tushar [` 4,00,000 x
6/11]
Share of Income of Mrs. Tushar [` 4,00,000 x
5/11]
Section 64(1)(iv) of the Income-tax Act, 1961
provided for the clubbing of income in the
hands of the individual, if the income earned is
form the assets (other than house property)
transferred directly or indirectly to the spouse
of the individual, otherwise than for adequate
consideration or in connection with an
agreement to live apart.
Income of minor son through a business
activity involving application his skill and talent.
In case the income earned by minor child is on 20,000
account of any activity involving application of
any skill or talent, then, such income of the
minor child shall not be included in the income
of the parent, but shall be taxable in the hands
of the minor child.
Therefore, the income of ₹20,000 derived by
minor son through a business activity involving
the application of his skill and talent shall not
be clubbed in the hands of the parent. Such
income will be taxable in the hands of the
minor son.
Total Income 2,72,182 1,81,818 20,000

223
6. Mr. Kamal, a resident individual aged 48 years, is working at a senior management
position in a private bank since past 20 years. During the previous year 2022-23, he
received the following emoluments from the employer:
(a) Basic Salary ₹3,50,000 per month.
(b) Client entertainment reimbursement of ₹20,000 per month out of which he
submitted bills for ₹2,00,000 for the relevant year.
(c) Leave travel allowance of ₹4,00,000 per annum. He took a trip to Goa with his
spouse and two children in December 2022, for which plane boarding tickets
of ₹1,00,000 and hotel bookings of ₹3,00,000 were submitted to the employer.
(d) Performance bonus amounting to 20% of annual basic salary.
(e) He is eligible to take a staff housing loan upto ₹20,00,000 at a concessional rate of 2.5%
p.a. He availed a housing loan of ₹15,00,000 out of the same on 1st June 2022. No
repayment of loan has been made during the F.Y. 2022-23. The lending rate of SBI
as on 1.4.2022 for housing loan may be taken as 8% p.a.
(f) The Bank also allotted 1,500 sweat equity shares to Mr. Kamal in May 2022 at the
rate of ₹1,300 per share. The Fair market value of the share was ₹1,500 per share on
the date of exercise of option by Mr. Kamal. He sold all the shares for ₹2,100 per
share on 31.03.2023 on recognised stock exchange. Assume Securities transaction
tax has been paid.
The following transactions were made by Mr. Kamal during the previous year 2022-23:
(a) He earned rental income of ₹35,000 per month from a 3 BHK residential flat situated
at Delhi. He purchased the said flat for ₹45 Lakhs in June, 2022 using the housing loan
availed from the employer and his own savings. It was let out from July, 2022.
Municipal taxes of ₹12,000 for F.Y. 2022-23 was paid by Mr. Kamal.

(b) He invested ₹30,00,000 in RBI Floating Rate Savings Bonds on 1st September 2022
earning an interest of 7% p.a. Interest is credited half yearly on 1st January and
1st July every year. (Assume receipt basis for taxation)
(c) He also paid LIC premium of ₹15,000 for self, ₹20,000 for wife and ₹30,000 for dependent
father, aged 75 years. Medical insurance premium paid on the health of dependent
brother and major dependent son amounted to ₹5,000 (paid by cheque)and ₹10,000
(paid in cash), respectively.
(d) In December 2022, he earned dividend income of ₹5,00,000 (gross) on shares of the
bank held by him.
You are required to compute his total income and tax liability for the assessment year
2023-24, clearly showing all workings. (Ignore section 115BAC provisions)

224
SOLUTION:
Computation of total income of Mr. Kamal for the A.Y. 2023-24
Particulars Amount (₹) Amount (₹)
I Income from salaries
Basic Salary [₹3,50,000 x 12] 42,00,000
Client entertainment reimbursement [₹2,40,000 - 40,000
₹2,00,000]
Leave Travel Allowance [₹4,00,000 - ₹1,00,000] 3,00,000
[Note 1]
Performance Bonus (20% of Basic Salary) 8,40,000
Interest on Housing loan 68,750
[₹15,00,000 x (8% - 2.5%) x 10/12]
Sweat Equity allotted by the employer 3,00,000
(₹1,500 - ₹1,300) x 1,500
Gross Salary 57,48,750
Less: Standard deduction 50,000
Taxable Salary 56,98,750
II Income from house property
Gross Annual Value under section 23(1) [Rent 3,15,000
received for 9 months has been taken as the
Gross Annual Value in the absence of other
information relating to Municipal Value, Fair Rent
and Standard Rent] [` 35,000 x 9]
12,000
Less: Municipal taxes paid [Paid by Mr. Kamal]
Net Annual Value (NAV) 3,03,000
Less: Deduction u/s 24
90,900
(a) @30% of NAV
31,250
(b) Interest on borrowed capital [15,00,000 x
2.5%x 10/12] 1,80,850

III Capital gains 9,00,000


STCG on sale of sweat equity shares [1,500 X(`
2,100 - ` 1,500)]
IV Income from other sources
Dividend Income 5,00,000
Interest on RBI bonds [` 30,00,000 X 7% X 4/12) 70,000 5,70,000
Gross total Income 73,49,600
Less: Deduction under Chapter VI-A
Deduction u/s 80C for LIC premium paid for self 35,000
and wife [Note 2]
Deduction u/s 80 D [Note3]
Nil 35,000

Total Income 73,14,600

225
Computation of tax liability of Mr. Kamal for the A.Y. 2023-24:

Particulars Amount (₹) Amount (₹)


Tax on STCG u/s 111A @15% on ₹9,00,000 1,35,000
Tax on other income of ` 64,14,600
Upto ₹2,50,000 Nil
₹2,50,001 - ₹5,00,000 @5% 12,500
₹5,00,001 - ₹10,00,000 @20% 1,00,000
₹10,00,001 - ₹64,14,600 @30% 16,24,380 17,36,880
18,71,880
Add: Surcharge@10% since total income exceeds 1,87,188
₹50 lakhs but does not exceed ₹1 crore
20,59,068
Add: Health and Education cess @ 4% 82,363
Tax Liability 21,41,431
Tax Liability (Rounded off) 21,41,430
Notes:
(1) Hotel Bookings and lodgings are not covered under leave travel facility. Hence, only
₹1,00,000 of cost of tickets would be exempt under section 10(5).
(2) Premium for life insurance policy of father is not allowed as deduction under
section 80C.
(3) Medical insurance premium on the health of brother is not allowable since brother
does not come within the meaning of family u/s 80D. In case of son, premium is
paid in cash, hence, the same is not allowed.

PROCEDURAL:
7. Mr. Aakash has undertaken certain transactions during the F.Y.2022 -23, which are listed
below. You are required to identify the transactions in respect of which quoting of PAN is
mandatory in the related documents –

S.No. Transaction
1. Opening a current account with HDFC Bank
2. Sale of shares of ABC (P) Ltd. for ₹1,50,000
3. Purchase of two wheeler motor vehicle of ₹1 lakh
4. Purchase of a professional laptop of ₹3 lakhs

226
Transaction Is quoting of PAN mandatory in related
documents?
1. Opening a current account with Yes, quoting of PAN is mandatory on openingof a
HDFC Bank current account by a person with bank.
2. Sale of shares of ABC (P) Ltd.for ₹ Yes, since the amount for sale of unlisted
1,50,000 shares exceeds ₹1,00,000
3. Purchase of two wheeler motor Since the purchase is of two wheeler motor
vehicle of ₹1 lakh vehicle, quoting of PAN is not mandatory
4. Purchase of a professional Yes, since the amount paid exceeds
laptop of ₹3 lakhs ₹2,00,000

227

You might also like